Orthopedics MCQS ONLINE OITE23
1.03 What is the most common complication associated with surgical fixation of a transverse midshaft humeral fracture using antegrade statically locked medullary nail?
-
Infection
-
Radial nerve palsy
-
Brachial artery injury during distal interlocking
-
Shoulder pain
-
Nonunion
1.03 Answer 4.
Disadvantages include: increased shoulder pain and stiffness, possibility of impingement from proximally prominent hardware, and risk of fracture comminution during reaming or nail insertion. In the quoted study in JOT, 6/38 patients who underwent im nailing had shoulder discomfort where as 0/46 of the plate group had shoulder discomfort. The untold part of the study is that 6/46 of the plating group had elbow discomfort and 0/38 of the im nailing group had elbow pain.
2.03 A 62 yo woman with primary osteoarthritis is scheduled to undergo cementless total hip arthroplasty. History reveals that she underwent pelvic irradiation for uterine cancer 10 years ago The patient should be considered at highest risk for which of the following complications of total hip arthroplasty?
-
Deep venous thrombosis
-
Osteolysis
-
Heterotopic ossification
-
Hip instability
-
Aseptic loosening
-
2.03 Answer 5.
![]() |
The quoted study looked at 1319 patients who had insertion of a hemispherical porous coated acetabular component without cement. 12 hips in 11 patients were placed in previous irradiation of pelvis patients. 3 patients died after less than 1 year. That left 8 patients with 9 hips. Average time of follow-up was 37 months. 4 of those 9 failed at an average of 25 months. Their conclusion was that “the insertion of acetabular components without cement in a preciously irradiated pelvis has a high rate of failure. However, a superior method of acetabular
back to this question next question
reconstruction in this difficult situation has yet to emerge.”
3.03
![]() |
Which of the following factors contributes to intervetebral disk degeneration?
-
Increase in cell numbers
-
Increase in local pH
-
Increase in aggregating proteoglycans
-
Decrease in collagen cross linking
-
Decrease in nutritional transport
3.03 Answer 5.
Proteoglycan and water concentrations decrease and collagen and noncollagenous protein concentrations increase as dense granular material accumulates throughout the matrix. Mechanisms of intervertebral disk degeneration include: declining nutrient and waste product transport mechanisms, decreasing concentration of viable cells, cell senescence, apoptotic debris, loss of aggregating proteoglycans, modification of matrix proteins, degradative enzyme activity, accumulation of degraded
matrix macromolecules, and fatigue failure of the matrix. Declining nutritional transport appears to be the most critical event responsible for the changes in central disk cells and their matrices. PH actually decreases with age.
4.03
Compared with other classifications of Legg_Calve_Perthes disease, the lateral pillar classification has the advantage of
-
Exhibiting greater interobserver agreement
-
Remaining unchanged throughout the course of the disease
-
Quantifying the osteonecrosis visible on the frog lateral hip radiograph
-
Predicting proximal femoral growth arrest
-
Describing the degree of metaphyseal osteonecrosis
-
4.03 Answer 1.
![]() |
The cited article goes into a whole lot of statistical mumbo jumbo. The jist of the article says that since the time Herring et al did their article, No one has been able to reproduce their results as far as the interobserver and intraobserver is concerned.
This study had 3 attendings , 1 fellow, and 1 resident grade films. Their results verified the results of Herring. Their conclusion: “the lateral pillar classification had good intra-interobserver reliability in this study. Ferguson noted that when the lateral ossified segment of the femoral head is present, it becomes a weight bearing strut that protects the central avascular segment. The radiographic presence of this visible lateral segment seen on an AP hip radiograph during the fragmentation phase is the cornerstone of the lateral pillar classification described by Herring. Tidbits from Miller: osteonecrosis of the proximal femoral epiphysis, usually boy s 4-8 with delayed skeletal maturation, symptoms include pain (sometimes knee),
back to this question next question
effusion, limp; decreased hip ROM (esp. abduction and internal rotation) and a Trendelenburg gait. Age is key to prognosis. Patients presenting with a bone age >6 have a significantly poorer prognosis. Picture of the lateral pillar classification in Miller page 179. Maintaining the sphereicity of the femoral head is the most important factor in achieving a good result.
5.03
![]() |
In a patient with cervical spondylotic myelppathy who requires surgical decompression, which of the following is a relative contraindication to a posterior approach such as laminaplasty or laminectomy?
-
Multilevel stenosis and spinal cord compression
-
Fixed cervical kyphosis
-
Obesity
-
Inability or unwillingness to comply with postoperative bracing
-
Severe (AP canal diameter of less than 12mm) stenosis
5.03 Answer 2.
likely to allow sufficient posterior translation of the spinal cord to diminish symptoms. This is a key point in choosing between posterior and anterior approaches for surgical treatment of myelopathy, as is the presence of instability.
6.03
-
Decreased infection rate
-
Decreased risk o injury to the axillary nerve
-
Decreased risk of deltoid avulsion
-
Decreased risk of injury to the biceps tendon complex
-
Increased rotator cuff healing rate
-
-
.03 Answer 3.
![]() |
Despite a track record of a high percentage of good or excellent results, the formal open repair was associated with some disadvantages. The open repair required some form of anterior deltoid takedown combined with a lateral deltoid split; if associated with a long, difficult procedure, open repair could also be associated with some traction injury. A formal open approach has two additional disadvantages related to surgical insult to thee deltoid during the rotator cuff repair. The deltoid takedown and repair usually requires a period of protection in order to avoid any inadvertent avulsion. In order to avoid anterior deltoid detachment, Levy et al, described the arthroscopically enhanced mini approach to rotator cuff repair in 1994. They performed a subacromial decompression through the scope which eliminated the need for the anterior deltoid detachment. Next, they performed an open rotator cuff repair through a lateral deltoid split.
back to this question next question
7.03
![]() |
What finding at initial presentation is most predictive of survival in patients with osteogenic sarcoma?
-
Alkaline phosphatase level
-
Pathologic feacture
-
P53 status
-
Size of tumor
-
Stage of the disease
7.03 Answer 5.
8.03
Initial management of a hypotensive adult trauma patient in the ER includes insertion of at least two large bore IV catheters and administration of
-
1 L bolus of hypertonic saline solution
-
2 L bolus of hypertonic saline solution
-
1 L bolus of lactated Ringer’s solution
-
2 L bolus of lactated Ringer’s solution
-
2 units of uncrossmatched blood
-
8.03 Answer 4.
![]() |
Straight quote from Skeletal Trauma. The most common cause of shock in the trauma patient is hypovolemia from hemorrhage. Infusion of a blaanced salt solution such as Ringer’s lactate or normal saline should be started. Hypertonic saline solutions and colloid containing solutions have also been utilized in trauma resuscitation protocols with varying degees of success. The severely hypotensive trauma patient in hypovolemic shock should receive 2 L of a balanced salt solution as rapidly as possibly.
9.03
![]() |
A 30 yo man who sustained a knee dislocation in a football game underwent an arthroscopically assisted anterior and posterior cruciate ligament reconstruction 1 week after injury. Examination 3 months after injury reveals a small
back to this question next question
effusion, decreased mobility in the patella, and passive range of motion from 20-60º despite physical therapy. What is the next most appropriate step in management?
-
Aggressive physical therapy
-
Corticosteroied injection
-
Revision ACL reconstruction
-
Manipulation
-
Arthroscopy
9.03 Answer 5.
![]() |
Another slap in the face from the Evil Empire. The cited reference tells us nothing about this problem. It tells us a lot of other things, but nothing about this problem. Lets break it down. The first one is wrong because you have already tried PT. The second one is wrong because you never really inject steroid into a recently operated extremity. The third is wrong because there is no evidence that your ACL has reruptured. It comes down to manipulation and arthroscopy. Sounds to me like you got some arthrofibrosis going on and the treatment for that is Arthroscopic Debridement. And lastly:
ARTHROSCOPY IS SOOOOO SEXY. This is just my
opinion however. The answer key makes me think – Darth Beaty strikes again.
10.03
![]() |
Active mobilization following flexor tendon repair is best accomplished with the wrist in
-
Flexion and the MCP joints in flexion
-
Flexion and the MCP joints in extension
-
Neutral and the MCP joints in extension
-
Neutral and the MCP joints in flexion
-
Extension and the MCP joints in flexion
10.03 Answer 5.
11.03
-
Anterior
-
Posterior
back to this question next question
-
Medial
-
Lateral
-
Within the tendon
-
11.03 Answer 4.
Complaints of posterior ankle pain are relatively common in ballet dancers. DDx: tendonitis, capsulitis, and mechanical impingement of the posterior elements of the talus between the posterior tibia and clacaneous. An accessory bone called the os
![]() |
trigonum is sometimes responsible. This ossicle can bexome tentrapped between the posterior tibia and calcaneus during
maneuvers involving the extreme plantar flexed ankle positions of pointe and demi-pointe. This is called the os trigonum syndrome. The origin of the os trigonum is as a persistent separation of the secondary center of the lateral tubercle from the remainder of the posterior talus. This separartion may be a result of repeated trauma during development. But this has never been proven.
Some anatomy: The talus normally contains two prominent tubercles. The lateral tubercle is larger and commonly referred to as the posterior process. Attached to theis lateral tubercle is the posterior talofibular ligament of the ankle joint. Therdeltoid ligament is attached to the base of the medial tubercle. The FHL tendon runs between these two processes in its own small groove. This is where the question of the relationship come. Hence answer 4 LATERAL.
12.03
![]() |
Which of the following approaches is most appropriate to correct a projected limb-length discrepancy of greater than 6
![]() |
cm at skeletal maturity?
-
epiphysiodesis (contralateral) alone
-
acute shortening alone
-
shoe lift alone
-
gradual distraction lengthening
-
physeal bar resection
Anwer- 4
12.03
![]() |
Method of treatment of leg length discrepancy in a child is mostly
dependent on the projected discrepancy at skeletal maturity.
Listed below are the most appropriate treatments based on
projected discrepancy at skeletal maturity.
![]() |
![]() |
![]() |
![]() |
![]() |
0-2 cm No treatment
![]() |
2-6 cm Shoe lift, epiphysiodesis, shortening
6-20 cm Lengthening, which may or may not be combined
![]() |
![]() |
with other procedures
>20 cm Prosthetic fitting
Since the leg length discrepancy at skeletal maturity is greater
![]() |
than 6 cm, lengthening is the only appropriate choice. All other answers are appropriate for smaller discrepancies.
13.03
![]() |
A 43 year old woman has a proximal humeral lesion that was found incidentally on the chest radiograph
back to this question next question
shown in figure 1a. She reports no pain and has full shoulder range of motion. An MRI scan is shown in figure 1b. What is the next most appropriate step in management?
-
Observation
-
Biopsy
-
Serum protein electrophoresis
-
CT of the chest, abdomen and pelvis
-
Administration of bisphosphonates
Answer
1
13.03
![]() |
These radiographic studies show a small cartilaginous lesion with intramedullary calcification without cortical involvement or soft tissue extension. This is most consistent with an enchondroma. An enchondroma is a benign cartilage lesion typically located in the metaphysis of the femur, tibia and humerus or in the small tubular bones of the hand. Intramedullary puntate or popcorn calcificatons are seen. Adaptive changes typical of more aggressive lesions are not seen. These lesions are usually aymptomatic and treatment with observation is acceptable
14.03
![]() |
Formation of heterotopic ossification in the hip abductor musculature after antegrade intramedullary femoral nailing is most closely associated with
-
ipsilateral tibia fracture
-
the age of the patient
-
timing of fixation
-
reaming
-
wound irrigation
back to this question next question
4
14.03
![]() |
Heterotopic bone may form in the hip abductor musculature after intramedullary nailing. Local factors such as muscle damage during the approach, reaming debris, and systemic factors may create an environment that promotes heterotopic ossification. Severe heterotopic ossification may limit hip motion, cause pain and limp, and make nail removal more difficult.
![]() |
Heterotopic ossification has been shown to develop more frequently in those who have had a reamed nail than in those with an unreamed nail. It is unclear if this finding is related to less reaming debris or less local tissue damage.
15.03
![]() |
A patient who received low molecular weight heparin after undergoing total hip arthroplasty 1 day ago had normal neurologic function and a hematocrit level of 30.8. On the third postoperative day, she reports severe hip pain, is unable to dorsiflex her foot, and has a hematocrit of 21.6. Radiographs show the implant in good position. These developments are most likely caused by
-
Limb overlengthening
-
hematoma
-
infection
-
deep venous thrombosis
-
heparin-induced thrombis syndrome
2
15.03
Answer
![]() |
This question is based on a case report published in the Journal of Arthroplasty. They reported 2 cases of a subfascial hematoma inducing a sciatic nerve palsy after total hip arthroplasty. The presenting symptoms of this are usually severe buttock and thigh pain. Neuropathy can develop within the first 12 hours. Once a decision is made to explore the wound and evacuate the hematoma, it should be done urgently because the result appears to be time dependent.
16.03
![]() |
Respiratory compromise following anterior cervical spine procedures is most closely associated with
-
multilevel approaches above C4
-
a surgical time of less than 3 hours
-
obesity
-
smoking
-
diabetes mellitus
1
16.03
back to this question next question
![]() |
The quoted study was published in Spine in 2002. It looked retrospectively at 311 anterior spine cases. Of these, 19 had an airway complication and 6 required reintubation. Symptoms developed on average, 36 hours after surgery. Most of these complications were due to pharyngeal edema. Variables that were found to be statistically linked to airway complication were exposing more than three vertebral bodies, a blood loss of > 300cc, exposures involving C2, C3 or C4, and an operative time of > 5 hours.
17.03
![]() |
![]() |
![]() |
![]() |
![]() |
Figures 2a and 2b show the radiographs of a patient who has disproportionately short stature. There
is a defect in the gene coding for
-
fibroblast growth factor receptor-3
-
fibrillin
-
cystathionine synthase
-
adrenocorticotropin
-
parathyroid hormone back to this questionnext question
Anwer- 1
17.03
These radiographs are classic for Acondroplasia. This is the most common form of of skeletal dysplasia seen and is due to a mutation in the gene coding for fibroblast growth factor receptor-3. This is transmitted as an autosomal dominant trait, but 80% of
such as the humerus and femur are foreshortened to the greatest degree. Also common in this disorder is the so called trident
hand (space between the third and forth ray), facial hypoplasia and frontal bossing, elbow flexion contracture, and thoracolumbar kyphosis/spinal stenosis.
Radiographic features include squared iliac wings, metaphyseal flaring, thick short diaphysis, genu varum, and exaggeration of the inverted V distal femoral physis. There is also the classic decreasing interpedicular distance caudally that is seen on spine PA x-ray. All of these features are seen on these radiographs.
Mutation of the fibrillin gene is seen in Marfans syndrome. Mutation of the gene for cystathionine synthase is seen in homocystinuria. Abnormalities in adrenocorticotropin and parathryroid hormone are seen in the associated endocrinopathies.
18.03
-
lateral process fracture
-
anterior process fracture
-
tongue type
-
comminuted posterior facet
back to this question next question
-
joint depression type 18.03
Answer- 3
The Essex-Lopresti reduction technique is a useful method for the treatment of tongue type calcaneal fractures. With this technique, a steinman pin is inserted percutaneously into the posterior facet fragment. This pin is then used to disimpact the fragment and reduce the fracture.
19.03
-
casting
-
bracing
-
CT
-
MRI
-
Stress radiographs
-
-
Anwer- 5
19.03
![]() |
This patient has valgus instability on examination of his knee after an injury with normal x-rays. In such a situation, a severe (grade III) sprain of the MCL needs to be differentiated from an unstable physeal fracture. Stress x-rays will reveal widening of the joint space versus widening of the physis and are indicated in any such presentation.
20.03
![]() |
What is the effect of tobacco smoke on musculoskeletal tissues?
-
increases the relative risk of a hip or wrist fracture
-
increases ligamentous laxity
-
causes osteomalacia
-
causes articular cartilage atrophy
-
causes skeletal muscle hypertrophy
Anwer- 1
20.03
back to this question next question
![]() |
Smoking has been shown to adversely affect bone mineral density, lumbar disk disease, the rate of hip fractures, and bone and wound healing. The increased risk of fracture is only 1.3-1.6% and decreases to normal 10 years after smoking cessation. None of the other answers have been tied to smoking yet.
21.03
![]() |
Figures 3a and 3b show the radiographs of a 29-year-old surgeon who reports persistent pain in the dominant wrist after a fall 3 months ago. Examination reveals that range of motion in the affected wrist is nearly symmetric with the opposite side, although he reports pain at the extremes of dorsiflexion and radial deviation. Management at this time should consist of
1 .scaphoid bone graft and screw fixation
-
scaphoid bone graft, screw fixation, and radial styloidectomy
-
scaphoid excision and capitolunatetriquentral-hamate fusion
-
proximal row corpectomy
-
vascularized bone graft from the distal radius to the scaphoid
21.03 Answer- 2
![]() |
An algorithm is outlined in the cited reference for treatment of scaphoid nonunion. First you check for capitolunate arthritis; If this has progressed to a significant degree than you would need to do a salvage such as an excision or fusion. Next you look for
![]() |
![]() |
![]() |
![]() |
![]() |
![]() |
![]() |
radial styloid degeneration. If this is present than you need to do a simultaneous radial styloidectomy. Next you look for large scaphoid cysts. If these are present than you need to do an interposition wedge graft (a wedge of bone is placed between fracture fragments). Next you need to check for punctate bleeding in the proximal pole. If this is absent than you would move to a vascularized graft. Finally you need to determine stability of the nonunion. Unstable fractures have greater than 1 mm step off, DISI deformity, or an intrascaphoid angle of 55 degrees or more. If unstable than you need to do an interpositional wedge graft. If stable than you can proceed with the standard Russe graft (a corticocancellous inlay graft).
![]() |
This case shows radial styloid degeneration in an unstable nonunion.
Therefore, bone graft with radial styloidectomy is the only correct answer.
22.03
![]() |
A 22-year-old transtibial amputee is an avid runner but find it difficult to run on uneven ground. What prosthetic foot modification would most likely address this problem?
-
Stationary ankle flexible endoskeleton
-
Articulated dynamic response
-
Solid ankle cushioned heel
-
Solid ankle split heel
-
Energy storing
22.03 Answer- 2
back to this question next question
The solid ankle cushioned heel is the most basic foot prosthetic. The stationary ankle flexible endoskeleton has a small piece of wood attached to the base of the keel with semirigid foam and belting materials to attach it. This permits small amounts of inversion/eversion as compared to the SACH foot. The articulating dynamic response foot has an articulation at the ankle with bumpers that allow specific amounts of motion. These feet are superior in accommodating uneven surfaces. This is therefore the best choice for someone requiring better tolerance of uneven ground.
23.03
A 60-year-old patient undergoes a cementless hemiarthroplasty for a displaced femoral neck fracture via a direct lateral (Hardinge) approach to the hip. What gait abnormality is most likely present at the 3-month follow-up examination?
-
Gluteus maximus lurch
-
Gluteus medius lurch
-
Vaulting gait
-
Circumducted gait
-
Stiff-legged gait
-
back to this question next question
![]() |
23.03 Answer: 2
![]() |
There are several main surgical approaches to the hip that you need to know. The approaches, intervals and dangers are outlined in a table in Miller on page 521.
The posteriorapproach is most familiar to us. It splits the gluteus maximus (inf. gluteal n.) en route to the hip and endangers the sciatic
![]() |
nerve. The anterior (Smith-Peterson) approach uses the interval between the femoral n. (sartorius / rectus femoris) and the sup. gluteal
n. (TFL / gluteus medius). The anteriorapproach endangers the lateral fem. cutaneus n. The anterolateral(Watson-Jones) approach uses a muscular interval between the TFL and the glut. Medius (both supplied by the sup. gluteal n.). The medialapproach (Ludloff) uses the interval b/t ant. division of the obturator (Add. Longus/Brevis) and the obturatur/tibial (Gracilis/Add. Magnus)
![]() |
The lateralor Hardingeapproach to the hip has no true nervous interval. It splits gluteus medius (sup. gluteal n.) and the vastus lateralis (femoral n.) in line with their fibers en route to the hip. The most commonly injured nerve on this approach is the superficial gluteal nerve. Injury to this nerve, as you know, will give you a gluteus medius (or abductor) lurch (ans. 2). Ramesh et al. in JBJS (Br.) 1996 found that 23% of patients who underwent the lateral (Hardinge) approach had gluteus medius weakness, of which 75% of these were permanent.
![]() |
![]() |
Gait abnormalities can be confusing. The main types tested are resulting from nerve injury or weakness (this question) and from prosthetic limb mal-fitting (not this question). Here are a few of the other commonly tested gait abnormalities and what they are usually looking for:
Circumducted gait – Prosthetic limb too long, excessive knee frictior
![]() |
Vaulting gait – Prosthetic limb too long, poor suspension
Stiff-
![]() |
legged gait – knee path
![]() |
![]() |
![]() |
Gluteus maximus lurch – weakness of glut maximus; hip hyperextension Antalgic gait – from painful stimulus, results in less relative time on affected limb Steppage gait – equinus deformity (tib. ant. weakness)
24.03
![]() |
Which of the following are considered the most characteristic features of shoulder arthritis following anterior stabilization procedures?
-
Internal rotation contracture and inferior glenoid wear
-
Internal rotation contracture and posterior glenoid wear
-
Internal rotation contracture and superior glenoid wear
-
External rotation contracture and posterior glenoid wear
-
External rotation contracture and anterior glenoid wear
24.03 Answer: 2
![]() |
![]() |
![]() |
![]() |
Anterior shoulder stabilization procedures all aim to reduce the ability of the glenohumeral joint to dislocate or subluxate anteriorly. There are many different techniques for anterior stabilization. Some of the more common ones are listed below:
![]() |
Bankhart – repair of the anterior capsulolabral complex
Inferior Capsular Shift – “pants-over-vest” reduction of the anterior capsule
![]() |
Magnuson-Stack—Subscapularis advancement
![]() |
Putti-Platt—Subscap advancement
![]() |
![]() |
Bristow—transfer of the coracoid onto the anterior glenoid rim Latterjet—more anatomic transfer of coracoid onto the anterior rim of glenoid
The advantages of anterior stabilization procedures is that they can keep the shoulder from recurrently dislocating. The disadvantages depend on the particular procedure chosen, but in general include reduction of external rotation.. This reduction of external rotation comes from subscap shortening and will result in increased intracapsular pressures, which results in both an internal rotation contracture and
posterior glenoid wear. Green and Norris in the JSES 2001in their study of 19 patients who had anterior stabilization procedures, showed the mean internal rotation contracture was 58-degrees and 3/19 had severe posterior glenoid wear. Their study concluded that though it is technically more challenging, total shoulder arthroplasty in these patients provided decreased pain and increased range of motion.
25.03
![]() |
Figures 4a and 4b show the AP and lateral radiographs of a patient who has knee pain after a fall. History is significant for femoral revision of a total hip arthroplasty 2 years ago. Treatment should consist of
![]() |
1-
open reduction and internal fixation with a cable-plate.
![]() |
2- open reduction with an allograft strut and cerclage wire fixation.
![]() |
3- closed reduction and retrograde intramedullary nailing.
![]() |
4- closed reduction with medial and lateral flexible retrograde rods.
![]() |
5- revision to a longer stem femoral component.
![]() |
Answer: 1
25.03
![]() |
The patient has a supracondylar femur below a long prosthetic stem of a total hip. Though full length femur films are not given, you can assume that the femoral stem is well-fixed and no additional fractures are present above this level on the femur.
![]() |
Periprosthetic femoral fractures following THA are increasing due to the younger age and increased number of THA being preformed.
The Vancouver classification system is commonly used and widely accepted and is divided into types A,B,C. It can be found in detail in OKU-7 p. 434. Type A fractures occur proximal to the prosthesis and involve either the greater or lesser trochanter. They are usually
back to this question next question
![]() |
![]() |
![]() |
non-operative, but need fixation if unstable. Type B fractures occur around or just below the stem. They are divided into B1 (well-fixed component)—typically treated with plating +/- allograft; type B2 (loose component)—treated with a longer prosthesis; and type B3 ( severe bone stock loss)— combination of longer prostheses and allograft +/- plating or cement. Type C fractures are well below the stem tip and are treated independently of the prosthesis.
![]() |
This patient has a Type C periprosthetic femur fracture. Treatment of this fracture type should include open reduction and internal fixation with a cable-plate(ans. 1). This is a well-fixed stem and does not require revision (ans. 5). Retrograde IM nailing and flexible nails would not help the problem. ORIF with allograft strut and cerclage wire fixation is a viable alternative, but is not as good as a cable-plate.
26.03
![]() |
A rotational periacetabular osteotomy offers what advantage over a medial displacement pelvic osteotomy for treating adolescent hip dysplasia?
-
Can be used for an incongruent hip
-
Provides intrinsic stability
-
Provides only lateral acetabular coverage
-
Provides hyaline cartilage coverage
-
Internal fixation not required 26.03
Answer: 4
![]() |
The acetabulum remodels until age 8 y/o and it is preferable to treat hip dysplasia (whether it is from DDH or CP) as a child with open growth plates. If hip dysplasia presents as an adolescent, there are two main types of pelvic osteotomies: medial displacement and rotational periacetabular.
![]() |
Medial displacement osteotomies are Salter, Triple and Shelf / Chiari. The acetabulum itself remains unchanged. These osteotomies can be used for an incruent hip. They provide only lateral acetabular coverage and can be preformed with or without
internal fixation. They often involve multiple incisions. A triple osteotomy involves a Salter cut with cuts of both the superior and inferior rami. The Shelf / Chiari osteotomy is a salvage procedure and is only used as a last resort for femoral containment.
![]() |
A rotational periacetabular osteotomy is called a Ganz or Bernese. It allows acetabular reorientation and either medial or lateral displacement. The advantages of this procedure is that it provides hyaline cartilage coverage (ans. 4)over the adolescent hip, can be performed with a single surgical approach and preserves the normal shape of the pelvis—permitting normal vaginal delivery. An additional advantage is preservation of blood supply to the acetabular fragments. No complete cut is made, so it is very stable. The disadvantage is that it is technically demanding and often additional training is needed for the surgeon to become comfortable.
![]() |
Reshaping (incomplete) osteotomies include the Pemberton and Degas require an open triradiate cartilage and can achieve posterior or posterolateral femoral head coverage. This question asked about the adolescent hip.
![]() |
There is a good review in JAAOS Sept / Oct 1999 pp.325-336.
![]() |
For the Atlanta crew, remember: God loves a Pemberton.
27.03
![]() |
Lumbar spine disk herniations are most likely to show the greatest degree of spontaneous resorption if they are 1- contained.
-
recurrent.
-
sequestered.
-
far lateral.
-
in the upper lumbar segments
27.03 Answer: 3
![]() |
Lumbar spine disk herniations refer to a displacement of the inner disk material (nucleus pulposis) creating a focal asymmetry in the outer circumference of the annulus fibrosis. Descriptive terms are used to describe the
back to this question next question
![]() |
degreeand locationof disk fragment displacement. Location is divided as follows:
![]() |
Central -- posterior midline
![]() |
![]() |
Posterolateral -- toward one side, but medial to the pedicle (most common) ( Subarticular )
![]() |
Foraminal -- within the medial / lateral borders of the pedicle
![]() |
![]() |
Far Lateral -- lateral to the intervertebral foramen / lateral to pedicle ( Extraforaminal ) Degree of disk herniation can be broken up into containedand noncontained.
![]() |
![]() |
Contained disk herniations are defined by an annulus which is not completely disrupted. An example of a contained disk herniation is a disk protrusion. There are two types of noncontained disk herniations: extrusions and sequestrations. An extrudeddisk is still in
continuity with the inner disk through the annular defect. A sequestereddisk is no longer in direct continuity with the inner disk space.
![]() |
Patients with disk herniations typically resolve their symptoms when treated nonoperatively. In Saal (Spine 1990), 90% of patients with disc extrusions and radiculopathy had a successful outcome without surgery. MRI has been used to document the resorption of disc herniations in nonsurgical patients. Herniations demonstrating the greatest reduction in size due to spontaneous resorption are extruded or sequestered fragments. The disk material exposed to the epidural space is separated from the nutrient supply of the disc and stimulates an inflammatory response. This is believed to be responsible for the near complete resorption of these disk fragments. Herniation location (far lateral), upper lumbar segment, recurrent or contained have no correlation with resorption.
28.03
![]() |
A claw toe is distinguished from a hammer toe by which of the following features?
-
Dorsiflexion of the metatarsophalangeal joint
-
Dorsiflexion of the proximal interphalangeal joint
-
Dorsiflexion of the distal interphalangeal joint
-
Plantar flexion of the metatarsophalangeal joint
-
Plantar flexion of the proximal interphalangeal joint
![]() |
28.03 Answer: 1
![]() |
Lesser toe deformities are common and can result from acquired deformities (footware), or congenital / neuromuscular causes.
The proper definitions important to understanding lesser toe deformities.
![]() |
A mallet toe involves involves the DIP joint: the distal phalanx is flexed on the middle phalanx.
![]() |
A hammer toe involves the PIP joint: the middle and distal phalanges are flexedon the proximal plalanx.
![]() |
A claw toe involves a hammer toe deformity of the phalanges and dorsiflexion(extension) deformity at the MTP joint. (ans. 1).
back to this question next question
A 50-year old man has had left groin pain for the past 3-months. A radiograph, 29.03 MRI scan, abdominal CT scan, and photomicrograph are shown in Figures 5a through 5d. Which of the following problems is considered a postoperative complication for this histologic type of bone lesion?
![]() |
![]() |
-
Infection
-
Deep venous thrombosis
-
Excessive bleeding
-
Hypercalcemia
-
Malignant hyperthermia
![]() |
![]() |
29.03
Answer: 3
![]() |
![]() |
![]() |
![]() |
![]() |
![]() |
The patient has a metastatic lesion to the left femoral neck. On the plain xray, there is a lytic lesion in the femoral neck which lights up (bright signal on T2 weighted coronal MRI cut) in figures 5a and 5b. The following two images (5c and 5d) make the diagnosis. An abdominal CT scan on an OITE test is concerning enough, and if present, like this one, usually has a large lesion
back to this question next question
growing off of the left kidney (like this patient has). This makes you think Renal Cell CA. The histology confirms the diagnosis with clear cells, a fluffy cytoplasm and a glandular component.
![]() |
Renal cell carcinoma typically has a delay before identification and treatment of the primary tumor. This is due to the large volume of the abdominal cavity which allows for expansion of the tumor. Patients occasionally complain of abdominal or back discomfort. Hematuria is common. Renal cell mets typically involve the cortex of bone and most likely produce large, purely lytic lesions.
![]() |
Renal cell mets are typically dangerously vascular, and preop arteriography is important in the work-up. Excessive bleeding (ans. 3) is a potential complication in this type of tumor.
![]() |
The sources of reference were from the Journal of Vasacular and Intervential Radiology and Acta Orthopaedics Scandinavia—you wouldn’t have read them and don’t need to have to answer this question correctly.
30.03
![]() |
Which of the following arteries provides primary blood supply to the posterior cruciate ligament?
-
Superior medial geniculate
-
Superior lateral geniculate
-
Middle geniculate
-
Inferior medial geniculate
-
Inferior lateral geniculate
![]() |
30.03 Answer: 3
The blood supply to structures around the knee has been well studied. The popliteal artery gives rise to five branches that supply blood to the knee joint:
Superior geniculate artery medial branch (1) lateral branch (2)
Inferior geniculate artery medial branch (3) lateral branch (4)
Middle geniculate artery (5)
The middle geniculate artery penetrates the posterior capsule of the knee and provides the major blood supply to the ACL, PCL, synovial membrane and posterior capsule
The distal portion of the PCL also receives a portion of its blood supply from capsular vessels originating from the inferior geniculateand popliteal arteries.
31.03
![]() |
Which of the following risk factors for venous thromboembolism is associated with a primary hypercoagulable state?
-
Inflammatory bowel disease
back to this question next question
-
Obesity
-
Varicose veins
-
Malignancy
-
Antithrombin III deficiency
31.03
![]() |
Answer: 5
![]() |
Thromboembolic disease is common in patients who have had major lower extremity trauma , total hip or knee arthroplasty, and spinal cord injury. The risk of venous thromboembolism is compounded in the presence of hypercoagulable states.
Hypercoagulable states are classified as either primary or secondary.
Primary(inherited) disorders involve deficiencies in one of the normal anticoagulant mechanisms. This list includes antithrombin III deficiency, protein C or S deficiency and activated protein C resistance.
![]() |
Secondaryhypercoagulable states are clinical conditions that predispose to thromboembolic disease. Examples of this
![]() |
include obesity, malignancy, varicose veins, IBD and others.
![]() |
![]() |
A full list is available in OKU-7 on p. 56. It’s a good bedtime read. 32.03
![]() |
A carpenter sustains a traumatic amputation through the distal interphalangeal joint of his dominant ring finger from a table saw. The wound is allowed to heal by secondary intention. One year after injury, the patient reports extension of the finger when he tightly grips a hammer. Treatment should consist of
-
step-cut lengthening of the central slip
-
spiral oblique retinacular ligament reconstruction 3- fourth and fifth dorsal interossei muscle slides.
-
release of the lumbrical tendon.
-
tenodesis of the lacerated profundus tendon to the terminal tendon.
32.03 Answer: 4
![]() |
This question describes the classic post-traumatic “lumbrical plus” finger. The listed reference is out of the Buliten of the Hospital for Joint Disease. You won’t have read it and neither did I.
![]() |
Here’s what you need to know about a lumbrical plus finger. It can result from a laceration of the FDP (isolated or associated with a DIP amputation), a loose FDP or adhesions of the lumbrical to the FDP. The distal aspect of the FDP inserts on both the distal phalanx and the lumbrical tendon and aids in both DIP flexion as well as the extension cascade of the PIP joint. When the distal insertion of the FDP is cut from the distal phalanx, it’s insertion on the lumbrical typically remains intact. Therefore, flexion of the DIP is blocked, but the FDP’s action to assist the extension cascade remains. When a fist is made, pulling of the FDP will cause a paradoxical extension of the IP joint instead of flexion of the DIP joint. This is the “lumbrical plus” finger.
![]() |
Treatment of this involves cutting or releasing the lumbrical tendon. (ans. 4) This action prevents the paradoxical extension and allow the patient to make a more complete fist.
33.03
![]() |
Which of the following findings is the best indication for the use of temporary external fixation of a femoral shaft fracture?
-
Type IIIA open fracture
back to this question next question
-
Hemodynamic instability
-
Segmental fracture
-
Distal one third fracture
-
Ipsilateral tibial shaft fracture 33.03
![]() |
Answer: 2
![]() |
These days, femoral shaft fractures at Tulane / Charity are commonly encountered by orthopaedic residents on the night-float team.
Despite the presence of a well-rested 4th year surgeon, definitive orthopaedic fixation is not always the correct answer for each trauma patient.
![]() |
Tulane defines “Orthopaedic Tunnel Vision” as a condition commonly associated with a young MD at the Bulldog without a proper wing-man, trying to make advances on the wrong patron due to his relatively easy 80-hour work week schedule and a few too many refined hops.
![]() |
Skeletal Trauma (p. 1967) describes “Orthopaedic Tunnel Vision” as looking at the orthopaedic injury without considering the patient’s injury in general. Femoral shaft fractures are typically high energy injuries which often do not occur in isolation. In these fractures, it is particularly important to not have tunnel vision.
![]() |
Indications for temporary bridging external fixation includes hemodynamic instability
(ans. 2), acidosis, hypothermjia, hypoxemia, coagulopathy, sepsis or severely contaminated soft tissues that cannot be adequately debrided. Definitive fixation is performed after the general surgical and medical issues have resolved.
![]() |
The other answer choices, including the type IIIA open fracture are not contraindications to definitive fixation in themselves (typically IM nailing—antegrade or retrograde).
34.03
A 12 month-old girl has left congenital fibular hemimelia type 1. The ipsilateral foot and the contralateral lower extremity are normal. A scanogram shows a limb-length discrepancy of 2 cm, with all of the shortening in the left tibia. Assuming no treatment is rendered, the limb-length discrepancy at skeletal maturity will most likely
-
remain 2 cm
-
gradually increase to 5 cm
-
increase at a constant rate of 2 cm with each year of growth
-
increase markedly b/c of the complete absence of growth in the left tibia
-
increase, with the left lower extremity remaining in a similar proportion to the right lower extremity
back answer
back to this question next question
Question 34.03 Answer = 5
Fibular Hemimelia
The precise cause is unknown in most cases. 15% with congenital absence of the fibula have associated deficiencies of the femur. One study showed that allof their fibular-deficient patients with 5-rayed feet and 50% of those with fewer rays had shortened femurs.
60 % of fibular growth from proximal physis 40% from distal physis.[1]
”It is the most common form of lower limb deficiency. Fibular shortening causes lateral ankle instability. Tibial deformities include shortening, anterior bowing, and valgus deformity. Foot deformities include absence of lateral portions of the foot, talocalcaneal fusions, equinus. Shortening is progressive but remains proportionally shortened to the opposite normal side.”[1] Coventry & Johnston classificationTypeCharacteristicsIPartial absence of fibulaIIComplete absence of fibulaIIIBilateral absence of fibula, orOther skeletal abnormalities associated with unilateral fibular absenceAcherman & Kalamchi ClassificationIPartial absence of fibula(any part of fibula is present)IAProximal fibula epiphysis is distal to tibial growth plateDistal fibular physis is proximal to dome of talusIBFibula is shorter by 30-50%Distally fibular does not provide and ankle supportIIComplete absence of fibula [1] Practice of Pediatric Orthopaedics, Staheli, pg 87
back to this question next question Figures 6a and 6b show the radiographs of a 43 year old man who
35.03 injured his right elbow in a fall. Which of the following surgical approaches will provide the best exposure for ORIF of the fracture?
-
lateral kocher
-
transolecranon osteotomy
-
triceps splitting
-
triceps reflection 5) medial extensile
-
![]() |
![]() |
back answer
Question 35.03
Answer = 2
The pictures show a comminuted, intraarticular both column distal humerus fracture. The goal is to expose and stabilize the entire distal humerus. Isolated medial and lateral approaches will not allow enough exposure of the contralateral column. Triceps splitting and reflecting won’t allow enough exposure either b/c you’ll need to see the entire distal humerus—you would end up completely avulsing the triceps.
”My preference has been to utilize the transolecranon approach, particularly for comminuted fractures.” Before creating osteotomy, ulnar nerve should be exposed. Once the fracture is identified, goal = reduce articular fragments onto the bony columns of the distal humerus—initially holding the reduction with provisional k-wires. [1]
[1] From: Master’s Elbow 2002, Chapter 4; Jesse Jupiter
back to this question next question
36.03
An 18 year old female Olympic basketball player is seeking advice on how to prevent future ACL injury. What recommendations can be made based on available scientific data?
-
Use of custom-fitted derotational bracing
-
Use of “off-the-shelf” derotational bracing
-
Arthroscopic prophylactic notchplasty
-
Plyometrics and neuromuscular training for 6 weeks 5) Oral contraceptive use
back answer Question 36.03
Answer = 4
The reference article is a landmark in Female-ACL Injuryliterature. Hewett et al, “Plyometric Training in Female Athletes.” Am. J. Sports Med 1996, 24(6) pg 765-773. The conclusion: “This training may have significant effect on knee stabilization and prevention of serious knee injury among female athletes.”
In researching this answer I searched Ovid, looked in sports texts and looked online for “derotation brace.” Every article mentioned “derotation brace” with “the ACL-deficient knee.” I could not find any mention of prophylactic ACL injury prevention with derotation brace.
Prophylactic surgery and oral contraceptive prescription would never be correct answers. As with “derotation brace”, they are buzzwordsin the femaleACL injury literature and you could pick them as an answer if you didn’t have enough knowledge of the subject. It’s hypothesized that estrogen levels may have a role in when females injure there ACLs.
Finally, notchplasty relates to resecting part of the lateral femoral condyle during ACL reconstruction for both visualization of graft placement and to prevent impingement on the graft.
back to this question next question Figure 7 shows the clinical photograph of the knee of a chronic
37.03 cigarette smoker 2 weeks after revision TKA. Treatment should now consist of
-
I&D with poly insert exchange
-
removal of components and insertion of antibioticimpregnanted cement spacer
-
debridement and a medial gastrocnemius muscle flap
-
debridement and free flap 5) knee arthrodesis
-
back answer
Question 37.03
Answer = 3
Because this is an acute problem (2 weeks) and there is no obvious hardware or bone exposed, the first objective should be to save the components (avoid removal). A simply I&D with poly exchange does not allow for soft tissue coverage. A knee arthrodesis would be a salvage procedure after failed attempt(s) to save the arthroplasty. This
narrows us down to either choice 3 or 4. Even if you don’t know much about flaps, you know that a rotational flap has a better chance of success then a free flap.
“The seemingly radical approach of prompt wound excision and reconstruction with a muscle flap is in fact the conservative approach.” Flaps include medial and/or lateral gastroc, as well as free latisimus, serratus anterior, and rectus abdominus. The medial head is the larger of the two and can frequently be used to cover defects that extend laterally. Lateral head is used for lateral defects. [1]
Medial Head: [2]- for defects over knee, medial head of gastrocnemius covers knee joint (& patella, if origin from medial femoral condyle is divided);- medial head of the gastrocnemius is ideally suited to cover exposed bone over the proximal 1/3 of the tibia;- medial or lateral heads of the gastrocnemius muscle can be expended with little or no deficit when walking or in normal running;- amoung disadvantages of the muscle flap is that a portion of proximal achilles tendon must be divided to free medial or lateral head of gastroc muscleBlood Supply:- each head of gastrocnemius is supplied by the sural artery, which is branch of the popliteal artery;- sural artery passes directly into proximal portion of both heads, (about 3 cm above the joint line) where it arborizes and then travels longitudinally along length of muscle;- there are also direct
fasciocutaneous arteries arising from popliteal artery as well as septocutaneous vessels arising from trifurcation of the popback to liteal arterythis questionof the leg next question
[1] Master’s Knee Arthroplasty 2003; chapter 24 Management of Skin Necrosis, LaRossa & Low [2] http://www.ortho-u.net/05/2420.htm ( wheeless ))
A 20 year old woman has progressive low back pain. An AP pelvic radiograph, CT scan, posterior pelvic bone scan, and
38.03 biopsy specimen are shown in Figures 8a-d.
Immunohistochemistry reveals CD99 reactivity but no staining for leukocyte common antigen. What is the most likely diagnosis?
![]() |
![]() |
![]() |
![]() |
![]() |
-
Ewing’s
-
Lymphoma
-
Multiple Myeloma
-
Langerhans cell histiocytosks
-
Osteomyelitis
back
answer
![]() |
Question 38.03
Answer = 1
The CT shows a lytic lesion with cortical breakthrough in the posterior ileum on the right. The bone scan is slightly darker in that region. Histology shows a homogenous field of small, blue cells. This eliminates osteomyelitis.
Histo: (+) for s -100
hematopoetic cells
(+) for CD-99 (CD-99 antibody attaches to the MIC-2 Antigen which is present in Ewing’s & PNET tumors)
(-) for leukocyte common antigen (= CD45; CD45 is expressed on all
exceptmature RBCs and platelets)
Ewing’s Lymphoma Multiple Myeloma Langerhan’
![]() |
![]() |
![]() |
cell is a dendritic skin onionskinning
soft tissue componentl arge soft tissue masses; plasma cell = eccentric nucleus, “clock face
a long lesion; thick cortex, ” chromatin, peronuclear halo;lytic no lysis & formation; diffuse sclerosis
Chemo XRT & Chemo chemoORIF XRTbisphosphonates back to this question next question XRT
Resection 39.03
Anatomic studies of the proximal tibia have shown that the capsular reflection of the knee joint extends the farthest distal to subchondral bone at what location?
-
anterior to the fibula
-
posterior to the fibula
-
directly medial to the tibia
-
directly posterior to the tibia 5) anteromedial to the tibia
back answer Question 39.03
Answer = 2
This online anatomy picture of the posterior knee clearly shows the capsule extending distally posterior to the fibula. http://www.bartleby.com/107/illus352.html ( Grant’s anatomy online )
![]() |
![]() |
back to this question next question
40.03
Which of the following is considered a normal age-related change in the intervertebral disk in the elderly?
-
increased proteoglycan concentration
-
increased biosynthetic function
-
increased viable cell concentration in the central region
-
decreased water content 5) decreased stiffness
back answer Question 40.03
Answer = 4
Remember 2 things Dr. Brinker has said about collagen:
“collagen dehydrates when it gets older” “water follows the proteoglycans”
We all know that as we get older our collagen is less pliable and, specifically, our disks shrink, bulge, and are stiffer. They’re smaller and stiffer because there’s less water. There’s less water because there’s less proteoglycans. There’s less proteoglycans because there’s less biosynthetic function. There’s less biosynthetic function because there are fewer viable cells. Why are there fewer viable cells? I don’t know. Neither do the authors of the Academy Basic Science Textbook. Presumably one day we’ll find the gene(s) / protein(s) / cue(s) that are responsible for the overall slow, programmed decline of our bodies that we call “aging.” [1]
![]() |
![]() |
![]() |
back to this question next question
![]() |
![]() |
![]() |
[1] Orthopaedic Basic Science 2nd ed. Pg 558
41.03
A tall, thin 13 year old girl with idiopathic scoliosis undergoes instrumented posterior spinal fusion from T4-L3. Posteroperative recovery is uneventful, and a clear liquid diet is begun on the third day. The patient responds with vomiting and is unable to tolerate any oral intake; however, bowel sounds are normal. NG suction reveals large amounts of persistent bilious fluid drainage. The patient is afebrile, with a normal WBC count and normal serum amylase and lipase. What is the next most appropriate step in management?
-
request psychological consultation and increase the patient’s sedative medications
-
aspirate the spinal wound and institute broad-spectrum antibiotic therapy
-
obtain a myelogram and revise the implants to lessen the correction 4) obtain an ultrasound of the kidneys and perform an exploratory laparotomy 5) obtain an upper GI study and institute parenteral feeding. back answer
Question 41.03
Answer = 5
Two of the referenced articles are “Superior mesenteric artery syndrome after segmental instrumentation: A biomechanical analysis” and “Pancreatitis after surgery in adolescent idiopathic scoliosis: incidence and risk factors.” Both are rare.
There is no mention of neurologic deficit, fever, urinary complaints, psychiatric disorder, or wound complications. So 1, 2, 3, and 4 can be eliminated just from what the question says (and doesn’t say). Amylase and lipase are makers for pancreatitis—this would likely manifest with some degree of abdominal pain in addition to vomiting. You may remember from your internship that we treat post-op ileus with NG tubes and bowel rest. We also start parenteral feeding to provide nutrition.
In this case, there’s also the rare possibility that lengthening the spine may have “stretched” the guts and caused SMA syndrome. This is defined as: “Duodenal obstruction caused by compression of the third part of the duodenum by the root of the intestinal mesentery which contains the superior mesenteric artery, vein, and nerve.”[1] SMA syndrome is also called Cast Syndrome. Here’s a quick abstract:
Munns-SW; Morrissy-RT; Golladay-ES; McKenzie-CN; J-Bone-Joint-Surg-Am. 1984 Oct; 66(8): 1175-7
The cast syndrome, recently called the superior mesenteric-artery syndrome, is a well recognized complication that can occur after a body cast has been applied. We are reporting the cases of three patients who had this syndrome following surgical correction of spinal deformity. The complication failed to resolve with the usual non-operative measures but did resolve with treatment by total parenteral nutrition.
[1] http://fred.hmc.psu.edu/ds/retrieve/fred/meshdescriptor/D013478)
back to this question next question
42.03
Management for Charcot neuropathy in the foot may include the use of
-
aspirin
-
bisphosphonates
-
cephalosporins
-
estrogen
-
vitamin D
-
back answer
Question 42.03
Answer = 2
I have to admit, I guessed on this one. I must have misplaced my November 1994 issue of Diabetic Medicine. I think Burak stole the copy of Diabetologia 2001out of my mailbox.
I figured that vitamin D, estrogen, and cephalosporins were distractors b/c Charcot isn’t related to Ricket’s, estrogen deficiency, and it’s not an infection. I know that diabetics and vasculopaths get Charcot so they maybe on aspirin BUT, this is an orthopaedics test—let’s give something to help limit bone resorption.
Here’s a clinical discussion of Charcot:
Stage |
Characteristics |
Treatment* |
0 Clinical stage |
Erythema, edema, increased temperature to foot |
Limited weight bearing (possibly TCC or PPWB), close observation |
1 Fragmentation stage |
Periarticular fractures, joint dislocation, instability, deformed foot |
TCC, limited weight bearing |
2 Coalescence stage next que |
Reabsorption of stionbone debris |
TCC followed by CROW |
Two theories (neurotraumatic and neurovascular) explain the pathogenesis of Charcot foot. The neurotraumatic theory attributes bony destruction to the loss of pain sensation and proprioception combined with repetitive and mechanical trauma to the foot. The neurovascular theory suggests that joint destruction is secondary to an autonomically stimulated vascular reflex that causes hyperemia and periarticular osteopenia with contributory trauma. Intrinsic muscle imbalance with increased heel and plantar forces can produce eccentric loading of the foot, propagating microfractures, ligament laxity and progression to bony destruction. [1] [1] http://www.aafp.org/afp/20011101/1591.html
back to this question
3 Reparative stage |
Stable foot |
Possible surgical intervention for removal of bony prominences associated with ulceration |
43.03
A 23 year old sustains an isolated right knee dislocation in an MVA. A closed reduction is performed and confirmed with radiographs. What is the next appropriate study?
![]() |
![]() |
![]() |
![]() |
![]() |
-
CT of knee
-
MRI of knee
-
radiographs of femur and tibia
-
angiography of the leg
-
noninvasive assessment of lower extremity perfusion
backanswer Question 43.03
Answer = 5
This is simple A, B, Cs and doctoring. History and Physical exam come first. Noninvasive assessement includes visual inspection, clinical examination (feel pulses, check capillary refill, feel temperature of skin), and ABIs. X-ray, CT, and MRI would be good secondary studies to identify bony or soft tissue injuries—after initial reduction and splinting/stabilizatoin. Angiography would be indicated if there were asymmetric pulses or an ABI < 0.9, or if there was any other indication that a vessel injury may have occurred.
OKU Truama 2 says “the use of ABI with the blood pressure cuff and Doppler evaluation of the distal circulation has been proposed as effective in determining any occult vascular injury. A ratio > 0.9indicates a normal study.”[1] The referenced article confirms this. “Confirmation of the safety and accuracy of physical exam in the evaluation of knee dislocation for injury of the popliteal artery.” J. Trauma2002; 52: 247-252
back to this question next question
[1] OKU Trauma 2 pg 151-153
44.03
![]() |
![]() |
Figure 9 shows the radiograph of a patient who has anterior knee pain and range of motion from 0-of the tibial tubercle. Treatment should consist of
-
lateral retinacular release
-
tibial tubercle transfer
-
revision of femoral component
-
revision of tibial component
-
proximal quadriceps realignment
back answer Question 44.03
Answer = 3
r the medial 1/3
The picture shows a laterally subluxed and a relatively-externally rotated patella on a femoral component. The question tells us that the femoral component is internally rotated and the tibial component is perfectly placed. If we know that the femoral component should be externally
rotated about 3 degrees, we know that in this patient, that’s where the problem is. So, we don’t have to do anything to the quads, tubercle or tibial component. We need to revise the femur. Besides, a lateral retinacular release may damage the lateral geniculates; since this patient most likely had a medial arthrotomy, this may devascularize the patella and lead to patellar AVN. A lateral retinacular release will not address the malpositioned components )
back to this question next question
45.03
![]() |
Intramedullary screw fixation of a Jones fracture has a statistically higher failure rate in
-
elite athletes
-
female patients
-
patients who did not undergo bone grafting
-
patients younger than age 40 years
-
fractures that have been fixed with screws larger than 4.5mm in diameter.
45.03 Correct answer = 1
![]() |
Jones fractures can be challenging to treat and can result in significant disability for some patients. Non-operative treatment has a particularly high nonunion rate in athletes. A recent study reported six refractrures after cannulated screw fixation of Jones fractures in athletes (4.0 – 5.0). Their recommendation was to use large screws only. DeLee recommends nothing other than a 4.5mm.
Early return to activity has also been shown to result in nonunion and refracture. No correlation was found between failure and bone grafting
![]() |
![]() |
![]() |
46.03
Figure 10a shows the radiograph of a
![]() |
![]() |
![]() |
![]() |
![]() |
![]() |
![]() |
![]() |
16-year-old boy who is being evaluated for an upper respiratory infection. Further evaluation includes the CT scan shown in figure 10b. A bone scan does not reveal any other sites of skeletal involvement. A biopsy specimen is shown in Figure 10c. What is the next most appropriate step in management?
![]() |
-
Systemic antibiotics 2-Chemotherapy
3-Radiation therapy 4-Wide resection
5-Curettage
back to this question next question
46.03 Correct answer – 4
47.03
Figure 11 shows the clinical photograph of a patient who underwent knee arthroplasty 12 weeks ago and now has wound complications. Management consisting of serial debridements, bacteria-specific antibiotics and local dressing changes has failed to resolve the problem. Examination reveals that the patient is afebrile and has a normal WBC count. In addition to continues use of antibiotics, management should include
1- retention of the total knee components and split thickness skin grafts 2- polyethylene exchange and split-thickness skin grafts. 3-removal of the total knee components, a local rotation flap, and split-thickness skin grafts.
4- primary knee fusion and split-thickness skin grafts 5- amputation
47.03 Correct answer -3
Normally, an antibiotic spacer is placed as well.
48.03
Collagen orientation in the deep layer of normal articular cartilage is aligned
-
randomly
-
parallel to the surface, not crossing the tidemark.
-
parallel to the surface, crossing the tidemark.
-
perpendicular to the surface, not crossing the tidemark. 5- perpendicular to the surface, crossing the tidemark.
back to this question next question
48.03 Correct answer -5
49.03
A patient with neutral ulnar variance sustains an extraarticular fracture of the distal radius that heals with normal palmar tilt but with a loss of radial height. Resultant ulnar variance is measured at +3mm. What percent of load transmission across the wrist will now be borne by the ulna?
1-10 %
2-25 %
3-40 %
4-75 %
5-90 %
49.03 Correct answer – 3
traverse it. In the distal aspect of the forearm, the radius dwarfs the ulna and accounts for 80 per cent of the force transmitted from the wrist to the forearm. The relative amount of force transmitted to the forearm from the wrist is closely associated with the relative lengths of the radius and ulna. Normally, the two bones are of nearly equal length. Ulnar variance results if they are not. Positive ulnar variance results when the ulna is longer than the radius distally, and negative ulnar variance occurs in the opposite situation. Shortening of the radius by even a small amount dramatically increased the amount of force transmitted to the ulnar aspect of the wrist. In this case the force increases from 20% to 40%. The ulna is a relatively straight bone that progressively enlarges proximally in the forearm. At the elbow, the ulna provides the site of attachment for both the medial and the lateral collateral ligament. At the level of the wrist, the ulna normally accepts only a small proportion of the load transmitted to the forearm by the carpus. In the proximal aspect of the forearm, the ulna accepts proportionately more load because of the transference of load from the radius through the interosseous membrane and the addition of local muscle forces. At the elbow, the radius transmits 60 per cent of the transarticular load, depending on the position of the forearm when the measurement is made. The radius has a gentle curve as it traverses the forearm.
50.03
-
an awl for entry portal creation
-
a tourniquet
-
a large intramedullary nail
-
a static locking construct
-
continuous traction
back to this question next question
50.03 Correct answer – 5
The tibial isthmus is short and rotational control is not possible just by reaming for a larger nail. Reaming seems acceptable up to a type IIIA open fracture. One should not ream immediately after removing an ex-fix, per the source. Restoration of length decreases muscular compartment volume. Therefore, traction with reaming will drastically increase compartment pressures.
Fortunately, reamed nailing can be delayed until the soft tissues have resolved, if this is an issue.
51.03
Which of the following are known predictive factors for curve progression in idiopathic scoliosis?
-
peak height velocity, female gender, open triradiate cartilage, premenarchal status
-
peak height velocity, male gender, open triradiate cartilage, preadrenarchal status
-
peak height velocity, female gender, closed triradiate cartilage, premenarchal status
-
peak height velocity, female gender, open triradiate cartilage, postmenarchal status
-
family history, female gender, open triradiate cartilage, premenarchal status
51.03 Correct answer -1
cessation of growth can be reliably predicted, so that patients can be told well in advance how long they will need to wear an orthosis.
52.03
What muscle is at greatest risk for denervation when the posterior capsule is released during a rotator cuff repair?
-
Deltoid
-
Subscapularis
-
Biceps
-
Supraspinatus
-
Infraspinatus
52.03 Correct answer – 5
The purpose of this study was to determine the anatomy of the neurovascular supply of the supraspinatus and infraspinatus muscles and to assess the risk for injury to the neurovascular pedicle when the rotator cuff is mobilized by a standard anterosuperior approach.
Under no circumstances should the supraspinatus or infraspinatus muscle be advanced more than three centimeters laterally.
back to this question next question
Dissection from the lateral approach through an anterosuperior incision should not proceed more than a few centimeters into either fossa, and there may be instances
when dissection as little as one centimeter into either fossa may injure the neurovascular structures.
When the rotator cuff is severely retracted, sufficient mobilization without neurovascular injury may be impossible, so that other alternatives may need to be considered. In all shoulders, the limiting factor for lateral mobilization was
tension on the motor branches. In three (10 per cent) of the thirty-one shoulders, the second motor branch to the supraspinatus muscle was avulsed by overzealous, unchecked lateral transposition of the muscle. In all
dissections, the suprascapular nerve was closely applied to the bone of the floor of the supraspinatus fossa, and it was tethered at thesuprascapular notch deep to the transverse scapular ligament. Lateral advancement of the supraspinatus caused the motor branches to this muscle to pivot approximately 180 degrees around the pedicle of the suprascapular nerve, which remained relatively fixed on the floor of the fossa The suprascapular nerve ran an oblique course toward the rim of the glenoid as it traversed the supraspinatus fossa and entered the infraspinatus fossa around the base of the scapula.
53.03
Within the functional spinal unit, the nucleus pulposus functions
-
to resist compressive loads.
-
to resist tension loads.
-
to resist shear loads
-
independent of the annulus fibrosis.
-
independent of the facet joints
53.03 Correct answer = 1
Disks, like other connective tissues, consist of a sparse population of cells and an abundandt extracellular matrix formed by an elaborate framework of macromolecules filled with water. Blook supply is found solely in the periphery of the normal disk, therefore nutrition is diffusional. The disk is formed by an outer annulus fibrosus (ring of organized dense type I collagen lamellae – some of which insert into the vertebrae), a larger fibrocartilaginous inner annulus fibrosus (less dense type II matrix, and the central nucleus pulposus. At birth the nucleus has interco0nnected notochordal cells, which eventually disappear leaving chondrocytes in place. Collagens account for as much as 70% of the dry weight of the outer annulus, but lexx than 20% of the dry weight of the nucleus in young individuals. Proteoglycans account for only a few percent of the dry eweight of the outer annulus, but as much as 50% of the dry weight of the nucleus (young). Type I collagen is absent in the nucleus and type II predominates. PRoteoglycan concentration increases with type II concentration. Therefore, the nucleus has a lot of proteoglycans. This gives the nucleus the compressive strength needed to resist axial load. The outer annulus fibrosus resists tensile loads and the nucleus provides a hydrostatic barrier that limits deformation.
54.03
A 52-year-old man sustains a displaced bimalleolar (Weber B) ankle fracture in a fall. History reveals that he has had type I diabetes mellitus for the past 30 years. Examination reveals a closed injury with mild swelling, palpable pulses and decreased sensation to the midcalf level. Management should consist of
1 .closed treatment with a short leg cast
-
percutaneous pin-in-plaster technique
-
external fixation spanning the ankle
back to this question next question
-
amputation
-
open reduction and internal fixation of the medial and lateral malleoli
54.03 correct answer = 5
Although the weber classification doesn’t really include the medial side, the point here is that the fibula fracture is at the level of the syndesmosis ( not above). One can assume stabilization of the medial side will stabilize the syndesmosis. The primary stabilizer of the ankle under physiologic loading is the deltoid ligament. The deltoid is intact here because the fracture exited at the medial malleolus. In the absence of medial tenderness, isolated lateral malleolar fractures may be treated nonoperatively. Bimalleolar ankle fractures are treated operatively typically. However, with significant co-morbidity, nonoperative treatment is acceptable. Ankle fractures in diabetics are associated with higher rates of complications than in nondiabetics. Surgical infection and wound dehiscence rates are higher. However, non-operatively treated fractures are associated with skin breakdown and infection. In addition, there is always the fear of a Charcot joint. Unstable fractures (this one) in which a reduction is difficult to achieve and maintain, may be treated surgically, because this will afford greater control over the fracture.
55.03
Treatment of complex regional pain syndrome of the knee inclues 1 . knee manipulation
-
passive range-of-motion exercises
-
beta-blocking agents
-
alpha-blocking agents
-
parasympathetic blockade.
55.03 Correct answer = 4
Schutzer and Gossling1 define the syndrome of reflex sympathetic dystrophy (RSD) as an exaggerated response to injury of an extremity, manifested by four more or less constant characteristics: (1) intense or unduly prolonged pain, (2) vasomotor
disturbances, (3) delayed functional recovery, and (4) various associated trophic changes.
Reflex sympathetic dystrophy is manifested by abnormal vasomotor, thermoregulatory, neurotrophic, sympathetic, and parasympathetic activity in the extremity. According to the literature, the initial treatment of RSD should include gentle exercise, the avoidance of aggressive manipulation, massage, contrast baths, biofeedback, limb elevation to control edema, NSAIDs, antidepressive medications, and psychological evaluation.
Several drugs have shown promise in relieving symptoms of RSD. Of the many actions of the sympathetic nervous system, the a-adrenergic action is the most important in
its effect as a vasoconstrictor in the skin and subcutaneous tissues. Phenoxybenzamine is the most effective a-blocking agent and has the fewest undesirable side effects. Patients treated with phenoxybenzamine must be followed up closely for postural hypotension
back to this question next question
56.03 Based on the radiographic findings shown in Figures 12a and 12b, the patient is most likely to have defects in what other organ systems?
-
Visual
-
Endocrine
-
Renal
-
Hematologic
-
Digestive
back
answer
Question 56.03 Answer = 3
The radiographs in figures 12a and 12b show fusions of the cervical vertebrae. This is consistent with a diagnosis of Klippel-Feil syndrome. Klippel-Feil syndrome is used to denote any congenital cervical spinal fusion. This can range from a Type I lesion, which consists of massive fusion of the cervical vertebrae to Type II lesions which consist of only one or two fused vertebral segments. Type III lesions are either type I or type II lesions with distant anomalies in the thoracic or lumbar spine.
The incidence and etiology of Klippel-Feil syndrome is unknown. The clinical triad of short neck, decreased range of motion, and a low posterior hairline is seen in 40-50% of patients. Complications include anomalies of the genitourinary, cardiovascular or gastrointestinal systems. The most common associated musculoskeletal anomaly is scoliosis (usu congenital) occurring in 60%. Sprengel deformity is reported in 20-30% of patients.
Hearing loss is also commonly associated and is due to abnormalities of the inner ear.
But, getting to this question, renal abnormalities are present in 25-35% of patients with KlippelFeil and is most commonly unilateral renal agenesis. Ultrasound is an effective screening tool. The references do not note any visual, endocrine, or hematologic abnormalities, but does report that the patients can rarely have cardiovascular and gastrointestinal anomalies.
back to this question next question
57.03 A multiply injured patient sustains a closed midshaft femoral fracture in a motor vehicle accident. An external fixator is applied for stabilization the day of injury because of hemodynamic instability. Four days after injury, the patient is stable and the pin
sites are clean. Treatment should now consist of 1- continued external fixation.
-
-
Removal of the external fixator and plate fixation
-
Removal of the external fixator and intramedullary nailing.
-
Removal of the external fixator and traction.
-
Removal of the external fixator and curettage of the pin tracts, followed by intramedullary nailing 2 weeks later. back answer
Question 57.03
Answer = 3
The study cited was from Shock Trauma where they looked at a two-stage stabilization of femur fractures in patients who were too sick to undergo immediate one-stage femoral nailing. They treated 54 multiply injured patients with a total of 59 femur fractures with immediate external fixation (within the first 24 hours) and conversion to intramedullary nails at an average of 7 days ( 1 -49 days). They noted that for 55 of the 59 the conversion was accomplished in a one-stage procedure. In the rest, draining pin sites caused the authors to remove the ext fix, curettage the pin tracts, place the patient in traction and then convert the treatment to im nail after the pin sites healed at an average of 7 days. Their infection rate was 1.7 percent and they felt that this was a safe treatment method.
In the above question, they tell you the pin sites are clean and so with the article in mind, you can go ahead and remove the ext fix and perform the IM nailing at the same setting.
back to this question next question
58.03 A 13-year-old wrestler has had swelling and redness over the anterior knee after sustaining an abrasion 2 weeks ago. Examination reveals no knee effusion and active motion form 0 to 100 degrees.
Radiographs show some soft-tissue swelling anterior to the patella. Management should consist of
-
nonsteroidal anti-inflammatory drugs.
-
Physical therapy.
-
MRI.
-
Corticosteroid injection 5- Aspiration.
back answer Question 58.03 Answer = 5
The cited reference from the University of Iowa looked specifically at prepatellar bursitis in wrestlers. This is a frequent injury to wrestlers and may either be due to an acute traumatic event or due to repeated trauma overtime. They noted 6 cases of septic bursitis, and noted that initially there was no clinical evidence of infection in 4 of these cases, but infection was confirmed by culture results. The infecting organism was most often Staph aureus, but in one it was Strep bovis. Two were cured with repeated aspiration and antibiotics alone. In the others, surgery which consisted of a bursotomy under local anesthesia with a blunt probe placed inside to break up loculations then placement of a wick to allow for drainage and a compression bandage. Diagnosis of prepatellar bursitis is by swelling superficial to the patella and relatively painless ROM of the knee. Septic cases are usually asymptomatic, but may present with erythema, warmth and fever. Recommended treatment for the initial episode of prepatellar bursitis is aspiration (sending fluid for culture, gram stain, etc), compressive wrap, NSAIDS, and immobilization for approx 1 week. Then you can allow the wrestler to return to workouts with a knee pad.
If there is a recurrence, the above is repeated with a longer period of immobilization.
Surgery (bursectomy) may be considered at this point, if it is at an appropriate point in the season. They noted that there were no recurrences after surgery and in some wrestlers treated non-op there were up to 15 recurrences.
In the general population, however, surgery is rarely needed.
In the above question, the wrestler had a possible source of infection (abrasion) and has erythema so the best choice is aspiration.
back to this question next question
59.03 A 28-year-old man underwent surgical fixation for an intra-articular distal humeral fracture 8 weeks ago now reports progressively restricted elbow motion. Radiographs at the time of union are shown in Figures 13a and 13b. Management should now consist of
-
oral indomethacin.
-
Irradiation with a single dose of 700 cGy.
-
Physical therapy with dynamic splinting.
-
Physical therapy and delayed ectopic bone excision at 12 months.
-
Immediate elbow release and ectopic bone excision.
-
back answer Question 59.03
Answer = 5
The radiographs show HO posteriorly in the triceps tendon and also anteriorly in the tendon. The fracture appears well-healed. At this point, oral indomethacin or single dose irradiation would not help as the HO is already there and these are typically used to prevent HO. Option #3 also would not help since there appears to be more of a bony block than soft tissue contracture. This leaves options 4 and 5. In the past, ectopic bone resection was
delayed until the heterotopic ossification was “mature”. This was signified by a cold bone scan and normal serum alk phos, as well as a mature appearance on xray. It was thought that by waiting until the HO was mature, recurrence would be avoided. However, in the cited reference, out of the widely read Journal of Hand Surgery, the authors obtained good results with increased range of motion, resolution of cubital tunnel syndrome and no recurrence of contractures or loss of motion with excision of ectopic bone and elbow release that was performed once bony union of fracture was obtained. They also used a 5 day course of indomethacin post-op.
back to this question next question
60.03 What is the most significant mechanism by which a cell controls its phenotype?
-
Protein degradation
-
mRNA degradation
-
Translation efficiency
-
Transcription
-
Post-transcriptional mRNA processing
back answer
Question 60.03
Answer = 4
All the answers given are ways that a cell controls its phenotype (physical characteristics expressed by an individual). The level of expression of certain gene products will determine the phenotype. Transcription is the synthesizing of RNA from a gene. Translation is making proteins from the mRNA. The best way therefore to control the phenotype is transcription because the cell does not waste energy synthesizing useless RNA. back to this question next question
61.03 A patient with carpometacarpal joint arthritis of the thumb undergoes trapezium excision and interposition arthroplasty. One year after treatment, radiographs reveal that there has been 25% subsidence of the thumb metacarpal compared with its preoperative height. This degree of subsidence will have what effect on the surgical outcome?
-
Will not affect functional outcome
-
Will result in diminished thumb motion
-
Will result in diminished pinch strength
-
Will result in diminished grip strength
-
Will result in moderate activity-related pain back answer Question 61.03
Answer = 1
Yang and Weiland, from New York studied subsidence after trapezium excision and interposition arthroplasty. They compared preoperative and postoperative x-rays at rest as well as with maximal lateral key pinch stress. Lateral key
pinch stress causes a large amount of axial compressive force through the CMC joint. They also compared preoperative and postoperative functional measurements, including key pinch strength, tipto-tip pinch strength, grip strength, and thumb ROM. Their findings were that postoperatively the first metacarpal subsided 21% at rest (p=0.001). and it subsided another 10.5% during maximal lateral key pinch. Even with this degree of subsidence, the patients experienced increases in pinch strength and grip strength. Also, all patients except for one who had bony impingement reported that they were subjectively much better and more functional.
Lin et. al. also studied trapezium excision and interposition arthroplasty and found no significant differences between results in patients with varying degrees of subsidence. Thus, subsidence does not affect the results of interposition arthroplasty for basal thumb arthritis.
back to this question next question
62.03 Early failure of a unicompartmental knee arthroplasty that is the result of polyethylene wear is primarily caused by
-
malalignment
-
instability
-
metal backing of the tibial component
-
gamma irradiation sterilization and shelf storage in air 5- obesity.
-
back answer Question 62.03
Answer = 4
When components are sterilized with gamma irradiation, there is the formation of a large number of free radicals, making the polyethylene prone to oxidation and decreasing the mechanical toughness. The cited article by Engh and colleagues reported on early failure of unicompartmental knees with an all poly tibial component that had been sterilized with gamma irradiation and had a prolonged shelf life, (>4 years). All the components that were revised showed visible wear, and some were fragmented with full thickness fractures of the polyethylene. They sent the first 4 retrievals for studies of oxidation and all were found to be highly oxidized.
back to this question next question
63.03 Figures 14a and 14b show the MRI scans of a 38year-old man who has an enlarging, painless mass in the left distal thigh. A biopsy specimen is shown in Figure Question 63.03
Answer = 5
![]() |
![]() |
![]() |
![]() |
![]() |
![]() |
![]() |
![]() |
![]() |
![]() |
![]() |
![]() |
![]() |
![]() |
14c. What is the most likely diagnosis?
-
Lipoma
-
Desmoid tumor
-
Fibrosarcoma
-
Angiosarcoma
-
Liposarcoma
Figures 14a and 14b show a soft tissue tumor in the posterior distal thigh that is dark on T1 and bright on T2. The signal is heterogeneous and is fairly well circumscribed. Figure 14c shows a histology slide that has fat globules in it.
Liposarcoma is the second most common soft tissue malignancy. Peak incidence is between the sixth and seventh decades, and there is a predominance of males. The thigh is a common site. Metastatic rate is closely linked with differentiation of the tumor. Lung is the primary site of metastases. Overall 5 year survival rate for extremity liposarcoma is approximately 70%.
back to this question next question
64.03 Which of the following findings is one of the diagnostic criteria for diffuse idiopathic skeletal hyperostosis?
-
Flowing ossification along the anterolateral aspect of at least four contiguous vertebrae
-
Disk space collapse in the involved vertebral segments
-
Marginal syndesmophytes over four contiguous vertebrae
-
Sacroiliac erosion or sclerosis
-
Facet joint ankylosis Question 64.03
-
Answer = 1
back answer
Diffuse idiopathic skeletal hyperostosis is a common disease, most prevalent in those over 50 years of age. The usual presentation is a middle-aged or older patient with chronic mild pain in the middle to lower back, spinal stiffness, and typical radiographic changes in the thoracic spine. Diffuse idiopathic skeletal hyperostosis is predominantly a radiographic diagnosis with 3 major diagnostic criteria. 1. Flowing ossification along the anterolateral aspect of at least four contiguous vertebrae. 2. Preservation of disk height in the involved vertebral segment; the relative absence of significant degenterative changes, such as marginal sclerosis in vertebral bodies or vacuum phenomenon. 3.
Absence of facet-joint ankylosis; absence of sacroiliac erosion, sclerosis, or intra-articular osseous fusion. Treatment is typically non-operative, with anti-inflammatories, activity modification and PT.
back to this question next question
65.03 An 18 month old child has bilateral “corner fractures” of the distal femoral metaphyses of unknown origin. Following a skeletal survey, the first step in management should consist of
-
notification of child protection services
-
bilateral long leg casts and discharge
-
bilateral percutaneous pinning, long leg casts, and discharge
-
hospital admission and Bryant’s traction. 5- Hospital admission and modified Bryant’s traction Question 65.03
Answer = 1
The key to this question consists in realizing that corner fractures are highly diagnostic for battered children. Thus, the first step in management is to notify protective services. The cited references however remind us that any broken bone could be indicative of child abuse.
back to this question next question
66.03 What is the most common location for longitudinal peroneus brevis tendon tears?
-
Musculotendinous junction
-
Proximal to the fibular groove
-
At the fibular groove
-
At and below the peroneal tubercle of the calcaneus
-
At the point of insertion on the fifth metatarsal.
-
Question 66.03
back answer
Answer = 3
The cited reference by Sammarco in Cincinnati, described 12 peroneus brevis tendon lesions associated with ankle ligament injuries in 11 patients that were found during lateral ankle ligament reconstruction with a split peroneus brevis tendon graft. In all patients, the primary symptoms were pain with activity and instability. During 47 ankle reconstructions, 11 were found to have longitudinal rents in the peroneus brevis tendon. Most had significant ankle trauma, predominant symptoms of pain, averaging 8 ½ years, and tenderness over the peroneal tendons at or distal to the lateral malleous. The defects were found on the deep aspect of the tendon or through and through as it comes in contact with the lateral malleolus during excursion.
back to this question next question
67.03 Figures 15a through 15c show the plain radiograph, CT scan and biopsy specimen of a 23-year-old man with acute onset hip pain. History reveals that he underwent a trochanteric
Question 67.03
![]() |
![]() |
![]() |
![]() |
![]() |
![]() |
![]() |
![]() |
![]() |
![]() |
![]() |
![]() |
![]() |
![]() |
![]() |
![]() |
advancement at age 6 years. What is the most likely diagnosis?
-
Osteomyelitis
-
Lymphoma
-
Eosinophilic granuloma
-
Tuberculosis
-
Ewing’s sarcoma
![]() |
back answer
Answer = 1
Figure 15a shows areas of permeative lucency and sclerosis in the proximal femur as well as evidence of a screw (from the previous trochanteric advancement). Figure 15b shows the CT scan of the femur with possible thickening of the cortex and a moth eaten appearance of the bone. Figure 15c shows the biopsy specimen with mixed inflammatory cells. These are all indicative of osteomyelitis. The first radiographic sign of osteomyelitis tends to be an ill-defined area of lucency, followed by areas of sclerosis and periosteal new bone formation as the bone reacts to the infection. Biopsy specimens should show mixed inflammatory cells.
back to this question next question
68.03
A 54 year old man sustains a patellar fracture in a fall. Examination reveal that the patient can perform a straight leg raise on the injured side. Radiographs reveal a transverse fracture with 2 mm of displacement of the articular surface and no comminution. Management should consist of
-
immobilization in a long leg cast and no weight bearing
-
immobilization with the knee straight and full weight bearing
-
no immobilization and full weight bearing
-
a tension band wiring construct
-
lag screw fixation 68.03
Non-operative treatment is advocated by most authors for non-displaced fractures with a preserved extensor mechanism. Indications for this form of treatment include fragments without significant displacement, minimal disruption of
the articular surface, and an intact extensor mechanism. Bohler accepted 2-3 mm of fragment separation. Brostrom accepted 3-4 mm of fragment separation and 2-3 mm of step-off in the articular surface. If a tense hematoma is present, aspiration is recommended by Brostrom; carpenter and collegues; depalma; and smillie.
back answer
allowed full weight bearing with crutches for support. Quadriceps exercises with straight leg raises are begun at 2-3 days after
Operative treatment is recommended in fractures with more than 2 mm of articular displacement or 3mm of fragment separation.
Rockwood andGreen: Fractures in Adults ed 4. Philadelphia, Pa, lippincottRaven,1996, pp1960-1962.
69.03
In which of the following studies can an incidence rate be determined?
-
Cross-sectional
-
Prospective cohort
-
Case Control
-
Randomized trial
-
Retrospective review
-
69.03
Answer 2
back to this question next question
As usual the artical were wonderful in delivering the answer. Except for their titles, they did not even come close to answering the question above. I found this site online “A Students guide to the Medical Literature” University of Colorado Health Sciences Center. http://denison.uchsc.edu/SG/pda/pdamain.html
The Glossary of stastical terms could be useful if you have any question reguarding stastics, formulas etc. I took from their extensive glossary the terms relative to the question . It should be sufficient.
cross sectional survey
A survey of a population at one point in time. Two types of cross sectional surveys are an interview survey and a mass screening program. A cross sectional survey is useful for looking at disease prevalence but is unable to establish a temporal relationship between a presumed cause and an effect.
cohort study
A prospective study that follows one or more groups of patients over time. Cohort studies are used to describe etiology or prognosis of a disease. A cohort is a clearly defined group that has some exposure(s) in common. A less common type of cohort study is a retrospective cohort study, which looks back in time to define risk groups.
case control study
A retrospective study which compares diseased with non-diseased individuals to establish the influence of a prior exposure. The actual risk of disease cannot be determined from a case-control study, since the underlying population is not accurately represented.
Instead, the odds ratio is calculated to provide an estimate of the relative risk due to the exposure. Case control studies are subject to recall bias. However, a case control study may be the best way to study rare diseases.
randomized control trial (RCT)
A study of treatment in which enrolled patients are randomly assigned to a treatment group or a control group. RCTs are said to be "double-blind" if neither the subjects nor the observers in the study are aware of the patient’s treatment (experimental treatment vs.
standard treatment or placebo). RCT’s are considered the gold standard for studying interventions, because of their ability to minimize bias in information obtained from the study subjects.
“A Students guide to the Medical Literature” University of Colorado Health Sciences Center. http://denison.uchsc.edu/SG/pda/pdamain.html
back answer
70.03
Which of the following factors is considered predictive of poor wound healing potential following lower extemity amputation?
Serum Albumin level of 4.0 g/dl
Absolute lymphocyte count of 1,750/mm3
![]() |
TcpO2 of 15 mm Hg Ankle-brachial index of 0.7
Toe pressures of 40 mm Hg 70.03
![]() |
![]() |
Answer 3
This information is tested at least once every year:
![]() |
Predictors of wound healing include:
![]() |
![]() |
Serum albumin less than 3.5 g/dl indicates malnutrition
![]() |
Total lymphocyte count less than 1,500/mm3 indicates and immune deficiency
![]() |
Either of the above are associated with 40-50% wound breakdown.
![]() |
![]() |
Poor serum glucose levels is also associated with poor wound healing and infections.
Ankle-Brachial index is defined as the ratio of the systolic blood pressure measured at the brachial artery to that measured at the
ankle. An index of less than 0.45 indicates incisions distal to the ankle will be unlikely to heal.
back to this question next question
Ankle pressure indices may be falsely elevated in the patient with peripheral vascular disease, because of vessel wall calcification. Because digital vessels rarely calcify in peripheral vascular disease toe pressures have been shown in some studies to be more reliable in these patients.
![]() |
Toe pressures less than 30mm of Hg being highly predictive of the need for amputation.
Doppler indices have been described for planning amputation level. Doppler pressure measured at the planned level of amputation divided by the doppler pressure at brachial artery. An index > 0.5 is considered indicative of minimal acceptable vascular inflow.
![]() |
![]() |
![]() |
![]() |
Transcutaneous oxygen pressure measurements assess the partial pressure of oxygen through the skin. This noninvasive test, which has become the gold standard measurement of vascular inflow, can be performed at any location on an extremity that has intact overlying skin. Recent studies have documented 88% sensitivity and 84% specificity of the test. A threshold value of less than 20 mm Hg has been shown to yield high rates of
wound healing complications.
Transcutaneous oxygen pressure may be falsely low in patients with swollen extremity, cellulitis or venous stasis changes.
Koval KJ (ed): Orthopaedic Knowledge Update 7. Rosemont IL., American academy of orthopaedic surgeons, 2002, pp 127-137.
71.03
A patient has numerous light brown skin macules, with freckling in the axillae and groin. Tibial radiographs are shown in Figures 16a and 16b. He has a defect in the gene coding for
-
merlin.
-
osteoclast-activating factor.
back answer
-
prostaglandin E2.
-
neurofibrillin.
-
dystrophin.
back to this question next question
71.03
![]() |
Answer 4
![]() |
Neuro fibromatosis type 1 is the most common disorder known to be caused by a mutation in a single gene, occurring in about 1 in 3000 newborns. NF-1 is autosomal dominant. The condition is caused by a mutation in the gene that
encodes for the protein neurofibrillin. Neurofibrillin plays a role in the Ras cell signal pathway. Mutation form of neurofibrillin increases the Ras signaling and abnormal cell growth. Most neoplastic processes are benign, but these children have a propensity for Wilm’s tumor and rhabdomyosarcoma. Clinical findings of NF-1 include cafe-au-lait spots, neurofibromas, axillary freckles, Lish nodules (retinal hamartomas), and bone deformity, most of which are absent in the newborn, but develop over time. Scoliosis is common. Most of which resembles the idiopathic type and is managed in similar fashion. Some are dystrophic, involving short segment 4-6 levels. Pseudoarthrosis of long bones is typical and the tibia is most common. Anteriolateral bow is precursor to pseudoarthrosis.
Koval KJ (ed): Orthopaedic Knowledge Update 7. Rosemont IL., American academy of orthopaedic surgeons, 2002, pp 209-218.
back to this question next question
72.03
![]() |
What is the most common site of fracture because of osteolysis following cementless total hip arthroplasty with an extensively porus-coated femoral component?
-
Central acetabulum
-
Ischial ramus
back answer
-
Greater trochanter
-
Between the lesser trochanter and the stem tip
-
Femoral stem tip 72.03
![]() |
Answer 3
Again referenced article is not helpful. Here is the thought process. Porous coating allows for bony ingrowth, and
![]() |
creates stress shielding. The force is seen through the femoral head into the stem and transfers to the bone at areas of
ingrowth and transmitted distally. As a result the part being skipped is the greater trochanter and leads to osteolysis by Wolf’s law and can result in fracture.
![]() |
The osteolysis created about the acetabulum is due an inflammatory mechanism imparted on the wear debris from the bearing surfaces. This osteolysis rarely creates fractures.
![]() |
The fractures about the stem are most commonly caused by the difference in modulus between the stem and bone back to this question next question
73.03
What is the most common infection occurring in a toddler’s and preschooler’s hand?
-
Paronychia
-
Felon (pulp infection)
-
Thenar space abcess
-
Herpetic whitlow
-
Septic flexor tenosynovitis 73.03
“The most common infection in the toddler and preschooler’s hand is herpetic whitlow, a viral infection caused by
Herpetic whitlow is selflimited and surgical treatment is not indicated. Children are also susceptible to hand infections as a result of bite wounds. The treatment for bite wounds in children (debridement and prophylactic antibiotics) is same for adults.”
back to this question next question
back answer
74.03
A 35 year old man who sustained a comminuted radial head fracture in a fall undergoes surgery the night of the injury. Post operative radiographs are shown in figures 17a and 17b. The anticipated success of this treatment would most likely be compromised if the
-
patient is employed as a computer operator.
-
Patient receives no prophylaxis for heterotopic ossification.
-
Patient has tenderness at the distal radioulnar joint.
-
Annular ligament is disrupted.
-
Elbow is splinted for 4 days before beginning range of motion exercises
-
back answer 74.03
![]() |
Answer 3
![]() |
Treatment of
type III(comminuted)
fractures generally is surgical.
The critical factor in the treatment of these
fractures is the determination of associated injuries. Excision alone should be reserved for patients in whom no origional episode of elbow subluxation or dislocation occurred, who are free of forearm or wrist pain or pathology, who have no associated coronoid fracture, and in whom the elbow is stable to valgus stress at 30deg of flexion after resection. If any of these conditions do not exist, then a metallic radial head implant, which has superior biomechanical properties to the previously available silastic implants, should be inserted.
![]() |
OKU: Trauma 2 , Rosemont, IL., American academy of orthopaedic surgeons, 2000, p45 back to this question
next question
75.03
A 92 year-old man who lives alone and is a community ambulator sustains a displaced femoral neck fracture. Following a detailed informed consent discussion that recommends surgery, he refuses treatment. What is the next most appropriate step.
1 Go ahead with the surgery after obtaining administrative consent. 2 Obtain a second orthopaedic opinion.
3 Obtain a consent from his next-of-kin
4 Assess the patient’s global mental capacity, including orientation to person, time, and place
5 Evaluate the patient’s specific decisional capacity by gauging his understanding of his condition, treatment alternatives, and consequences.
75.03
![]() |
Answer 5
![]() |
“The descision to respect or to override a patient’s refusal reflects an autonomous choice. Autonomy is the ability to be a good ruler over ones self. It involves: 1) the ability to adopt values, principles and goals; 2) the ability to understand: 3) the ability to make choices that reflect one’s commitments: and 4) the ability to act on them.”
back answer
![]() |
Capozzi JD, Rhodes R: Assessing a patient’s capacity to refuse treatment. JBJS 2002;84:691-693. back to this question next question
76.03
A 37 year-old woman with thoracic back pain has had hemoptysis and dyspnea for the past month. A thoracic spine radiograph shows a compression fracture of T-9 and a mass in the right lung. What is the best course of action?
-
Metastatic work-up
-
Physical therapy
-
Transpedicular biopsy
-
Thoracotomy with partial resection of the mass
-
T9 corpectomy with anterior column reconstruction 76.03
![]() |
Answer 1
![]() |
Here is an algorithm from OKU 6 Tumors of the Spine pp 723-736
![]() |
![]() |
This question gives you the history, skips right over physical exam and goes into plain films. With this patient’s history of hemoptysis and dyspnea and a lung mass, the next steps in this patients work-up would include the competion of initial work-up, metastatic work-up, then pre-operative planning. Goals of treatment are to 1) protect or restore neurologic function, 2) control pain,
![]() |
![]() |
3) maximize physical capacity and 4) obtain local control of the disease. Other trivia from the references include:
![]() |
![]() |
![]() |
![]() |
![]() |
![]() |
![]() |
![]() |
![]() |
![]() |
![]() |
![]() |
![]() |
![]() |
After the lung and liver the skeletal system is the third most common site of metastasis. The spine is the most common site of skeletal metastasis. 60% of all skeletal lesions and 36% are asymptomatic. Breast, prostate, lung and renal carcinoma comprise 80% of the carcinomatous skeletal metastasis.
![]() |
70% metastasis occur in the thoracic and thoracolumbar regions. 21% had involvement of the lumbar and sacral regions. 8% involved the cervical and cervicothoracic regions together.
![]() |
As many as 90% of patients who die of cancer may have Spinal metastasis at autopsy, and only half of patients who die from cancer will have symptoms from spinal mets. Fewer than 10% with spinal mets are treated surgically. Beaty JH (ed): Orthopaedic Knowledge Update 6. Rosemont IL., American academy of orthopaedic surgeons, 2002, pp 723-736.
back to this question next question 77.03
What is the most frequently encountered complication following juvenile hallux valgus correction?
-
Recurrence of the deformity
-
Hallux varus
back answer
-
Transfer metatarsalgia
-
Nonunion of the first metatarsal osteotomy
-
Osteonecrosis of the first metatarsal osteotomy 77.03
![]() |
Answer 1
![]() |
Coglin’s referenced paper is a study out of Idaho. 11 year retrospective study of 45 patients (60 feet). A multiprocedural approach was used to surgically correct the deformity. There were 6 recurrences of the deformities and eight
complications (6 cases of hallux varus, one case of wire breakage and one case of undercorrection.) So according to their reference this question has two correct answers.
![]() |
![]() |
Postoperative complications have been frequently reported following juvenile hallux valgus corrections. Recurrence following surgery is probably the most frequently reported complication and is likely due to the high rate of congruency associated with a juvenile hallux valgus deformity.
![]() |
OKU Foot and Ankle 2 Rosemont IL., American academy of orthopaedic surgeons, pp135-150.
back to this question next question
78.03
The primary mechanism by which nonsteroidal antiinflammatory drugs exert their effectiveness is inhibition of
-
lipoxygenase
-
leukotrienes
-
phospholipase
-
cyclooxygenase
-
thromboxane A2
78.03
![]() |
Answer 4
![]() |
NSAIDs inhibit the enzyme cycolooxygenase (COX) by a variety of mechanisms. Asprin binds covalently with a serine residue of the enzyme, leading to an irreversible steric henderence of the active site. Ibuprofen and piroxican, however
are reversible competative inhibitors of COX. Others such as indomethacin and diclofenac have the ability to act on the lipooxygenase side of the aracadonic metabolism pathway, leading to inhibition of the leukotriene inflammatory mediators.
![]() |
Cortico steroids inhibit phospholipase A2. Phospolipase inhibition blocks both the Cox and Lipoxygenase pathways of the arachidonic acid metabolism.
![]() |
Buckwaleter JA, Einhorn TA, Simon SR (eds); Orthopaedic Basic Science: Biology and Biomechanics of the Musculoskeletal System, ed 2. Rosemont IL., American Academy of Orthopaedic Surgeons, 2000 pp217-237.
back answer
back to this question next question
![]() |
![]() |
![]() |
79.03
A 22 year old man who sustained a gunshot wound to the hip 5 months ago is unable to anbulate without crutches and reports a recent increase in hip pain with activity. Examination reveals no open wound, and the neurologic and vascular status of the extremity is intact.
Radiographs are shown in figures 18a and 18b. Management should consist of 1. iliac crest bone grafting.
-
hemiarthroplasty.
-
revision of cannulated screw fixation.
-
valgus intertrochanteric osteotomy.
-
quadratus femoris muscle pedicle bone graft.
back answer
79.03
Pictured is a femoral neck fracture non-union. Non-union and avascular necrosis of the femoral head are the main complications of fracture of the femoral
neck. In spite of improved operative techniques, non-union is still reported in some 10-20% of cases. The problem is essentially biomechanical, as demostrated by Pauwels over 50 years ago. For treatmetn he described an abduction osteotomy at the intertrochanteric level, which converts shearing forcesinto compression.
osteotomy provides a high proportion of good results even in the presence of AVN, providing there has been no collapse. If the osteotomy fails, prosthetic replacement is still possible.
back to this question next question
80.03
A child who has a malunion of a supracondylar humeral fracture now has a typical cubitus varus(gunstock) deformity. What is the most common indication for early surgical correction of this deformity?
-
poor cosmesis
-
Tardy ulnar nerve palsy
-
posterior shoulder instability
-
rotatory subluxation of the radiocapitellar joint
-
anterior subluxation of the medial triceps back answer
-
80.03 ans-1
![]() |
Just a quick review- SCH fx, most common, extension type, deformity is posteromedial – varus deformity, deformity posterolateral – valgus deformity. Baumans angle, baumans angle, baumans angle – “best treatment for cubitus varus is prevention” tachadjian. Restore baumans angle, with treatment, either casting or cr/or pp. the key to this question is “most common”, cubitus varus rarely causes a problem clinically to the patient- usually some slight loss in flexion, tardy ulnar nerve palsy comes w/ valgus deformity usually lateral condyle fx and the 3 and 5 are rare complications sited in the articles. The deformity is more a problem for the parents than the kids and the solution is a lateral closing wedge osteotomy.
81.03
Nonsurgical management of cubital tunnel syndrome includes use of a night splint that should be designed to maintain the elbow at 1-45° and the forearm in pronation
2-45° and the forearm in supination 3-45° and the forearm in neutral
4-90° and the forearm in neutral
5-90° and the forearm in pronation back answer
81.03 ans. 3
back to this question next question
![]() |
![]() |
![]() |
![]() |
![]() |
The cubital tunnel serves as major constraint for the ulnar nerve as it passes behind elbow; the syndrome occurs most commonly between 30 to 60 years, and is exceptionally uncommon in children under 15 years. the internal anatomy of the ulnar nerve can partially explain the predominance of hand symptoms from in cubital tunnel syndrome; nerve fibers to the FCU and FDP are located centrally, where as sensory fibers and nerve fibers to the hand intrinsics are located peripherally; generally, the peripheral nerve fibers are more sensitive to external compression, and this may explain why the hand intrinsics are often more involved than the FCU and FDP; Well once again the cited articles don’t fully address the question.
![]() |
Conservative treatment of cubital tunnel syndrome as outlined in the hand clinics article are as follows: pts with mild symptoms or who present early are given an elbow pad which covers the posteromedial elbow to protect against trauma and to serve as a reminder to the patient to keep their elbow extended, they also recommend placid a pillow or small towel in the antecubital fossa to prevent elbow flexion beyond 45º, If the symptoms are worse with constant pain and paresthesias a thermoplastic splint is constructed to keep the elbow at less than 45º of flexion, and the picture shows the forearm in neutral, however they make no specific mention of it in the text. I suppose the neutral provides the most relaxed position for the nerve at the elbow? w/ good compliance 50% of patients can avoid surgery; avoidance of repetitive elbow flexion and pronation, and avoidance of vibrating tools
82.03
Figures 19a and 19b show the radiographs of a 14 yr-old boy who fractured his talus and ankle 12 weeks ago. What conclusion can be drawn based on the radiographic findings?
-
osteomyelis is present
-
complex regional pain syndrome is present
-
malunion is present
-
union is absent
-
osteonecrosis is absent
back answer
82.03 ans. 5
![]() |
the xrays show an ankle w/ multiple screws in the talus and tband in the medial malleolus. All you can say about the acute injury is that it was a bad fx, based on the amount of screws in place and it was probably displaced (they fixed it) – cant really say if it was Hawkins II-IV. The answers are a tad subjuective but here is my take. Osteo – nah, no real bony lesions to suggest it, ie periosteal rx, ST swelling; CRPS – tricky, you could say there is some osteopenia in the talus and foot but you’d be kind of a nutjob to jump to that conclusion based on the xray w/ no hx; malunion – no reduction l;ooks anatomic; union is absent – fx lines are absent or barely present so I wouldn’t say that; AVN is absent, huh, AVN…Talus…, that Hawkins dude… oh shit Hawkins sign, what the crap is that again. Subchondral atrophy in the dome of the talus in the AP view 6-8wks after injury. The presence of this atrophy means that the vascularity has been maintained to the talar body. AVN usually presents by 3months with sclerosis of the body more than the surrounding bone. So in this xray the is definitely no sclerosis, right! Maybe some atrophy but with the following ans choices 5 is the only one that will get you double potatoes from k-rob.
back to this question next question
83.03
What is the best predictor for a low back injury in a collegiate athlete?
-
Type of sport
-
Gender
-
History of back injury
-
Level of athletic experience
-
Satisfaction with coaching staff
back answer
83.03 ans. 3
84.03
Which of the following substances has been implicated in the pathogenesis of osteoarthritis?
-
Interleukin 1
-
Tissue inhibitor of matrix metalloproteinase
-
Cyclooxygenase 1
-
BMP 2
back to this question next question
-
Antigen-specific T cells
-
84.03 ans.1
![]() |
![]() |
![]() |
Unfortunately this is kind of one of those memory questions. There is a great chart in Miller review text on the cascade of OA. It basically a food chain w/ IL-1 on top.
IL-1 is secreted by monocytes and chondrocytes in the face of inflammation and stimulates the release of enzymes from chondrocytes which break down cartilage matrix.
Any more info than that and you need to be in Cook’s lab and not the operating room. Lets move on!
85.03
Figures 20a and 20b show the radiographs of a 14-year-old boy who had LeggCalve-Perthes disease when he was 8 years-old. He now reports hip pain, poppinp, and locking. Treatment should consist of?
-
incisional biopsy
back answer
-
hip fusion
-
removal of loose body
-
total joint arthroplasty
-
core decompression
85.03 ans.3
This kid has an OCD and probably a loose body related to it. OCD’s occur in 3% of kids with LCP, usually late onset ( age>8) and has a prolonged ineffectual repair.
86.03
An otherwise healthy 64-year-old man undergoes total shoulder arthroplasty for osteoarthritis. Because of a long-standing internal rotation contracture, a subscapularis lengthening procedure is performed to rebalance the joint. What important concept needs to be kept in mind regarding postoperative physical therapy?
-
Initiation of physical therapy must be delayed by several weeks.
back to this question next question
-
Elevation should be performed in the sitting position. 3- External rotation isometrics should be avoided.
-
External rotation stretching should be limited
-
Internal rotation should be limited to eccentric techniques.
86.03 Ans.4
87.03
A 27-year-old patient sustains the closed femoral fracture shown in figures 21a and 21b. This fracture pattern is most likely the result of which of the following forces?
back answer
-
Pure torsion.
-
Pure bending.
-
Pure compression.
-
Four-point bending.
-
Torsion plus bending.
-
87.03 Ans. 2
Pure bending. – transverse fx
Pure compression. – crush/comminuted depending on surface area of injury
back to this question next question
Torsion plus bending – long spiral w/ butterfly 88.03
Highly cross-linked ultra-high molecular weight polyethylene has what effect on the tensile and fatigue strength when compared with ultra-high molecular weight polyethylene.
-
increased tensile and fatigue strength.
-
Increased tensile strength and decreased fatigue strength.
-
decreased tensile and fatigue strength.
-
decreased tensile strength and no change in fatigue strength.
-
no change in tensile and fatigue strength.
88.03 Ans. 3
back answer
UHMWP. It essentially becomes harder so it is more prone to crack propagation under severe fatigue. So its tensile and fatigue strength decrease, i.e. it is less resistant to tensile and fatigue stresses. Apparently these properties are advantageous in low contact stress joints such as the hip, where surface wear mechanisms predominate (abrasion and adhesion) and have exhibited significantly lower wear in hip simulators in–vitro. However in the knee joint fatigue wear mechanisms predominate, such as pitting and delamination which we now know HC-UHMWP is not as resilient to.
back to this question next question
89.03
A newborn is undergoing serial manipulation for congenital idiopathic clubfoot deformity. Which of the following deformities should be corrected last?
-
Forefoot pronation.
-
Forefoot adduction.
-
Medial talonavicular subluxation 4- Hindfoot varus. 5- Ankle equinus. back answer
89.03
The deformities associated with idiopathic clubfoot include equinus of the
Hindfoot, varus (inversion) of the subtalar joint complex, cavus (plantarflexion of the forefoot on the Hindfoot), and adductus of the forefoot on the midfoot. For those of you who have not seen TEV (talipes equinovarus) it is unmistakable, the kid is scratching his medial shin with his big toe! The posetti casting method basically addresses everything in the initial casts except the Hindfoot equinus, which is the last thing to correct and usually needs at least at heel cord release. The casting should begin as soon as possible w/in 1-2 weeks of birth. The forefoot is basically supinated and abducted with digital pressure on the head of the talus (laterally). It can take 6-8 weeks, i.e. 6-8 casts then the equinus is addressed with attempts to dorsiflex the calcaneus, not the midfoot! Or you will get rocker bottom foot. Bennett thinks most kids need a PMR to release the posterior capsule as well as the Achilles which is also contracted.
Ans. 5
90.03
The pelvic radiograph in figure 22 reveals which of the following findings?
-
Spur sign.
-
Femoral head subluxation.
-
Femoral head fracture.
-
Anterior wall acetabular fracture.
-
Posterior wall acetabular fracture.
-
back to this question next question
90.03
91.03
back answer
Ans. 5
The bony process shown in figure 23 may be associated with compression of which of the following nerves.
-
Ulnar.
-
radial.
3 . Musculocutaneous 4- Posterior interosseous.
5- Median.
91.03
to be confused with the arcade of struthers. This
is a basic anatomy/memory question, the
median N. runs right under that piece of bone
and can be compressed by it or the ligament of struthers, Supracondylar Process of the Humerus and Struther's Ligament: - median nerve may be
compressed as it passes under the bondy spur
- pt c/o of weak hand grip and in
pronating, & tingling in lateral digits; -
there may be objective weakness in the
pronator teres, FCR, FPL, FDS, and lateral half of FDP, and thenar mucles; - presence of pronator weakness distinguishes this syndrome from pronator and anterior interosseous syndromes; nerve conduction studies may have shown slowing in the conduction velocity in the median nerve in the arm;- EMG may show denervation potentials in median innervated muscles in the forearm and hand; - clinical syndrome may be associated with symptoms and signs of ischemia in the distribution of the
back to this question next question
brachial artery; - bony process may or may not be palpable; - tangential or oblique views of humerus may show supracondylar process to greater advantage than routine AP & lateral views; - brachial arterogram may be necessary if vascular comprimise is possible -surgical therapy consists of excision of Struther's ligament and removal of the bony spur;
92.03 Figure 24 shows the radiograph of a 74-year-old patient with type I diabetes who has insidious onset ankle pain. What is the most likely diagnosis?
-
Syndesmotic disruption
-
Osteosarcoma
-
Maisonneuve fracture
-
Neuropathic fibular fracture
-
Osteomyelitis of the fibula
back answer
Question 92.03
Answer = 4
A maisonneuve fracture would show an X-ray of a proximal spiral fibula fractured and so this is an unlikely diagnosis. The other three incorrect answers are possible answers, but testmanship eliminates them. A syndesmotic disruption would be indicted on AP and mortise views of the ankle by a fibula fracture at least 4.5 cm above level of mortise, asymmetry of the ankle mortise, a medial clear space >4 mm, or tibia/fibula overlap < 6-10mm . A lytic lesion with questionable soft tissue calcification seen in the X-ray for this question is possibly consistent with osteosarcoma.
Furthermore, osteomyelitis could also appear like this X-ray. While this lesion is 4.5 cm above the mortise and could be either osteosarcoma or osteomyelitis the details of the question provide the answer. This is a patient with insidious onset ankle pain but she has diabetes. If it were a tumor they would have likely provided pathology, if it were infection they would have likely given lab values or other clues, if it were a syndesmotic or Maisonneuve injury they would have given a mechanism of injury.
Reference(s)
back to this question next question
Crim j: imaging of the Foot and Ankle. London, England, Martin Dunitz, 1996, 127-131.
93.03 Figures 25a through 25e show the plain radiograph, MRI scans, bone scan, and biopsy specimen of a 14-year-old girl who has
increasingly worse left hip pain. What is the most likely diagnosis?
-
Nonossifying fibroma
-
Desmoplastic fibroma
-
Osteofibrous dysplasia
-
Fibrous dysplasia
-
Fibrosarcoma
back
Question 93.03
Answer = 4
answer
Fibrous dysplasia is a developmental anomaly of the bone-forming mesenchyme in which the maturation of bone is arrested at the immature woven type. It can be mono- or polyostotic and radiographically appears as deep endosteal scalloping and prominent ground glass matrix. The proximal femur may develop a Shepard’s crook deformity and other long bone deformities may develop. Histologically it looks like “Chinese letter.” Albright’s syndrome is a combination of polyostotic fibrous dysplasia, café-aulait spots, and endocrinopathies.
Reference(s)
Keijser Lc, Van Tienen TG, Schreuder HW et al: Fibrous dysplasia of bone: Management and outcome of 20 cases. J Surg Oncl 2001; 76:157-166.
back to this question next question
94.03 The strongest bone-pin interface will be achieved with external fixation pins that are coated with
-
calcium sulfate
-
titanium
-
hydroxyapatite
-
polyglycolic acid
-
silver
back answer
Question 94.03
Answer = 3
Hydroxyapatite is the calcium phosphate that comprises the inorganic mineralized component of bone. It can be plasma sprayed onto metal surfaces and as such is a bioceramic material with high biocompatibility and low degradation. It is capable of forming a tight, durable link with the underlying metallic surface and living bone tissue. This improves osteointegration and consequently the mechanical stability of the bone-pin interface. Calcium sulfate is a graft material that is a void filler with high osteoconductive properties. Titanium is a softer metal often used in joint arthroplasty (although not as a bearing service - the more resilient cobalt chrome is used for this purpose). Polyglycolic acid is a resorbable polymer. I am unaware of any use for silver in orthopaedics.
Reference(s)
Moroni A, Faldini c, Rocca m et al: Improvement of the bone-screw interface strength with hydroxyapatite-coated and titanium coated AO/ASIF coritcal screws. JOT 2002;16:257-263.
back to this question next question
95.03 A 27-year-old chef sustains a traumatic injury to the index finger in an electric mixing machine. Examination reveals an isolated 2cm x 2cm loss of palmar skin over the proximal phalanx with exposure of the flexor tendons. Coverage of this defect is best accomplished by
-
an axial flag flap from the long finger
-
split-thickness skin grafting
-
full-thickness skin grafting
-
a Moberg advancement flap
-
a thenar flap
back answer
Question 95.03
Answer = 1
When a flap receives its blood supply from a single, constant vessel, it is termed axial. The axial flag flap requires no pedicle dissection because it is based on the web space of the donor finger. The dorsal metacarpal artery has been reliably shown to be present in the second interspace and can be raised from either the index or long finger. Mobility is this flaps greatest advantage and the pedicle need only be as wide as the vessels. From the dorsum of the long finger it can be rotated to cover the palmar surface of either proximal phalanx. The Moberg advancement flap is used in thumb tip amputations (of the distal phalanx). The thenar flap (as well as the cross finger flap) are often used in the repair of fingertip defects that have exposed bone and more palmar tissue loss than dorsal. The thenar flap is best used on young female patients because there is no secondary defects and the increased flexion required for this is better tolerated in her supple joints. Incidentally the cross finger flap can also be used for palmar and dorsal digital defects. In general split thickness grafts should not be used on palmar surfaces, though they can be on the dorsal surface. Full thickness grafts are seldom used on the dorsum of the hand though it they can be used on the volar surface. If there is not a good bed of vascularity (such as on bare tendon, bone denuded of periosteum, and nerves and vessels) then this graft will not work. These exposed tendons were not good vascular sources for a full thickness skin graft.
Reference(s)
back to this question next question
Iselin F: The flag flap. Plast Reconstr Surg 1973;52:374-377.
96.03 The equinovarus foot position in patients with Duchenne muscular dystrophy is caused by
-
Achilles tendon shortening and persistent function of the long toe flexors.
-
contracture of the gastrocnemius-soleus complex and spasticity of the long toe flexors.
-
contracture of the gastroc-sol complex and spasticity of the tibialis anterior.
-
contracture of the gastroc-sol complex and persistent function of the posterior tibialis.
-
spasticity of the gastroc-sol complex and the posterior tibialis muscles
back answer
Question 96.03
Answer = 4
Duchenne muscular dystrophy is a progressive neuromuscular disorder characterized by muscle weakness and lower extremity contractures. The disease is caused by a gene defect on the p21 locus of the X chromosome that results in deficiency of the muscle protein dystrophin. The ability to ambulate is usually lost by age 10. In spite of weakness of other muscles, persistent function of the posterior tibialis tendon has been noted. This is believed to be the major contributor, along with gastroc-sol contracture to the equinovarus deformity in these patients. The most common procedure performed to achieve a plantigrade foot in these patients includes Achilles tendon lengthenings, toe flexor tenotomies, and posterior tibialis tendon transfers versus tenotomies. The question can be answered by realizing muscular dystrophy causes muscle weakness, not spasticity (as is the case with cerebral palsy).
Reference(s) back to this question next question
Scher DM, Mubarak SJ: Surgical prevention of foot deformity in patients with Duchenne muscular dystrophy. JPO 2002;22:384-391.
97.03 A 17-year-old soccer player reports lateral and posterior pain after jamming her knee in a game. Examination reveals range of motion of 0 to 100 degrees and a 1+ knee effusion. The Lachman test is positive. External rotation of the tibia is increased with the knee in 30 degrees but not 90 degrees of flexion. There is increased laxity to varus stress with the knee in full extension. What is the most likely diagnosis/
-
Isolated tear of the ACL
-
Isolated tear of the PCL
-
Tears of the ACL and PCL
-
Tears of ACL and posterolateral complex 5- Tears of PCL and posterolateral complex
-
back answer
Question 97.03
Answer = 4
The lachman exam is the most sensitive test for detecting ACL ruptures and it is positive. There is no mention of a posterior drawer test so suspicion for a PCL tear should be low. This is confirmed when looking at the results of the dial test. This test is performed with the patient laying down in either the prone or supine position. It tests for PCL and/or posterolateral corner injury. With the knee flexed to 90 if there is increased passive external rotation on the tested side versus the normal side (and no difference at 30) this indicates PCL injury. With the knee at 30 if there is increased passive external rotation compared to the normal side (and no difference at 90) this indicates posterolateral corner injury. If there is increased passive external rotation at both 30 and 90 this indicates injury to both posterolateral corner and PCL. The increased varus laxity in full extension indicates damage to the LCL (part of the posterolateral complex) as well as a cruciate ligament. Isolated collateral ligament laxity is identified with varus/valgus stress at 30 knee flexion.
Reference(s)
Koval Kj (ed): OKU 7. rosemont, IL, AAOS, 2002, pp489-511.
back to this question next question
98.03 A 46-year-old man undergoes a bone grafting procedure from the anterior iliac crest. Two weeks after surgery, he reports decreased sensation about the anterior thigh. The most likely diagnosis is neuropraxia of what nerve?
-
Ilioinguinal
-
Femoral
-
Cluneal
-
Saphenous
-
Lateral femoral cutaneous
back answer
Question 98.03
Answer = 5
Injury to the lateral femoral cutaneous nerve causes numbness, burning, itching, and pain over the lateral aspect of the thigh – termed meralgia paraesthetica. When harvesting anterior iliac crest bone graft, it is recommended that an incision should be terminated 15 to 20 mm posterior to the ASIS in order to avoid injury to the LFCN. Incidences in the literature varies from 1.7% to 31%.
Although this nerve usually emerges from the lateral border of the psoas major muscle and crosses the ilium towards the ASIS, its course is variable. It occasionally runs across the iliac crest 2.0 cm posterior to the ASIS and may be injured at this site during exposure of the crest. Other variations include passing over the inguinal ligament, passing > 2cm posterior to ASIS, and crossing close to the iliacus under the inguinal ligament.
Reference(s)
Ahlmann e, Patzakis M, Roidis n et al: Comparison of anterior and posterior illiac crest bone grafts in terms of harvest site morbidity and functional outcomes. JBJS am 2002;84:716-720.
back to this question next question
99.03 What outcomes instrument is the most specific for disorders involving the lumbar spine?
1- SF-36
-
-
-
Oswestry disability index
-
Minnesota multiphasic personality index
-
Visual analog scale
-
Patient self-assessment of satisfaction
back answer
Question 99.03
Answer = 2
The Oswestry disability index is a low back pain questionnaire that has been proposed as a reliable means of scoring the disability of patients with low back pain. It was introduced in 1984 as an adjunct to decision making in surgical treatment of the degenerate lumbar spine.
Reference(s)
Deyo RA, Battie M, Beurskens AJ et al: Outcome measures for low back pain research: A proposal for standardized use. Spine 1998;23:2003-2013. back to this question next question 100.03 Figures 26a through 26e show the plain radiograph, MRI scans, bone scan, and biopsy specimen of a 62-year-old man who has progressive left shoulder pain. Immunohistochemistry staining on the tissue is positive for leukocyte common antigen and negative for CD99.
![]() |
![]() |
![]() |
![]() |
![]() |
![]() |
![]() |
![]() |
![]() |
![]() |
![]() |
Staging shows no disease elsewhere. What is the best local treatment?
-
Forequarter amputation
-
Extra-articular wide excision
-
Intra-articular wide excision
-
Intralesional curettage
-
Radiation therapy
back
Question 100.03
Answer = 5
answer
This patient had malignant lymphoma of bone. Patients generally present with pain. Xrays often have bone destruction and often has a mottled appearance. Reactive bone formation and cortical bone destruction are common; cortex may be thickened. A mixed cell infiltrate is usually present. Treatment centers around multiagent chemotherapy and irradiation. Surgery may be used in selected cases to achieve local control.
Reference(s)
Durr HR, Muller Pe, Hiller E et al: Malignant lymphoma of bone. Arch Orthop Trauma Surg 2002;122:10-16. back to this question next question
101.03 Which of the following conditions is characterized by congenital by congenital vertebral anomalies, anal and renal anomalies, tracheoesophageal fistula, and partial or complete radial dysplasia?
-
VATER syndrome
-
Goldenhar’s syndrome
-
Poland’s syndrome
-
Klippel-Feil syndrome
-
Sprengel’s deformity
back answer
Question 101.03
Answer = 1
VATER syndrome’s name constitute an acronym for the system and defect’s involved:
vertebral, anus and renal anomalies, cardiac, trachealesophageal fistula, and radial dysplasia. Goldenhar syndrome is also known as “ocular-auricular-vertebral dysplasia”, with anomalies of the eyes, ears, and vertebrae. Poland’s syndrome …. Klippel-Feil syndrome consists of congenital fusions of the cervical vertebrae clinically exhibited by the triad of low posterior hairline, a short neck, and variabily limited neck motion.
Approximately one-third of these people have an associated Sprengel’s deformity, which is a congenital undescended scapula that is manifested clinically by limited shoulder abduction to 90-100degrees as well as associated vertebral, clavicular, and renal anomalies. Poland’s syndrome is unilateral absence of pectoralis minor and sternal portion of pectoralis major with associated ipsilateral hand anomalies such as hypoplasia of the hand and digits with syndactyly, brachydactyly, and reduction deformities.
Reference(s)
Beals RK, Robbins JR, Rolfe b: Anomalies associated with vertebral malformations. Spine 1993;18:1329-1332.
back to this question next question
102.03 In patients with osteoporosis, the incidence of fracture is directly related to
-
age at the time of peak bone mass.
-
age at the onset of menopause.
-
serum calcium levels.
-
the degree of bone mass loss.
-
osteoclast and osteoblast activity.
back answer
Question 102.03
Answer = 4
Osteoporosis is a metabolic bone disease characterized by low bone mass and micro architectural deterioration of bone tissue, leading to enhanced bone fragility and a consequent increase in fracture risk. Calcium and phosphorus levels are normal, as opposed to osteomalacia. Age in and of itself is not a risk for increased fx though it is known osteoporosis type I commonly occurs in postmenopausal women, mostly involves trabecular bone, and is a high turnover disease; osteoporosis type II occurs commonly in the age of late 60’s and 70’s, is equal among men and women, is low turnover, and affects cortical and trabecular bone. While osteoclast and osteoblast activity have a role in bone turnover, it is the degree of bone mass loss that is directly related to fracture risk.
Reference(s)
Fardon DF, Garfin SR, Abitol JJ et al: OKU:Spine 2. Rosemont, IL, AAOS, 2002, pp123-133.
back to this question next question
103.03 The force required to initiate sliding of the lag screw in cephalomedullary nails used for stabilization of proximal femoral fractures is inversely related to the
-
length of the nail.
-
length of the lag screw medial to the nail.
-
length of the lag screw lateral to the nail.
4 – length over which contact occurs between the lag screw and the device. 5- diameter of the screw.
back answer
Question 103.03
Answer = 4
As opposed to the sliding hip screw, the cephalomedullary nail has two arcs – one at either side of the wall of the intramedullary nail. Since these arcs are narrow, the available sliding surface is small. When the sliding surface is small, higher forces are required to initiate sliding. This was determined in a biomechanical paper in JBJS 1998. Thus the answer is the length over which contact occurs between the lag screw and the device –which also happens top be the longest answer Reference(s)
Loch DA, Kyle RF, Bechtold JE et al: Forces required to initiate sliding in second-generation intramedullary nails. JBJS Am 1998;80: 1626-1631.
back to this question next question
104.03 In addition to phantom limb pain and residual limb discomfort, which of the following areas is likely to be painful in patients after a lower extremity amputation for trauma?
-
Ipsilateral hip
-
Contralateral knee
-
Lower back
-
Upper extremity 5- Neck
-
back answer
Question 104.03
Answer = 3
The OKU reference states that virtually all adults experience phantom limb sensation after amputation, which is the feeling that all or part of an amputated limb is present. This usually diminishes with time. Phantom limb pain is a burning, painful sensation in the distribution of the amputated limb, reported to occur in more than 60% of adult amputees. This is treated with noninvasive therapies such as increased prosthesis use, physical therapy modalities, intermittent compression, and transcutaneous electrical nerve stimulation. If persistent, proximal nerve blocks may be performed. Residual limb pain is more localized, and related to an incompetent soft-tissue envelope, prominent underlying bony projections, or scarred deep tissue structures. In the amputee with peripheral vascular disease, ischemia of the residual limb can cause pain. Local nerve entrapment also can be a cause of residual limb pain. One report demonstrated that only 9% of major amputees were completely pain free over a 4 week period of observation. Back pain was a significant cause of discomfort in this group.
Reference(s)
Smith DG, Ehde DM, Legro MW, Reiber GE, del Aguila M, Boone DA: Phantom limb, residual limb, and back pain after lower extremity amputations. Clin Orthop 1999;361:29-38. Koval KJ (ed): Orthopaedic Knowledge Update 7. Rosemont, IL, American Academy of Orthopaedic Surgeons, 2002, pp 127-137.
back to this question next question
105.03 A 26-year-old woman has chronic toe pain after hitting a bedpost 3 months ago. A radiograph is shown in Figure 27. Her injury represents an avulsion of the
![]() |
-
lumbrical insertion.
-
extensor digitorum brevis.
-
extensor digitorum longus.
-
extensor hallucis longus.
-
flexor digitorum longus.
back answer Question 105.03
Answer = 3
The main function of the EDL is extension of the MTP joints of the lesser toes, so injury results in a claw toe deformity if left unrepaired. The EDL originates on the lateral tibial condyle, the anterior crest of the fibula, and the interosseous membrane and inserts on the base of the terminal phalanges of the four lesser toes. Innervated by the deep peroneal nerve, the EDL functions to extend the toes at the DIP joint and to dorsiflex and evert the foot. The EDL divides into two separate tendons beneath the superior retinaculum and then further divides into two lateral tendons to the fourth and fifth toes and two medial tendons to the second and third toes. The individual tendon of the EDL to each toe is joined on the lateral aspect by the tendon of the EDB. They are anchored at the level of the MTP joint by a fibroaponeurotic structure.
The EDB originates on the distal lateral and superior surface of the calcaneus and inserts on the
lateral aspect of the flexor digitorum longus tendon and also on to the base of the proximal phalanx of the first through fourth toes. There is no EDB tendon to the fifth toe. If an EDB laceration is easily identified at the time of an EDL repair, than it may be repaired as well, otherwise repair of the EDL alone is sufficient.
Reference(s)
Heckman JD: Fractures and dislocation of the foot, in Rockwood CA Jr, Green DP, Bucholz RW, Heckman JD (eds):Rockwood and Green’s Fractures in Adults, ed 3. Philadelphia, PA, JB Lippincott, 1991, pp 2166-2169.
Coughlin MJ: Disorders of tendons, in Coughlin MJ, Mann RA (eds): Surgery of the Foot and Ankle, ed 7. St Louis, MO, Mosby, 1999, pp 787-788.
back to this question next question
106.03 When compared to a 95° screw-plate construct, an intramedullary nail used to treat a reverse obliquity-type intertrochanteric femoral fracture most often results in
-
a shorter time to fracture union.
-
a lower reoperation rate at 1 year.
-
a higher perioperative mortality rate.
-
a higher perioperative transfusion requirement.
-
improved function at 1 year.
back answer
Question 106.03
Answer = 2
The first reference is the better one. JBJS 2002, a prospective randomized study out of Sweden by the Synthes people comparing cephalomedullary nails with a 95° screw plate (DHS), 20 patients in each group, followed for minimum 1 year. The nail patients had shorter OR times, less blood transfusion requirements, shorter hospital stays, and a decreased rate of implant failure, defined as nonunion or breakage. Five patients had screw cut-out from the femoral head, one had a plate breakage, and one had a nonunion with intact hardware. Only one patient with a cephalomedullary nail went on to non-union, and was treated with simple dynamization of the nail distally.
Reference(s)
Sadowski C, Lubbeke A, Saudan M, Riand N, Stern R, Hoffmeyer P: Treatment of reverse oblique and transverse intertrochanteric fractures with use of an intramedullary nail or a 95 degrees screw-plate: A prospective, randomized study. J Bone Joint Surg Am 2002;84:372-381.
Haidukewych GJ, Israel TA, Berry DJ: Reverse obliquity fractures of the intertrochanteric region of the femur. J Bone Joint Surg Am 2001;83:643-650.
back to this question next question
107.03 What is the most common congenital hand anomaly?
-
Symbrachydactyly
-
Camptodactyly
-
Syndactyly
-
Polydactyly
-
Constriction ring syndrome
-
-
back answer
Question 107.03
Answer = 3
Symbrachydactyly is characterized by fingers that fail to form normally. This condition differs from constriction ring syndrome, in which fingers have prenatal deformation. Transverse failure of formation, or terminal Reference(s) deficiency, occurs in about 1.5 in 10,000 births and 98% of cases are
unilateral. One characteristic is incomplete separation of short digits. Koval KJ (ed): Orthopaedic Knowledge Update 7. Rosemont, IL, American Academy of Orthopaedic Surgeons, 2002, pp 329-337.
Camptodactyly, is a nontraumatic flexion contracture of the proximal interphalangeal joint, usually of the small finger.
Syndactyly, or incomplete separation of digits, is the most common congenital hand malformation,m occurring in 1 in 2,500 births. It may occur as an isolated anomaly or as part of a syndrome. Up to 40% of cases are inherited in an autosomal dominant pattern.
Polydactyly, or digit duplication, is the second most common congenital hand malformation in the United States, and the thumb is the most common duplicated, or split, digit. Thumb polydactyly is most often sporadic.
Constriction ring syndrome has an incidence of approximately 1 in 15,000. Its etiology is unclear, but it is not genetic. It frequently affects both arms, or all four extremities, and is associated with clubfoot and craniofacial clefts. Digits may be amputated, constricted by a partial or complete ring ( a tight band of tissue) and/or webbed.
back to this question next question
108.03 A 36-year-old man sustained a traumatic brachial plexus injury to his dominant right upper extremity 3 years ago. Examination reveals no active deltoid or biceps function, and he is unable to bring his hand to his face. He has 1/5 elbow flexion strength, 4/5 pectoralis major strength, and 4/5 triceps strength. His trapezius and rhomboids are intact. Distally he has good hand function. What surgical procedures will best restore his ability to bring his hand to his face?
-
Glenohumeral arthrodesis and pectoralis major transfer
-
Glenohumeral arthrodesis and triceps transfer
-
Glenohumeral arthrodesis and Steindler flexorplasty (proximal flexor pronator transfer)
-
Scapulothoracic arthrodesis and pectoralis major transfer
-
Scapulothoracic arthrodesis and Steindler flexorplasty ( proximal flexor pronator transfer)
back answer
Question 108.03
Answer = 1
This patient’s problem is the inability to “bring his hand to his face,” which greatly interferes with his capacity to perform activities of daily living. The references suck for this one. We are faced with two basic questions – type of fusion, and type of transfer. Question one – scapulothoracic versus glenohumeral fusion – the question tells us traps and rhomboids are intact, these motor the scapula while the deltoid is out, which motors the humerus, so cross out the last two options. The second reference is an operative technique for glenohumeral arthrodesis with a pelvic recon plate. Question two – type of elbow flexorplasty (pec major transfer, triceps transfer, or Steindler flexorplasty). A Steindler flexorplasty involves transposition of the common flexor origin to a point more proximal on the humerus. The pectoralis major transfer involves suturing this tendon to a portion of the distal biceps tendon reflected back proximally. The second reference on glenohumeral arthrodesis briefly refers to all three transfers without hinting at a preference, and the first reference compares pec transfer to Steindler and concludes the functional outcomes to be equivalent: no difference in range of motion testing. It does reference the pec transfer to be marginally stronger in all positions except supination, which was stronger in the Steindler group. They reference a work that suggests the pec transfer “should have mechanical advantages over the Steindler.” They state: “We have no evidence to suggest one method is superior to the other. Accordingly, we use the pectoralis major tendon transfer (when pectoralis major is present) in preference to the Steindler procedure when flexorplasty and shoulder arthrodesis are performed. The Steindler procedure is reserved for those patients without pectoralis major function or who do not require shoulder arthrodesis.”
Reference(s)
Beaton DE, Dumont A, Mackay MB, Richards RR: Steindler and pectoralis major flexorplasty: A comparative analysis. J Hand Surg Am 1995;20:747-756.
Richards RR, Sherman RM, Hudson AR, Waddell JP: Shoulder arthrodesis using a pelvic-reconstruction plate: A report of eleven cases. J Bone Joint Surg Am 1988;70:416-421.
back to this question next question
109.03 An otherwise healthy 75-year-old woman who lives alone sustains the fracture shown in Figures 28a and 28b. Management should consist of
-
sling immobilization until comfortable, followed by functional rehabilitation.
-
open reduction and internal fixation using a blade plate.
-
hemiarthroplasty.
-
intramedullary nailing. 5- total shoulder arthroplasty.
![]() |
back answer
Question 109.03
Answer = 3
This is a three- or four-part fracture of the proximal humerus in an elderly patient. We can’t tell exactly without a CT, but it doesn’t really matter either way. We can start by crossing out the obvious wrong answers: no need to go straight to TSA when the glenoid is relatively normal. We all know that the testmakers hate intramedullary humeral nails unless it’s a pathologic fracture. So non-op, ORIF, or hemi. The second reference is a review from the cream journal that looks at the management of 3- and 4-part humerus fractures. It refers to Neer’s criteria to define fracture “parts”
with displacement greater than 1cm and angulation greater than 45°. In the case of displaced 3- and 4-part fractures, the physiologic age and bone quality also help guide treatment selection. In young patients with good bone quality, attempts to preserve the humeral head by meticulous handling of soft tissues and the use of low-profile implants to secure fracture fragments is recommended. Elderly patients and those with poor bone quality have a greater risk of loss of reduction after open reduction and internal fixation, and the current consensus is that early hemiarthroplasty is the appropriate treatment. Late reconstruction necessitated by malunion and soft tissue contracture is technically difficult, and the outcome is less favorable. The first reference is a review of primary and secondary hemiarthroplasty for proximal humerus fractures, and their conclusions are that early hemi after fracture has better outcomes than late hemi after attempted non-op management.
Reference(s)
Bosch U, Skutek M, Fremerey RW, Tscherne H: Outcome after primary and secondary hemiarthroplasty in elderly patients with fractures of the proximal humerus. J Shoulder Elbow Surg 1998;7:479-484.
Naranja RJ Jr, Iannotti JP: Displaced three- and four-part proximal humerus fractures: Evaluation and management. J Am Acad Orthop Surg 2000;8:373-382.
back to this question next question
110.03 A 34-year-old man struck by a car sustains the closed injury shown in Figure 29. The patient is awake and alert, has no dorsiflexion of the foot or toes, and has decreased sensation on the dorsum of the right foot. Leg compartments are soft. Compartment pressure measurements are 28 mm Hg, 32 mm Hg, 18 mm Hg, and 20 mm HG in the anterior, lateral, superficial posterior, and deep posterior compartments, respectively. The blood pressure is 120/70 mm Hg. The anklebrachial index measures 0.95. After appropriate splinting, the next step in management should consist of
-
fasciotomies of the anterior and lateral compartments without peroneal nerve exploration.
-
fasciotomies of all four compartments without peroneal nerve exploration.
-
peroneal nerve exploration.
-
lower extremity angiography.
-
continued observation in anticipation of stabilization procedures.
back answer
Question 110.03 Answer = 5
Here we have a bad proximal tibia fracture with questionable extension into the articular surface of the plateau. This is a high-energy injury, and the patient already has motor/sensory disturbances. Options now are nerve exploration, angio, fasciotomy, or observation. We simply don’t do acute nerve explorations unless we are already there with an open fracture or penetrating trauma and the nerve is already exposed through that or a fasciotomy site. The first reference, from JOT, evaluates non-invasive vascular tests in excluding occult arterial trauma. They used Doppler arterial pressure index (API) of <0.90 as an indication for angio in 100 injured limbs, and in the 83 limbs with an API >0.90, there were no major injuries, and only 5 minor lesions (four pseudoaneurysms, and one arteriovenous fistula). Concluding an API >0.90 as a safe, accurate, and cost effective screening method. Here it is 0.95. Court-Brown’s classic article is referenced for compartment syndrome evaluation, a prospective study of 116 patients with tibial diaphyseal fractures who had continuous monitoring of anterior compartment pressure for 24 hours. They used a threshhold of 30mmHg differential pressure as an indication for fasciotomy, ( differential meaning diastolic pressure minus compartment pressure ) performing only 3 in this study. A simple threshhold of 30mmHg compartment pressure would have led to 50 fasciotomies (43% versus 2.6%). The outcomes led to no missed compartment syndromes and there were no adverse sequela to observing elevated compartment pressures that didn’t meet this criteria. In our question, the differential pressures are 42, 38, 52, and 50 – all safe within the observation categories of Court-Brown.
Reference(s)
Johansen K, Lynch K, Paun M, Copass M: Non-invasive vascular tests reliably exclude occult arterial trauma in injured extremities. J Trauma 1991;31:515-519.
McQueen MM, Court-Brown CM: Compartment monitoring in tibial fractures: The pressure threshold for decompression. J Bone Joint Surg Br 1996;78:99-104.
Koval KJ (ed): Orthopaedic Knowledge Update 7. Rosemont, IL, American Academy of Orthopaedic Surgeons, 2002, pp 479-488.
back to this question next question
111.03 A 30-year-old woman has isolated knee pain and swelling exacerbated by activities. A radiograph and MRI scans are shown in Figures 30a through 30c. A biopsy
![]() |
![]() |
![]() |
![]() |
![]() |
![]() |
![]() |
specimen is shown in Figure 30d. What is the most likely diagnosis?
-
Synovial sarcoma
-
Synovial chondromatosis
-
Rheumatoid arthritis
-
Hemophilia
-
Pigmented villonodular synovitis
![]() |
back
Question 111.03
Answer = 5
answer
Radiographs are negative for any bony changes here, and the MRI has the classic lesion of PVNS – dark on T1 and dark on T2. PVNS most commonly occurs in the knee joint as a monoarticular proliferative process involving the synovial membrane, and affecting adults in the third and fourth decades. There is a slow progression of symptoms, and it tends to be episodic in nature with complaints of discomfort, warmth, swelling, stiffness, and occasionlly a palpable mass. Findings of multiple joint involvement and instability are rare. Pathologic findings consist of nodular proliferation of large, round, polyhedral, or spindle cells with prominent cytoplasm and pale nuclei with hemosiderin laden cells in the center. This is classic PVNS as presented here. Chondromatosis always has a picture of the multiple calcified nodules as a path sample. RA would probably show us blood studies and some sort of articular changes on XR. Hemophilia would also present with bony changes and bleeding times, usually in an adolescent patient. Sarcoma is more rare than PVNS, and the micro slide would have to portray a more malignant picture?
Reference(s)
Flandry F, Hughston JC: Pigmented villonodular synovitis. J Bone Joint Surg Am 1987;69:942-949. Dorfman HD, Czerniak B: Bone Tumors. St Louis, MO, Mosby, 1998, pp 1041-1086.
back to this question next question
112.03 A 28-year-old man who sustained the injury shown in Figure 31 is hemodynamically unstable. In addition to fluid resuscitation, the next most appropriate step in management should include
-
angiography and embolization.
-
an emergent exploratory laparotomy.
-
external pelvic stabilization.
-
open reduction and internal fixation.
-
closed reduction and percutaneous screw fixation.
![]() |
![]() |
![]() |
backanswer Question 112.03
Answer = 3
The radiograph shows us a pelvic inlet view of an APC II or III pelvic ring injury with syndesmotic disruption greater than 5cm, and a right SI joint disruption as well. APC (and some VS) injuries are associated with increases in pelvic volume allowing occult blood loss. All of the responses are viable options, but on a spectrum of timing. ORIF and CRPSF are both more elective options that require appropriate pre-operative planning and a stable patient, not to be undertaken in a hemodynamically unstable patient. The article referenced from JBJS 2002 reviewed 150 patients with pelvis fractures and the use of angiography for the management of “haemorrhage.” They concluded/recommended skeletal stabilization as the first line of treatment, followed by possible laparotomy and packing of pelvic retroperitoneum as a second line of treatment, with pelvic angiography and embolization only in those patients that were unresponsive to both previous interventions. They based their recommendations also on anatomical studies that suggested that the surfaces of the fracture and veins, rather than arteries, were the major sources of bleeding in these patients. External pelvic stabilization in their study included external fixators, pneumatic anti-shock garments, and pelvic clamps.
Reference(s)
Cook RE, Keating JF, Gillespie I: The role of angiography in the management of haemorrhage from major fractures of the pelvis. J Bone Joint Surg Br 2002;84:178-182.
Koval KJ (ed): Orthopaedic Knowledge Update 7. Rosemont, IL, American Academy of Orthopaedic Surgeons, 2002, pp395-405.
back to this question next question
113.03 A 68-year-old man who has severe knee pain is no longer able to carry out routine activities despite the use of anti-inflammatory drugs and intra-articular cortisone injections. Radiographs are shown in Figures 32a through 32c. Treatment should now consist of
1- total knee arthroplasty with lateral soft tissue release.
![]() |
-
rotating hinge knee arthroplasty.
-
simultaneous distal femoral osteotomy and total knee arthroplasty.
-
combined distal femoral and proximal tibial osteotomy.
-
distal femoral osteotomy.
back
Question 113.03
answer
Answer = 3
Radiographs show us long-leg AP, and AP/Lat of a left knee with valgus-extension malunion of a distal femur fracture resulting in severe DJD of the lateral compartment of the knee. Both the femoral malunion, and the lateral compartment disease need to be addressed. Neither answers 1 and 2, ( which are isolated TKA options) nor answer 5 (isolated osteotomy) address both problems. Answer 4 recommends combined femoral and tibial osteotomies, which is unneccesary with a normal tibia. So the main question is do the two procedures have to be staged, and which one should be performed first? The reference is a retrospective review of simultaneous corrective osteotomy and TKA out of JBJS 2000, in which they evaluated 11 knees with 46 months of average follow-up, and concluded that “simultaneous femoral osteotomy and total knee arthroplasty
is a technically difficult but effective treatment for patients with severe femoral deformity associated with ipsilateral osteoarthritis of the knee. (They recommended) that the femoral osteotomy site be secured with a plate or a locked intramedullary nail, depending on the location of the deformity and the subsequent osteotomy.”
Reference(s)
Lonner JH, Siliski JM, Lotke PA: Simultaneous femoral osteotomy and total knee arthroplasty for treatment of osteoarthritis associated with severe extra-articular deformity. J Bone Joint Surg Am 2000;82:342-348.
back to this question next question
114.03 Figure 33 shows the radiograph of a 48-year-old man who sustained a fracturedislocation of his dominant arm and a significant head injury in a fall from a roof. Eight days after injury he is medically cleared for surgery. Treatment should consist of
![]() |
![]() |
![]() |
![]() |
-
rotator cuff repair.
-
open reduction.
-
shoulder arthrodesis 4- total shoulder arthroplasty. 5- humeral head arthroplasty.
back answer Question 114.03
Answer = 5
As stated, the radiograph shows a multiple-part fracture dislocation of the
proximal humerus, with what appears to be a head-splitting component. Answer 1
(rotator cuff repair) is certainly not indicated yet – maybe as a secondary procedure
once the bone is healed, or as a subsequent procedure while dealing with the fracture.
Similar to question #109, there is no need to resurface the glenoid unless we have
evidence of significant degenerative changes, which we don’t. ORIF of head-splitting
fractures hasn’t had any promising results, and AVN is always an issue here. Both
articles referenced discuss the outcomes of acute hemiarthroplasty in the
treatment of 3- and 4-part proximal humerus fractures versus late replacement for
failed non-operative treatment, and the benefits of early management in regards
to difficulty of procedure, post-operative pain, and functional ROM/strength outcomes. Arthrodesis is a viable option, but not with the good results of hemiarthroplasty as evidenced here.
Reference(s)
Goldman RT, Koval KJ, Cumom F, Gallagher MA, Zuckerman JD: Functional outcome after humeral head replacement for acute three- and four-part proximal humeral fractures. J Shoulder Elbow Surg 1995;4:81-86.
Norris TR, Green A, McGuigan FX: Late prosthetic shoulder arthroplasty for displaced proximal humerus fracture. J Shoulder Elbow Surg 1995;4:271-280.
back to this question next question
115.03 A 10-year-old boy sustains a 100% displaced closed transverse midshaft femoral fracture. Neurovascular examination is intact. Initial radiographs show a 5-cm overriding of the fragments with no comminution. Management should consist of
-
closed reduction with immediate hip spica casting.
-
skin traction, followed by hip spica casting in 3 weeks.
-
a rigid reamed intramedullary antegrade nail, entering at the piriformis fossa.
-
a rigid unreamed retrograde nail.
-
two flexible intramedullary nails.
-
back answer
Question 115.03
Answer = 5
Morrissy’s article out of Atlanta’s Scottish Rite Hospital in JBJS 1999 reviewed 85 fractures in 81 patients age 6 to 16 treated by early spica casting, traction followed by spica casting, flexible intramedullary nailing, external fixation, compression plating, or reamed intramedullary rod. He stated that early spica casting resulted in satisfactory outcome in patients younger than 6 with minimal cost to treatment, but that in older patients it resulted in significant limb-length discrepancy. Similarly, antegrade reamed rodding is the standard of choice in patients older than 16 with closed physes. The study evaluated outcomes with regard to clinical, complications, and cost.
“Indications from this report are that early spica casting is the best method when feasible ; flexible IM rods result in earlier healing with fewer complications than external fixation and are best in simple transverse and short oblique fractures; external fixation is best for long spiral and comminuted fractures, and reamed IM rodding should be reserved for those at or near skeletal maturity.”
The costs of traction followed by spica casting were equivalent to acute surgical treatment due to cost of extended hospital stay. Reference(s)
Heinrich SD, Drvaric DM, Darr K, MacEwen GD: The operative stabilization of pediatric diaphyseal femur fractures with flexible intramedullary nails: A prospective analysis. J Pediatr Orthop 1994;14:501-507.
Stans AA, Morrissy RT, Renwick SE: Femoral shaft fracture treatment in patients age 6 to 16 years. J Pediatr Orthop 1999;19:222-228.
back to this question next question
116.03 A cognitively intact 83 year-old woman who lives with her daughter sustains a proximal humeral fracture. Which of the following observations is most likely to lead to suspicion of elder abuse?
-
Contusion on the patient’s forehead
-
Disparity in histories from the patient and her daughter
-
History of a femoral neck fracture
-
Spiral fracture pattern
-
Ecchymosis extending down the arm to the elbow
back answer
Question 116.03
Answer = 2
Reference(s) Chen AL, Koval KJ: Elder abuse: the role of the orthopaedic surgeon in diagnosis and management. J Am Acad Orthop Surg 2002; 10:25-31.
back to this question Go to explanation
![]() |
![]() |
![]() |
![]() |
![]() |
![]() |
![]() |
![]() |
![]() |
![]() |
![]() |
![]() |
![]() |
![]() |
![]() |
![]() |
![]() |
116.03 Explanation – JAAOS review of elder abuse. Main take home points are out of
the tables
back
next question
117.03 What is the predominant cell type found in fracture callus just prior to calcification of the chondroid callus?
-
Chondroclasts
-
Osteoblasts
-
Osteocytes
-
Hypertrophic chondrocytes
-
Proliferative chondrocytes
back answer
Question 117.03
Answer = 4 Reference(s)
Buckwalter JA, Einhorn TA, Simon SR (eds): Orthopaedic Basic Science: Biology and Biomechanics of the Musculoskeletal System, ed. 2 Rosemont, IL, AAOS,
2000 , pp 371-399
back to this question go to explanation
117.03 Explanation – 6 stages of fracture healing out of the AAOS basic science textbook (no pictures). However, if it helps, review endochondral ossification because this very similar to how fracture callus turns into bone.
Bone healing by a combination of both intramembranous and endochondral ossification but the question deals with the endochondral side of bone healing
Stage 1 – hematoma and inflammation
Stage 2 – angiogenesis and cartilage formation (proliferative then hypertrophic chondrocytes) Stage 3 – cartilage calcification (hypertrophic chondrocytes)
Stage 4 – cartilage removal (chondroclasts and osteoclasts) Stage 5 – bone formation (osteoblasts)
Stage 6 – bone remodeling (osteoclasts and osteoblasts)
So before calcification of chondroid matrix by activity of calcium regulating matrix vesicles, the chondrocytes become hypertrophic. Once calcified, chondroclasts and osteoclasts can resabsorb the calcified tissue followed by osteoblasts laying down new bone.
back next question
118.03 A 17 year old boy sustains a wound to his proximal forearm from a handgun. Radiographs reveal a radiopaque projectile in the soft tissues of the midforearm but no fractures. Examination reveals that he has no wrist or finger extension, and he reports minimal pain with passive finger motion. Initial management should consist of
-
surgical debridement of the wound and removal of the bullet
-
surgical debridement of the wound and exploration of the radial nerve in the proximal forearm.
-
local wound care and exploration of the radial nerve in 2 to 3 weeks if there are no signs of infection
-
local wound care and immediate electromyography 5- local wound care and delayed electromyography
back answer
Question 118.03
Answer = 5 Reference(s)
Hand Clinics 1999; 15: 233-244
Journal of Neurosurgery 1989; 70: 166-174
The reason to delay the EMG is because initially, nerve conduction will be essentially normal. The delay is able to tell how much damage is present due to the loss of signal.
back to this question go to explanation
118.03 Explanation -
High velocity wound
Peripheral nerve injury secondary to missile Compartment Syndrome
Vascular repair required
Progressive Neuropathy Fracture ORIF required
Early operative treatment Nerve exploration
Early operative treatment not required
Serial clinical assessment Serial EMG/NCV Occupational Therapy
continuity
Nerve disrupted Nerve in
Nerve ends sutured to
adjacent soft tissue
Follow up 3 to 6 months
Secondary repair or nerve
graft at 2 – 3 weeks
No regeneration
Surgical Exploration Conservative NAP recording management
Regeneration
(+)NAP (-)NAP
back next question Low velocity wound with min soft
tissue injury
Neurolysis Resection, repair, or graft
119.03 Figure 34 shows the radiograph of a 77 year old woman who underwent total hip arthroplasty 10 years ago. What is the predominant cause of the proximal femoral bone loss?
-
Stress shielding
-
Polyethylene debris-induced osteolysis
-
Senile osteoporosis
-
Modulus of elasticity of the femoral stem
-
Diffuse osteopenia
back answer
Question 119.03
Answer = 1 Reference(s)
Clin Ortho 1999; 369:230-242
Clin Ortho 1988; 231:7-28
back to this question go to explanation
119.03 Explanation – When looking at the Xray, you can see that there is decrease in the cortical thickness around the extensively porous coated stem. The CORR references talk about long term 10 year results regarding extensively coated stems. There is mention of stress shielding as a concern but does not go into a description of what stress shielding does to bone. Remember, according to Wolff’s law, bone stresses cause a change in the activity in bone remodeling such that increase in stress leads to increase in bone deposition along the lines of stress. Now, with extensively coated stems where fixation is diaphyseal, this subsequently protects or shields the more proximal portion of the bone. This leads to thinning or decreased density of the bone as seen in the xray.
back next question
120.03 The retrocalcaneal bursa is located between the
-
skin and Achilles tendon
-
skin and superior calcaneal tuberosity
-
posterior calcaneal tuberosity and the Achilles tendon
-
posterior and superior calcaneal tuberosities
-
Achilles tendon and the superior calcaneal tuberosity
back answer
Question 120.03
Answer = 5 Reference(s)
OKU: Foot and Ankle 2; pp 175-183
Mann’s – Surgery of the Foot and Ankle (1999); pp 826-834
back to this question go to explanation
120.03 Explanation – This picture is right out of the book. Remember, Haglund’s deformity which is an enlargement of the posterior superior calcaneal tuberosity can cause posterior heel pain and inflame the retrocalcaneal bursa.
back next question
121.03 A 52 year old woman has had a painful planovalgus foot deformity for the past several years. Immobilization, shoe modification, and an orthotic have failed to provide relief. Examination reveals that she is unable to perform a single stance heel rise. Hindfoot valgus and forefoot supination are not manually correctable. Treatment should consist of
-
Achilles tendon lengthening and triple arthrodesis
-
posterior tibial tendon debridement and flexor digitorum longus transfer
-
posterior tibial tendon reconstruction and medial displacement calcaneal osteotomy’ 4- posterior tibial tendon reconstruction and lateral column lengthening
-
-
subtalar arthrodesis
back answer
Question 121.03
Answer = 1 Reference(s)
Mann’s- Surgery of the Foot and Ankle (1999); pp 733-767 Clin Orthop 1999; 365: 91-99
back to this question go to explanation
121.03 Explanation – This paper is written by a Foot and Ankle/Oncology Orthopedist in Tampa, FL. I worked with him briefly when I was medical student. Anyway, this paper from CORR is a good review of the indications, techniques and outcomes of triple arthrodesis for rigid planovalgus. In general when dealing with foot deformities, a rigid passively uncorrectable deformity needs some type of osteotomy or bony fusion to correct the deformity. If the deformity is not rigid then soft tissue procedures can usually do the trick. So, knowing that this patient has a rigid flatfoot, which of the choices deals with some type of bony procedure and does not rely on a soft tissue procedure to restore a plantigrade foot? 1 and 5. 2,3,4 deal with the posterior tibialis tendon which will not help rigid flatfoot. Then, choice 5 does not address the midfoot deformity and chopart’s joint. The indications for a triple arthrodesis are rigid flatfoot deformity. In addition, you also need to address the achilles pull contributing to the valgus deformity. This is done by achilles lengthening.
back next question
122.03 Figures 35a and 35b show the radiographs of a 70 year old patient who has severe left shoulder pain. What is the most likely diagnosis?
-
Chondrosarcoma
-
Tumoral Calcinosis
-
Pigmented villonodular synovitis
-
Synovial Chondromatosis
5- Moyositis Ossificans
![]() |
![]() |
![]() |
![]() |
back
Question 122.03
Answer = 4 Reference(s)
Milgram JW: Synovial Osteochondromatosis: a histopathological study of thirty cases. JBJS Am 1977; 59:792-801.
answer
back to this question go to explanation
122.03 Explanation – Looking at the xrays you can see that there is opacities all over the proximal humerus. The key is to recognize that they involve the entire synovium. More commonly we see synovial osteochondromatosis as loose bodies within the joint. This paper in JBJS is very informitive about the three recognizable phases of synovial osteochondromatosis. Phase one – active intrasynovial disease with no loose bodies. Phase two – transitional lesions with both active intrasynovial proliferation and free loose bodies. Phase three – multiple free osteochondral bodies with no demonstrable intrasynovial disease.
back next question
123.03 An 11 month old infant has arthrogryposis multiplex congenita. Examination reveals flexion, abduction, and external rotation hip contractures. Radiographs reveal that both hips are dislocated. Management of the dislocations should consist of
-
a Pavlik harness
-
adduction traction
-
closed reduction under general anesthesia, adductor longus tenotomy, and a spica cast
-
open reduction 5- femoral head resection
back answer
Question 123.03
Answer = 4 Reference(s)
Journal of Pediatric Orthopaedics 2002;22:359-363 Journal of Pediatric Orthopaedics 1987; 7:681-685
back to this question go to explanation
123.03 Explanation – For those of you who do not know what arthrogryposis is here you go. Arthrogryposis Multiplex Congenita is a disease characterized by congenital contracture of joints in multiple areas. The most common form is amyoplasia where there is symmetric involvement of the joints in both upper and lower limbs. The incidence of hip involvement is about 65-80%. These include contracture, subluxation and dislocation. The reference from 2000 is from Hong Kong and is a 20 year follow up for hip problems in arthrogryposis. Overall the treatment of choice in dislocated hips is open reduction – the earlier the better. Closed reduction rarely works in arthrogryposis. The long term sequelae was stiffer hips but overall function was comparable with other hips not needing surgery.
back next question
124.03 A 22 year old man sustains a proximal tibial fracture at the junction of the metaphysis and the diaphysis. Which of the following fracture site deformities are most commonly associated with intramedullary nailing using a starting point well centered on the AP view?
-
Posterior angulation and varus
-
Posterior angulation and valgus
-
Posterior angualtion and internal rotation
-
Anterior angulation and valgus
-
Anterior angulation and varus
back answer
Question 124.03
Answer = 4 Reference(s)
Clin Orthop 1995; 315:25-33
Clin Orthop 1995; 315:64-74 OKU 7 pp 479-488.
back to this question go to explanation
124.03 Explanation – Most of you probably do not have a problem with this question. My problem is semantics and terminology. Proximal third tibia fractures treated with intramedullary nailing have deforming forces that cause apex anterior and valgus deformity. This is due to the pull of the posterior compartment muscles and lateral compartment muscles. My problem is that the angle is directed posteriorly and the question is not specific about apex or bowing. In this case, the tibia would have an anterior bow or apex anteriorly. This was somewhat confusing to me because of the terminology. Think of distal radius fractures when we say dorsally angulated. We should more accurately say dorsally displaced with apex volarly. Nonetheless, the answer to proximal tibia fractures and subsequent deformity is apex anteriorly and valgus.
back next question
125.03 A 35 year old man sustains a dislocation of his dominant shoulder in a fall. The shoulder is reduced and placed in a sling, but returns 6 hours later with shoulder dislocated again, despite use of a sling. A CT scan is shown in figure 36. Management should now consist of
-
replacement of the sling with a modified spica cast
-
open reduction and internal fixation
-
percutaneous pin fixation
-
arthroscopic labral repair
-
arthroscopy and removal of the loose fragment
![]() |
![]() |
![]() |
![]() |
back answer Question 125.03
Answer = 2 Reference(s)
AM J Sports Med 1998;26:41-45 JBJS Am 1993;75:479-484
back to this question go to explanation
125.03 Explanation – Biggerstaff is the expert on this subject. The CT scan shows the glenohumeral joint with a fracture involving the glenoid. This patient, much like Bigg’s, has an unstable glenohumeral and is not going to be stable until it is fixed. Casting won’t help. Arthroscopy would help if this was a labral injury or loose fragment. This is indeed a fracture through the glenoid. Percutaneous pinning might sound reasonable but difficult to gain a reduction. The main reference looks at open reduction and internal fixation of scapular glenoid fractures and their outcomes. They used a posterior approach. They admit in the first paragraph that which fractures and what approach to use is debatable. Furthermore, there is no description of what percent involvement of the glenoid. All fractures were displace 4 mm or more and their outcomes were deemed excellent in regard to shoulder function.
back next question
126.03 What is the major advantage of allowing early active motion of a repaired zone II flexor tendon injury?
-
Increased tendon excursion
-
Greater repair strength
-
Less postoperative pain
-
Better patient compliance
-
Faster tendon healing
back answer
Question 126.03
Answer = 1 Reference(s)
J Hand Surg Am 1991;16:669-680 J Hand Surg Am 1992;17:559-566
back to this question go to explanation
126.03 Explanation – This paper in the journal of hand surgery talks about various mobilization methods for zone II flexor tendon injuries. The purpose of early mobilization is to help tendon excursion by preventing peritendonous adhesions. Let’s break down the answer choices:
-
correct – more excursion
-
greater repair strength is dependent on the amount of strands that cross the repair not early mobilization
-
less postoperative pain: well would it hurt more to move your recently cut on finger or would it hurt more to keep it still. Give it a try!!
-
Let’s see; move you finger as much as you can and oh by the way, it is really going to hurt! Early motion does not help patient compliance
-
Faster healing: How many times have you heard I am a fast healer? The references do not mention this. But they do mention that early mobilization did not have a significant effect on wound dehiscence. A3, A4 level injuries have such a small amount of excursion that it may not be possible to prevent adhesions at these levels.
back next question
127.03 Item deleted !!
back next question
128.03
![]() |
![]() |
![]() |
![]() |
A 22 year old laborer sustains the injury shown in Figures 37a and 37b. Figures 37c and 37d show the postoperative radiographs obtained after application of a short leg cast.
Management should now consist of
1- revision of the fibular fixation. 2- Repair of the deltoid ligament.
-
Arthroscopy and retrieval of the osteochondral fragment.
-
- Open reduction and internal fixation of the medial malleolus fracture.
-
Reduction and fixation of the syndesmosis.
![]() |
![]() |
![]() |
answer back
Question 128.03
Answer = Reduction and fixation of the syndesmosis Reference(s)
Koval KJ (ed): Orthopaedic Knowledge Update 7. Rosemont, IL, American Academy of Orthopaedic surgeons, 2002, pp 547-564.
Carr JB , Trafton PG: Malleolar fractures and soft tissue injuries of the ankle, in Browner BD, Jupiter JB, Levine
AM, Trafton PG (eds): Skeletal Trauma, ed 2. Philadelphia, PA, WB Saunders, 1998 pp 2327-2404 back to this question next question 0129.03
![]() |
A twelve year old boy has pain in the medial arch of his left foot with weightbearing activities. Nonsurgical management has failed to provide relief. Radiographs show an Ogden type II accessory by navicular (accessory ossicle joined to the prominence of the navicular by a synchondrosis). Treatment should consist of
-
arthodesis of the ossicle to the navicular
-
excision of the ossicle and the navicular prominence
-
talonavicular arthrodesis, with elevation of the medial arch
-
calcaneal neck lengthening by opening wedge osteotomy and bone graft
-
subtalar joint arthrodeis
back answer
Question 129.03
Answer = 2- excision of the ossicle and the navicular prominence
Reference(s) Mosca VS: The foot, in Morrisy RT, Weinstein SL (eds): Lovell and Winter’s Pediatric Orthopaedics, ed 5. Philadelphia, PA, Lippncott Williams and Wilkins, 2000, pp 1151-1215.
Herring JA: Disorders of the foot, in Herring JA (ed): Tachdjian’s Pediatric Orthopaedics, ed 3. Philadelphia, PA, WB Saunders, 2002. pp 891-11037
back to this question next question
130.03
A patient underwent shoulder arthroplasty 8 months ago. Prior to surgery, he was able to elevate his arm 90°. Postoperatively he is unable to elevate his arm and has a significant cosmetic deformity. A postoperative radiograph is shown in figure 38. What is the most likely explanation for his inability to elevate the arm?
![]() |
![]() |
-
The coracoacromial arch is disrupted.
-
The humeral component is too large.
-
The supraspinatus muscle was not repaired.
-
The coracohumeral ligament was divided. 5- The glenoid was resurfaced.
back answer
Question 130.03
Answer = 1- The coracoacromial arch is disrupted
Reference(s)
Lee TQ, Black AD, Tibone JE, McMahon PJ: Release of the coracoacromial ligament can lead to glenohumeral laxity: A biomechanical study. J Shoulder Elbow Surgery 2001: 10: 68-72.
Bigliani LU, Levine WN: Subacromial impingement syndrome. J Bone Joint Surg Am 1997: 79:1854-1868
back to this question next question
131.03 A 64 year old man who underwent revision total knee arthroplasty 6 months ago has leg pain after walking. AP and lateral radiographs are
![]() |
![]() |
![]() |
![]() |
![]() |
![]() |
![]() |
![]() |
shown in Figures 39a and 39b. The cause of pain is most likely related to
1- prior patellecomy 2- tibial stem position.
3- limb malalignment
4- the posterior stabilized component 5- the notched femoral component
answer back Question 131.03
Answer = 2- tibial stem position Reference(s)
Barrack RL, Rorabeck C, Burt M. Sawhney J: Pain at the end of the stem after revision total knee arthroplasty. Clin Orthop 1999:367:216225.
back to this question next question
132.03 Persistent chronic draining osteomyelitis of the tibia is most likely to develop into what type of malignancy?
-
multiple myeloma
-
osteosarcoma
-
chondrosarcoma
-
melanoma
-
squamous cell sarcoma
back answer
Question 132.03
Answer = 5- squamous cell sarcoma
Reference(s)
McGory JE, Pritchard DJ, Unni KK, Ilstrup D, Rowland CM: Malignant lesions arising in chronic osteomyelitis. Clin Orthop 1999;362:181-189. Saglik Y, Arikan M, Altay M, Yildiz Y: Squamous cell carcinoma arising in chronic osteomyelitis. Int Ortho 2001;25:389-391
back to this question next question
133.03 A 45y/o male recently underwent a transfemoral amputation as a result of a traumatic even. The patient desires to remain very active and can walk at a very fast pace. What type of prosthetic knee joint is the most appropriate?
-
hydraulic
-
pneumatic
-
constant friction
-
polycentric
-
manual locking
back answer
Question 133.03 Answer = 1- hydraulic
Reference(s)
Koval KJ (ed): Orthopaedic Knowledge Update 7. Rosemont, IL, American Academy of Orthopaedic Surgeons, 2002, pp133-135.
back to this question next question
134.03 Figure 40 shows the radiograph of a patient who has arm pain. What is the best initial course of action?
![]() |
![]() |
![]() |
![]() |
-
MRI
-
Skeletal survey
-
Open biopsy
-
Immobilization
-
Methylprednisolone injection
back answer
Question 134.03
Answer = 4- Immobilization
Reference(s)
Capanna R, Dal Monte A, Gitelis S, Campanacci M: The natural history of unicameral bone cyst after steroid injection. Clin Orthop 1982;166:204-211.
Scaglietti O, Marchetti PG, Bartolozzi P: Final results obtained in the treatment of bone cysts with methylprednisolone acetate (depomedrol) and a discussion of results achieved in other bone lesions. Clin Orthop 1982;165:33-42.
back to this question next question
135.03 Which of the following cell types is the key initiator of the events associated with periprosthetic osteolysis?
-
osteoblast
-
osteocyte
-
osteoclast
-
neutrophil
-
macrophage
back answer
Question 135.03
Answer = 5- Macrophage
Reference(s)
Buckwalter JA, Einhorn TA, Simon SR (eds): Orthopaedic Basic
Science: Biology and Biomechanics of the Musculoskeletal System ed 2. Rosemont IL, American Academy of Orthopaedic Surgeons, 2000 pp401-426.
Jacobs JJ, Roebuck KA, Archibeck M, Hallab NJ, Giant TT: Osteolysis: Basic science. Clin Orthop 2000;393:71-77.
back to this question next question
![]() |
![]() |
![]() |
![]() |
136.03 Figures 41a and 41b show the radiographs of a 28 year old man who sustained a twisting injury to his lower extremity while playing soccer.
![]() |
Closed reduction under anesthesia is unsuccessful. What structure is most likely blocking reduction?
-
Os trigonum
-
Anterior tibial tendon
-
Posterior tibial tendon
-
Extensor digitorum longus tendon 5- Extensor digitorum brevis muscle
backanswer
Question 136.03
Answer = 5- Extensor digitorum brevis muscle
Reference(s)
Koval KJ (ed): Orthopaedic Knowledge Update. Rosemont, IL, American Academy of Orthopaedic Surgeons, 2002, pp547-564.
Ciraulo DL, Cowell V, Jacobs LM: Evaluation and treatment of the multiple-injured patient, in Browner BD, Jupiter JB, Levine AM, Trafton PG (eds): Skeletal Trauma, ed . Philadelphia, PA, WB Saunders, 1998, pp131-149.
back to this question next question
137.03
-
- Neurilemmoma of the popliteal nerve
-
- Superficial peroneal nerve entrapment
-
- Popliteal artery aneurysm
-
- Popliteal artery entrapment syndrome
-
- Exercise-induced transient compartment syndrome
-
Preferred Response –
137.03 4.
back to this question next question
Popliteal artery entrapment syndrome (PAES) is a rare cause of calf pain in the young athletic individual, most commonly seen by sports medicine specialists. The condition closely mimics exercise-induced compartment syndrome, but the treatment is completely different. Four types of PAES have been described:
![]() |
![]() |
Type I – popliteal artery runs medial to the medial head of the gastrocnemius. Normal medial gastroc insertion.
Type II – popliteal artery is entrapped by the medial gastroc, which abnormally inserts on the femoral metaphysic, superior and lateral to its normal attachment.
![]() |
Type III – accessory band of the head of the medial gastroc entraps artery
![]() |
Type IV – The popliteal artery loops medial to the medial gastroc and beneath the popliteus muscle, which compresses the artery.
The classification is mostly of academic, not practical, importance. The condition is bilateral in up to 67% of patients. Tingling of toes may be reported but is not always present. The main complaint is cramping pain in the affected calf with exercise. Pulses may be diminished or absent, particularly with the knee hyperextended and the foot dorsiflexed. Knee is sometimes warm to touch due to increased collateral circulation, and the foot may be cool.
Complications include thrombosis of the artery due to intimal damage and/or aneurysm, and embolization with subsequent gangrene. Compartment pressures in PAES may be reduced after exercise, whereas they are elevated in exertional compartment syndrome.
When damage to the artery has already occurred, appropriate vascular procedures are needed. Otherwise, the musculotendenous structure compressing the vessel should be divided.
138.03 Examination of an 11 year old boy reveals a gait that includes bilateral Trendelenburg lurch and a right short leg pattern. He has accentuated lumbar lordosis and a protuberant abdomen. He has no pain. Radiographs are shown in Figures 42a and 42b. Treatment should consist of
-
open reduction of both hips
-
open reduction of the left hip and arthrodesis of the right hip
-
bilateral greater trochanter advancement
-
bilateral total hip arthroplasty
-
epiphyseodesis at an appropriate time to achieve limb length equality at completion
of growth
answer
![]() |
![]() |
![]() |
![]() |
back
![]() |
![]() |
![]() |
![]() |
138.03 5.
![]() |
The problem depicted here is bilateral developmental dysplasia of the hip (DDH) with the right leg shorter than the left. The hips have been out so long that open reduction is not an option at this point. Bilateral trochanteric advancement would do nothing to alter the course of this disease. The
Preferred Response –
patient is far to young for total hip arthroplasty. Since he has no pain and the leg length discrepancy is causing most of his problems, answer 5 is the best answer for this question.
139.03
![]() |
Which of the following tests is considered most accurate (both sensitive and specific) in detecting spinal infection? 1 - Indium 111-labeled WBC scan
2 - Technetium Tc 99m scan
3 - Combined technetium Tc 99m and gallium
67 citrate scans
4 - CT
5 - Plain radiograph
back to this question next question
139.03 3.
![]() |
While either technetium or gallium scans can indicate the presence of an infection prior to the development of radiographic changes (i.e. seen on plain film or CT), the two scans combined have greater sensitivity and specificity. MRI is the principal method of evaluating spinal infections, but is not listed as a choice. Indium labled white cell scans are not recommended as a screening for pyogenic vertebral osteomyelitis as they have a high rate of false negative results.
140.03
![]() |
Preferred Response –
![]() |
![]() |
![]() |
![]() |
![]() |
A 25-year-old man has had medial knee pain with sports for the past 4 weeks. He has no swelling, locking, or giving way. Examination reveals no effusion, 5/5 quadriceps strength, medial joint line tenderness, active range of motion from 0 to 140 degrees, and negative Apley and McMurray tests. Radiographs are normal. A sagittal proton density-weighted image is shown in figure 43. What is the next most appropriate step in management?
-
- Arthroscopy with medial meniscus repair
-
- Arthroscopy with partial meniscectomy
-
- Arthroscopy with open meniscus cyst exploration
-
- Aspiration of meniscus cyst
-
- Observation
140.03 5.
![]() |
The image shows what is potentially a tear in the anterior horn of the lateral meniscus. The problem here is that the false positive rate for anterior horn meniscal tears on MRI can be upwards of
75%. None of the invasive options address the possibility of an anterior horn tear, and when correlated with the physical exam, observation is the best choice for this question
141.03
![]() |
back to this question next question
Figures 44a and 44b show the radiograph and bone scan of a 70-year-old man who has right-sided pelvic pain. What is the most likely diagnosis?
-
- Hemangioma
-
- Senile osteoporosis
-
- Multiple myeloma
-
- Metastatic prostate cancer 5 - Paget’s disease
141.03 5.
Second only to osteoperosis as the most common metabolic bone disease, Paget’s disease is most often asymptomatic and diagnosed incidentally on routine x-rays. There three distinct histologic phases:
-
increase in bone resorption with increase in the number and size of osteoclasts, AKA the “hot” phase.
-
Rapid increase in osteoblast activity and new bone formation, AKA the “intermediate” phase
-
Decrease in both osteoclastic and osteoblastic activity, AKA
-
“cold” phase
![]() |
![]() |
![]() |
![]() |
![]() |
![]() |
![]() |
Preferred Response –
In long bones, radiographs initially show a radiolucency in the metaphysis that progresses into the diaphysis. The entire progression of the disease takes years.
The radiograph here shows the typical Paget’s lesion in the right iliac wing with mixed lytic and sclerotic areas.
142.03
Figure 45 shows the radiograph of a patient with rheumatoid arthritis who sustains a fracture as a result of a fall. Which of the following factors is
![]() |
the most likely cause of the fracture? 1 - Anterior femoral notching
-
- Stress riser from total hip arthroplasty
-
- Osteoporosis
-
- Limb malalignment
-
- Osteolysis
142.03 3.
Femoral notching can only be seen in the lateral view, which is not given. The total hip looks well fixed and no osteolysis is present. The
next question
back to this question
overall limb alignment is good. What is evident from this X-ray is diffuse osteoporosis, which predisposes this patient to fracture.
143.03
-
- Bone scan
-
- Ultrasound
-
- CT
-
- Electrodiagnostic studies of the median and ulnar nerves
-
- MRI of the wrist
-
143.03 3
![]() |
Of concern with the mechanism and
location of the patient’s pain is a fracture of the hook of the hammate. Although difficult to see on standard X-rays, special radiographic views can usually show the fracture. However if these fail to delineate the problem, CT scan is the next step.
Preferred Response –
144.03
![]() |
A 3-year-old girl has a left thoracolumbar scoliosis measuring 40 degrees that has progressed 12 degrees in the past year. Neurologic examination is intact. Radiographs show a hemivertebra at T12, with contralateral bar from T11 to L1. Based on these findings, what is the best course of action?
-
- Brace treatment to prevent development of compensatory curves
-
- In situ anterior and posterior fusion
-
- Anterior fusion
-
- Posterior fusion with “growing rod” technique
-
- Excision of congenital bar
back to this question next question
![]() |
![]() |
![]() |
144.03 Preferred Response – 2.
![]() |
The treatment of choice for congenital scoliosis due to hemivertebra with
contralateral bar is in situ anterior and posterior fusion. Brace treatment will not prevent progression in this case and is not indicated. Anterior or
posterior fusion alone will result in crankshaft phenomena. Excision of the
bar does not address deformity due to the hemivertebra. Growing rods are controversial and not routinely indicated.
145.03
![]() |
Figures 46a and 46b show the MRI scans of a 29-year-old man who has shoulder pain and weakness. Physical therapy has failed to provide relief. History reveals that he injured the shoulder 3 years ago when dove for a volleyball. Examination reveals full shoulder motion, external rotation
![]() |
![]() |
![]() |
weakness, and a positive impingement sign. In addition to decompression of the cyst, what is the most appropriate surgical procedure to address this problem?
-
- Distal clavicle excision
-
- Acromioplasty
-
- Bankart repair
-
- Rotator cuff repair
-
- Superior labral repair
145.03 Preferred Response – 5.
![]() |
The MRI shows a cyst in the interval between the supraspinatus and the glenoid. Decompression of this cyst should alleviate most of this patient’s symptopms, However, this patient also has a superior labral tear best seen on the T2 image. The cyst, as with most cysts around joints, arises from the labral tear. The underlying pathology (the tear) needs to be addressed or the cyst will likely recur.
146.03
![]() |
What structure exits from the lesser sciatic foramen?
-
- Piriformis
-
- Inferior gluteal nerve
-
- Posterior cutaneous nerve of the thigh
-
- Nerve to the obturator internus
back to this question next question
-
- Obturator internus
146.03 Preferred Response – 5.
![]() |
The reference here is Netter. Plate number 465. More commonly asked are the structures that pass through the greater sciatic notch… Seven nerves, three vessels, and one muscle:
Seven Nerves |
Three Vessels |
One Muscle |
Sciatic |
Superior Gluteal |
Piriformis |
Superior gluteal Inferior Gluteal |
|
|
Inferior gluteal Internal pudendal |
||
Posterior femoral cutaneous Nerve to quadratus femoris Nerve to obturator externis |
147.03
![]() |
Which of the following fracture positions is most likely to result in a poor outcome following a tibial plateau fracture? 1 - 3 mm of condylar widening
-
- 3 mm of articular step-off
-
- 5 mm of articular gap
-
- 5 degrees of varus malalignment
-
- 5 degrees of valgus malalignment
147.03 Preferred Response – 4.
148.03
-
- Observation with repeat radiographs in 6 months
-
- Physical therapy for abductor strengthening
-
- distal transfer of both greater trochanters
back to this question next question
-
- percutaneous pinning of both proximal femoral physes
-
- bilateral hip valgus osteotomies
-
148.03 Preferred Response – 1.
![]() |
The radiographs depict congenital coxa vara. PT would be a good choice for Perthe’s disease. Trochanteric transfer would be of no benefit. This is not a SCFE, so pinning in not indicated. While valgus osteotomies are an accepted treatment for this condition, the recurrence rate is 50% and since the femoral heads are well seated in the acutabuli, observation is more prudent at this time.
149.03
![]() |
![]() |
![]() |
![]() |
![]() |
![]() |
![]() |
![]() |
![]() |
![]() |
Figure 48 shows the radiograph of a 10-yearold girl who has an asymptomatic humeral mass. What is the best course of action?
-
- Open biopsy
-
- Needle (core) biopsy
-
- Repeat radiographs in
-
months
-
- MRI
-
- CT of the chest
149.03 Preferred Response – 3.
![]() |
The radiograph shows an osteocondroma of the proximal humerus. As the patient is asymptomatic, observation is the only intervention needed.
150.03 Phantom limb pain has been reported in what percentage of adult lower extremity amputees?
back to this question next question
1- 0 % to 10%
2- 20 % to 30%
3- 40 % to 50%
4- 60 % to 70%
5- 80 % to 90%
back answer
Question 150.03
Answer = 4
Phantom limb pain occurs more frequently than previously thought. A recent study in CORR found that it occurred in 63% of lower extremity amputees. Nonpainful phantom limb sensations are even more common at 84%.
Reference(s)
Smith DG, Ehde DM, Legro MW, Reiber GE, del Aguila M, Boone DA: Phantom limb, residual limb, and back pain after lower extremity amputations. CORR 1999; 361: 2938.
back to this question next question
151.03
Figure 49 shows the AP radiograph of a 50-yearold patient with rheumatoid arthritis who has hip pain. History reveals that the patient underwent total hip arthroplasty 6 years ago but has had poor function for the past year. Laboratory studies for infection are negative. Reconstruction of the acetabulum is best managed with
-
a bipolar head with morcellized bone graft
-
an antiprotrusio cage with morcellized bone graft
-
a cementless acetabular cup with structural bone graft
-
a cemented acetabular cup with structural bone graft 5- a cemented acetabular cup with the defect filled with cement
back answer
Question 151.03
Answer = 2
The xray shows a large cavitary defect with extensive bone loss. The ideal revision option is to have a solid rim fit with morcellized bone graft, but that is not possible in this case –requiring the antiprotrusio cage. Structural bone graft has been shown to have poor long term results with graft dissolution and poor incorporation, unlike morcellized bone graft which incorporates much more rapidly and viably. Bipolar revision has also had poor results and is only considered as a salvage procedure in the elderly patient with questionable stability.
Cemented revision results have also been discouraging but might be indicated in the previously irradiated hip, in the infirm very low demand patient, and in cases of sepsis where antibiotic cement is considered useful. Reference(s)
Haddad FS, Shergill N, Muirhead-Allwood SK: Acetabular reconstruction with morcellized allograft and ring support: A medium term review. J Arthroplasty 1999; 14:788-795.
Markovich GD: Acetabular reconstruction in revision total hip arthroplasty: A review of options. Am J Orthop 1998; 27: 662-670.
back to this question next question
152.03
A 26-year-old patient sustains the closed, isolated injury shown in Figure 50. Management should consist of
-
closed reduction and casting
-
open reduction and internal fixation using rush rods
-
open reduction and internal fixation using rush rods and supplemental bone grafting
-
open reduction and internal fixation using dynamic compression plates
-
open reduction and internal fixation using dynamic compression plates and supplemental bone grafting
back answer
Question 152.03
Answer = 4
Closed reduction and casting is not an option – the results are poor. ORIF with plates is the standard of care. There is some comminution, which in the past some surgeons would acutely bone graft, however studies have found no difference in union rates with or without bone graft. Rush rods are rarely used if at all, but could be justified in segmental fractures, open fracture with soft tissue or bone loss, pathologic fracture, failed plating, or multiple injuries.
Reference(s)
Wright RR, Schmeling GJ, Schwab JP: The necessity of acute bone grafting in diaphyseal forearm fractures: A retrospective review. J Ortho Trauma 1997;11:288-294
Wei SY, Born CT, Abene A, Ong A, Hayda R, Delong WG: Diaphyseal forearm fractures treated with and without bone graft. J Trauma 1999;46:1045-1048.
Koval KJ (ed):OKU 7. Rosemont, IL, AAOS, 2002, pp 307-316.
back to this question next question
153.03
A 19-year-old boy has had right lower leg discomfort for the past 4 months and a recent fever. Laboratory studies reveal a WBC count of 10,000 and an ESR of 31. A radiograph and MRI scan are shown in Fig 51a and 51b. A biopsy specimen is shown in
Question 153.03
![]() |
![]() |
![]() |
![]() |
![]() |
![]() |
![]() |
![]() |
![]() |
![]() |
![]() |
![]() |
Fig 51c. What laboratory finding is most likely associated with
this disease entity?
-
acid fast bacilli
-
gram-positive cocci
-
translocation (9:22)
-
translocation (11:22)
-
mutation of the Rb gene
back answer
Answer = 4
The first question is what is it? The xray shows a mass in the leg with periosteal reaction in the fibula. The MRI shows a soft tissue mass arising from the fibula that looks aggressive. The path shows a bunch of small blue cells. The combination of small blue cells with a soft tissue mass and and onion skin appearance on xray in a patient in the 2nd decade of life has to scream Ewing’s sarcoma. 85% of Ewing’s sarcomas have the 11:22 translocation. None of the other answers are associated with Ewing’s.
Reference(s)
Gibbs CP, Weber K, Scarborough MT: Malignant bone tumors. Inst Course Lect 2002;51:413-428
back to this question next question
154.03
Figure 52 shows the anteroposterior axis of the distal femur (vertical line) and epicondylar axis (horizontal line). How is the epicondylar axis oriented in relationship to the posterior condylar line (line between the most posterior aspect of the posterior femoral condyles)?
-
Parallel
-
Approximately 3° internally rotated
-
Approximately 3° externally rotated
-
Approximately 6° internally rotated
-
Approximately 6° externally rotated
back answer
Question 154.03
Answer = 3
The epicondylar axis is 3 degrees externally rotated compared with the posterior condylar line. This is important because aligning the component parallel with the posterior condylar line will result in a component too internally rotated causing
patellofemoral problems. 34% will require lateral release, compared with only 6% in knees with 3 to 5 degrees of external rotation in relation to the posterior condylar line.
Reference(s)
Griffin FM, Math K, Scuderi GR, Insall JN, Poilvache PL. Anatomy of the epicondyles of the distal femur: MRI analysis of normal knees. J Arthroplasty 2000;15:354-359.
back to this question next question
155.03
A 35-year-old woman sustained a lateral subtalar dislocation of the left foot in a motor vehicle accident. Initial attempts at closed reduction under sedation are unsuccessful. The inability to reduce the joint is most likely the result of
-
the lack of adequate anesthesia
-
interposition of the extensor digitorum brevis
-
entrapment of the posterior tibial tendon
-
impingement of the talar head on the calcaneus
-
fracture of the sustentaculum tali
back answer
Question 155.03
Answer = 3
A lateral subtalar dislocation (everything below the talus moves lateral) is less common than medial – medial is 4 times more common. Closed redution is successful in 15-20% of lateral dislocations. The most common block to reduction is the posterior tibial tendon – not to be confused with the EDB in medial dislocations – see question 136.
Reference(s)
Koval KJ (ed):OKU 7. Rosemont, IL, AAOS, 2002, p 547-564.
Bellabarbara C, Sanders R: Dislocations of the foot, in Coughlin MJ, Mann RA (eds): Surgery of the Foot and Ankle, ed 7. St Louis, MO, Mosby, 1999, p 1520-1530.
back to this question next question
A 12-month-old boy has refused to move his right lower
156.03 extremity for the past 24 hours; however, his parents deny any history of trauma. He has a rectal temperature of 102.2. Examination reveals marked guarding during attempted passive motion of the right hip; however, motion of the lumbar spine and right knee is tolerated. Lab studies show a peripheral WBC count of 15,500 and an ESR of 60. A pelvic radiograph is normal. Management should now consist of
-
radiographs of the spine
-
radiographs of both lower extremities
-
blood cultures
-
needle aspiration of the right hip
-
pelvic MRI
-
-
Question 156.03
Answer = 4
back answer
A recent study in JBJS looked at 4 factors to predict the likelihood of septic arthritis in children – fever, non-weight-bearing, ESR > 40, and WBC >12000 – CRP was not included but is also a useful test. If only 1 factor is positive there is less than 5% probability of septic arthritis. With 2 it is 33-57%, 3 it is 93-97%, and all 4 it is 99.8%. The exam of the spine and knee is normal, and the clinical picture points to a septic hip. Needle aspiration should be done in the OR to allow for subsequent I&D. Blood cultures should be drawn but don’t address the problem. U/S can also be useful if the diagnosis is questionable. MRI is used less frequently, but can help if a soft tissue infection such as psoas abscess is suspected.
Reference(s)
Sucato DJ, Schwend RM, Gillespie R: Septic arthritis of the hip in children. JAAOS 1997;5:249-260.
Kocher MS, Zarakowski D, Kasser JR: Differentiating between septic arthritis and transient synovitis of the hip in children: An evidence based clinical prediction algorithm. JBJS Am 1999;81:1662-1670.
back to this question next question
157.03 What structure is the primary static restraint to inferior glenohumeral translation with the arm in 0° of abduction?
-
Anterior band of the inferior glenohumeral ligament
-
Posterior band of the inferior glenohumeral ligament
-
Middle glenohumeral ligament
-
Superior glenohumeral ligament 5- Superior labral complex
back answer
Question 157.03
Answer = 4
The superior glenohumeral ligament is the primary static restraint to inferior translation with the arm in 0 abduction (along with the coracohumeral ligament which is stronger). The inferior glenohumeral ligament comes into play with increasing abduction (primarily the anterior band, the posterior band is less consistent). The MGHL functions in the middle and lower ranges of abduction but is variable and absent in 40% of the population. The labrum helps add stability by increasing the total contact surface area.
Reference(s)
Matsen FA, Lippett SB, Sidles JA, Harryman DT: Practical Evaluation and Management of the Shoulder. Philadelphia, PA, WB Saunders 1994, p 59-110.
Blaiser RB, Guldberg RE, Rothman CT: Anterior shoulder instability: Contribution of rotator cuff forces and capsular ligaments in a cadaver model. J Shoulder Elbow Surg 1992;1:140-150.
back to this question next question
158.03
A 23-year-old woman who sustained a femoral shaft fracture in a motor vehicle accident is treated the night of injury with reamed, statically locked intramedullary nailing. An intraoperative radiograph is shown in Fig
53 . Management should now consist of
-
touchdown weight bearing for 6 weeks
-
weight bearing as tolerated
-
open reduction and stabilization of the femoral neck fracture
-
conversion to a cephalomedullary (reconstruction) nail with piriformis entry
-
conversion to a cephaolmedullary nail with trochanteric entry
back answer
Question 158.03
Answer = 3
Femoral neck fractures occur in combination with shaft fractures in 2.5 to 5% of cases. The goal is anatomic reduction and stable fixation of both fractures.
Nonoperative treatment in this 23 year old should be obviously wrong. Cephalomedullary nail would be an option if this were noted prior to nailing the femur, but removal of the nail has too much potential to further displace the neck fracture and disrupt the blood supply. ORIF with screws anterior to the nail is the best option in this case. There is no consensus as to the best treatment if the fracture is found prior to nailing the femur – cephalomedullary nail, antegrade nail with screws anterior to the nail, and retrograde nail with fem neck screws are all options.
Reference(s)
Browner BD, Jupiter JB, Levine AM, Trafton PG (eds): Skeletal Trauma, ed 2. Philadelphia, PA, WB Saunders, 1998, pp 1787-1789.
Swiontkowski MF: Intracapsular fractures of the hip. JBJS Am 1994; 76: 129-138.
back to this question next question
159.03
Figures 54a and 54b show the radiograph and MRI of an 18-year-old woman who has knee pain that awakens her from sleep. A chest CT is negative for metastatic disease. A bone scan is shown in Fig 54c, and a biopsy specimen Question 159.03
is shown in Fig 54d. What is the Musculoskeletal Tumor Society stage of her disease?
-
Ia
-
Ib
-
IIa
-
IIb
-
III
-
back
Answer = 4
answer
The images show an osteosarcoma of the distal femur, which is an isolated lesion on bone scan and no metastases on chest CT. The staging system consists of benign lesions are stage 0; low-grade malignant lesions are stage IA(intracompartmental) or
IB(extracompartmental); high-grade malignant lesions are stage IIA(intracompartmental) or IIB(extracompartmental); metastatic lesions are stage III regardless of grade. The MRI shows that the tumor has extended into the soft tissue, making it extracompartmental - IIB.
Reference(s)
Menendez LR (ed): OKU: Musculoskeletal Tumors. Rosemont, IL, AAOS 2002, p 11-20
Enneking WF, Spanier SS, Goodman MA: A system for the surgical staging of musculoskeletal tumors. CORR 1980;153:106-120.
back to this question next question
160.03
Most medical errors result primarily from
Question 160.03
Answer = 3
-
negligence
-
malice
-
system failure
-
inexperience
-
physician fatigue
Negligence, malice, and inexperience are obviously wrong because that would mean that there are a lot of really bad doctors practicing. Fatigue is tempting, especially with all the hype about the 80 hour work week, but again it would place blame on the doctors themselves – the same ones who wrote the question. System failure is the only answer that doesn’t put the blame squarely on the physician.
Reference(s)
Brennan TA, et al: Incidence of adverse events and negligence in hospitalized patients: Results of the Harvard Medical Practice Study I. N Engl J Med 1991;324:370-376.
Kohn LT, Corrigan JM, Donaldson MS (eds): To Err is Human: Building a Safer Health System. Committee on Quality Health Care in America. Institute of Medicine, Washington, DC, National Academy Press, 2000.
Leape, LL: Error in medicine, JAMA 1994;272:1851-1857.
back to this question next question
161.03
On the axial T1-weighted MRI scan shown in Figure 55, taken through the L4-L5 disk space, the structure at the tip of the arrow represents 1- the dorsal root ganglion.
-
the vertebral artery.
-
a facet joint cyst.
-
a far lateral disk herniation.
Back answer
-
a central disk herniation.
![]() |
161.03
Answer: 1. By process of elimination, one can answer this question. The structure shown is not fluid-filled such as an artery or cyst, and is not central and is symmetric. In addition, it has the same signal as the spinal cord and is located where the dorsal root ganglia are found
Beattie PF, Myers SP: Magnetic resonance imaging in low back pain: General principles and clinical issues. Phys Ther 1998; 78:738-753. Finneson BE: Low Back Pain, ed 2. Philadelphia, PA. JB Lippincott, 1981, pp 27-44.
162.03
162.03
Answer: 1
A 1-year-old boy has a 2-cm limb-length discrepancy and asymmetrical limb circumference. Management should consist of 1-abdominal ultrasound and serum alpha-fetoprotein.
-
skeletal survey.
-
skeletal survey and a PA hand radiograph to determine bone age.
-
MRI of the lower extremeties.
-
collagen propeptide screen.
95% of the population has a limb length discrepancy of less than 1.1 cm between sides. Pathologic hemihypertrophy( or atrophy) is defined as a difference of greater than 5%. Idiopathic hemihypertrophy is associated with an increased risk of renal malformations and with intra abdominal neoplasms such as Wilms tumor and hepatic tumors (which often show increased serum AFP) It is recommended to obtain an ultrasound and alpha feto protein) at time of diagnosis of the hemihypertrophy and then every 6-12 mos until the child is 5 yrs old. Moseley CF: Assessment and prediction in leg-length discrepancy.
Back answer
Instr Course Lect 1989; 38:325-330.
Sponseller PD (ed): Orthopaedic Knowledge Update: Pediatrics 2. Rosemont, IL, American Academy of Orthopaedic Surgeons, 2002, pp 183-190.
163.03
163.03
Answer: 2
For complete amputations through the midforearm, replantation usually is not recommended if the warm ischemia time is more than how many hours?
1- 4
2- 6
3- 8
4- 12
5- 24
An amputated part should be wrapped in a saline-soaked gauze in a sturdy plastic bag placed on ice. Patient age, any associated medical condition, smoking, type of injury, and duration of ischemia are factors to consider before replantation is done. Warm ischemia time should typically be less than 6 hours. Because of the minimal muscle present in fingertips, they remain potentially viable up to 12 hours. Ischemia time can be prolonged to 24 hours by cooling the digits. A study by Axelrod in 1991, of 29 patients with an incomplete (26) or complete amputation
(3) of the upper extremity proximal to the wrist showed that limb survival rates are very high (93%). No clear correlation between level of injury, degree of nerve lesion, bone pathology, and functional outcome achieved was found. There was a weak correlation between the type of wound and the functional recovery. He did not specifically address ischemia time.
Manske PR (ed): Hand Surgery Update. Englewood, CO, American Society for Surgery of the Hand, 1994, pp 30-31.
Koval KJ (ed): Orthopaedic Knowledge Update 7. Rosemont, IL, American Academy of Orthopaedic Surgeons, 2002, pp 352-353.
back to this question next question
Axelrod TS, Buchler U: Severe complex injuries to the upper extremity:
Revascularization and replantation. J Hand Surg Am 1991;16:574-584. 164.03
Unreamed nails are associated with an increased occurrence of what complication when compared with reamed nails in the treatment of distal femoral shaft fractures?
164.03
Answer: 1.
-
Delayed union
-
Infection
-
Pulmonary dysfunction
-
Compartment syndrome
-
Shortening
. A review of the literature included that reamed nailing remains the treatment of choice. A lower need for secondary procedures to obtain union without causing an increase in pulmonary dysfunction. Clatworthy performed a study comparing unreamed versus reamed nails and found that fractures in the unreamed group were slower to unite (34.9 weeks) than those in the reamed group (28 weeks). In the unreamed group, 14 secondary procedures were required to enhance healing versus 3 in the reamed group. To achieve quicker union and fewer implant failures, they recommended the use of reamed nails of at least 12 mm in diameter in female patients and 13 mm in male patients.
Back answer
In a study by Tornetta, there was no statistical difference in operative time, transfusion requirements, or hypoxic episodes between the groups. Intraoperative blood loss was greater in the reamed group. The time to union was 80 days for the reamed group and 109 for the unreamed group. This difference was most dramatic in the distal femur, with union in the reamed group occurring in 80 days compared with 158 days in the unreamed group. There were more technical complications and delayed unions in the unreamed group.
Koval KJ (ed): Orthopaedic Knowledge Update 7. Rosemont, IL. American Academy of Orthopaedic Surgeons, 2002, pp 453-463.
Clatworthy MG, Clark DI, Gray DH, Hardy AE: Reamed versus undreamed remoral nails: A randomized, prospective trial. J Bone Joint Surg Br 1998; 80:485-489.
Tornetta P III, Tiburzi D: Reamed versus undreamed anterograde femoral nailing. J Orthop Trauma 2000; 14:15-19.
back to this question next question
165.03
An 18-year-old soccer player has acute thigh pain after a game. Initial radiographs are normal. Sagittal MRI scans are shown in Figures 56a and
165.03
-
Myositis ossificans
-
Soft-tissue sarcoma
-
Hemangioma
-
Muscle contusion
-
Muscle rupture
-
56 B. What is the most likely cause of the rectus femoris signal abnormality?
![]() |
![]() |
![]() |
![]() |
![]() |
Back answer
Answer: 5.
The history is consistent with all of the above choices; therefore, we rely on the imaging study for diagnosis. One can see that the rectus femoris muscle is divided with fluid interposed. This is consistent with a muscle rupture. The fluid signal is dark in T1 and bright on T2, and is circumscribed by the fascia of the muscle. In fact, Dr. Neitzschman, of Tulane, described such a case in the Journal of Louisiana Medical Society.
Weishaupt D. Schweitzer ME, Morrison WB: Injuries to the distal gastrocnemius muscle: MR findings. J Comput Assist Tomogr 2001; 25:677-682.
Neitzschman HR: Radiology case of the month: Painless mass in my right thigh. Rectus femoris tear of the musculotendinous junction presenting as a pseudotumor. J La State Med Soc 2001; 153:17-18.
El-Khoury GY, Brandser EA, Kathol MH, Tearse DS, Callahan JJ: Imaging of muscle injuries. Skeletal Radiol 1996; 25:3-11
Weishaupt D. Schweitzer ME, Morrison WB: Injuries to the distal gastrocnemius muscle: MR findings. J Comput Assist Tomogr 2001; 25:677-682.
Neitzschman HR: Radiology case of the month: Painless mass in my right thigh. Rectus femoris tear of the musculotendinous junction presenting as a pseudotumor. J La State Med Soc 2001; 153:17-18.
El-Khoury GY, Brandser EA, Kathol MH, Tearse DS, Callahan JJ: Imaging of muscle injuries. Skeletal Radiol 1996; 25:3-11. 166.03
A 28-year-old woman undergoes anterior cruciate ligament reconstruction using a quadruple-looped semitendinosus-gracilishamstring autograft. During the first 6 weeks of therapy, the patient should refrain from 1- prone hangs.
-
passive extension.
-
passive and active assisted range of motion.
-
isokinetic quadriceps strengthening.
-
patellar mobilizations.
back to this question next question
166.03
Answer: 4.
![]() |
![]() |
![]() |
According to Campbell’s, at 6 weeks, a patient with a bone patella bone graft will be doing all of the above exercises. However, the quadriceps strengthening does not occur until 4 to 6 weeks, whereas, the other choices will occur before one month. The authors note that they proceed slower with postoperative ACL rehab when a hamstring graft has been used.Phillips
BB: Arthroscopy of the lower extremity, in Cannle ST (ed): Campbell’s Operative Othopaedics. St. Louis, MO, Mosby Yearbook, 1998, pp 1470-1561.
Arendt EA (ed): Orthopaedic Knowledge Update: Sports Medicine 2, Rosemont, IL, American Academy of Orthopaedic Surgeons, 1999, pp 307-316.
167.03
167.03
Answer: 2.
Figure 57 shows the radiograph of a patient who reports the sudden onset of hip
pain. What is the primary cause of the failure? 1- Excessive hip range of motion
-
Implant manufacturing method
-
Patient weight
-
Patient activity
-
Acetabular liner dissociation
Back answer
The picture shows a fractured head of the femoral implant. Ceramics have been known to fracture. The early ceramic implants made in the 1970s had a fracture rate as high as 13.4%. The pioneer ceramic materials were developed for industrial applications. Some of the material had insufficient purity, low density, and course grained microstructure, which led to poor mechanical strength of the material. The original materials were sintered in air. Now the standard procedure is to use hot isostatic pressing which optimizes the material density and microstructure. The current rate of ceramic head failure is somewhere in the range of 4 per 100,000. A study by Willmann concluded that the technical improvements in ceramics for a total hip replacement have improved the reliability of ceramic femoral heads
Willmann G: Ceramic femoral head retrieval data. Clin Orthop 2000; 379:22-28. 168.03
168.03
Answer: 3.
Sterilization by irradiation significantly decreases the structural and mechanical properties of an allograph at what dosage? 1- 1 Mrad
-
2 Mrad
-
3 Mrad
-
4 Mrad
-
5 Mrad
back to this question next question
This question is based on a study by Gibbons. Frozen specimens were exposed to either 2 or 3 Mrads of gamma irradiation. They found that mechanical properties of the graft were not significantly altered following 2 Mrads of irradiation; however, 3 Mrads significantly reduced the graft strength
.Buckwalter JA, Einhorn TA, Simon SR (eds): Orthopaedic Basic Science: Biology and Biomechanics of the Musculoskeletal System. Rosemont, IL, American Academy of Orthopaedic Surgeons, 2000, pp 582-616.
Gibbons MJ, Butler DL, Grood ES, et al: Effects of gamma irradiation on the initial mechanical and material properties of goat bone patellar tendon-bone allografts. J Orthop Res 1991; 9:209-218.
169.03
169.03
Answer: 3.
A 37-year-old man has neck pain and stiffness
![]() |
following a motor vehicle accident 1 week ago. Examination reveals decreased active range of motion in all planes of the cervical spine, weakness of grip strength, and a bilateral Hoffman’s sign. A
lateral radiograph is shown in Figure 58. Further evaluation should include
-
electromyography and nerve conduction velocity studies of the upper extremity.
-
MRI of the lumbar spine.
-
MRI of the cervical spine.
-
flexion-extension radiographs.
-
a bone scan.
Back answer
The patient’s history and physical is consistent with cervical myelopathy. Proper imaging of the cervical spine is essential in patients with cervical myelopathy. Lateral radiographs, as well as lateral radiographs made with the spine in flexion and extension can demonstrate narrowing of the disk space, osteophytes. Although the flex/ex films may demonstrate instability, an MRI is an essential test in this patient who is having neurological symptoms
![]() |
![]() |
![]() |
![]() |
![]() |
![]() |
![]() |
![]() |
.Bernhardt M, Hynes RA, Blume HW, White AA III: Cervical spondylotic myelopathy. J Bone Joint Surg Am 1993; 75:119-128.
MacDonald RL, Fehlings MG, Tator CH, et al: Multilevel anterior cervical corpectomy and fibular allograft fusion for cervical myelopathy. J Neurosurg 1997; 86:990-997.
Truutness E, Herkowitz HN: Cervical spondylotic myelopathy and radiculopathy. Instr Course Lect 2000; 49:339-360.
170.03
Figures 59a and 59d show the current radiographs and biopsy specimens of a 56-year-old woman who has increasing pain 1 year after undergoing curettage and stabilization of a distal femoral bone lesion. Chest CT and bone scans do
next question
![]() |
![]() |
not show lesions elsewhere. Management should now consist of 1-chemotherapy.
-
radiation therapy.
-
wide excision of the distal femur.
-
marginal excision of the posterior exophytic mass.
back to this question
5- irrigation and debridement.
![]() |
![]() |
![]() |
170.03
Answer: 3.
The lesion is a chondrosarcoma. It is the second most common primary malignant tumor of bone after osteosarcoma. The most common presenting symptom is localized pain of relatively long duration anywhere from several months to 10 years.
Radiographic features are characteristic. Central chondrosarcomas of long bones occur primarily in the metaphysic or diaphysis. There is cortical thickening, large erosions, expansion of the bone, and often matrix calcification. Histological features include a lobular growth pattern, many cells with plump nuclei, permeation of the bone trabeculae, and myxoid change. Wide resection is the only treatment, as it is almost always resistant to chemotherapy or radiation.
Back answer
Campanacci M. Enneking WF: Central chondrosarcomas, in Campanacci M, Enneking WF (eds): Bone and Soft Tissue Tumors, ed 2. New York, NY, SpringerVerlag, 1999, pp 283-318.
McCarthy EF, Frassica FJ: Primary bone tumors, in McCarthy EF, Frassica FJ (eds):
Pathology of Bone and Joint Disorders with Clinical and Radiographic Correlation. Philadelphia, PA, WB Saunders, 1998, pp 235-244.
171.03
Slip progression in isthmic spondylolisthesis occurs in
-
5% of patients with spondylolisthesis, is more common in girls, and is rare after skeletal maturity.
-
5% of patients with spondylolisthesis, is more common in boys, and is rare after skeletal maturity.
-
5% of patients with spondylolisthesis, is more common in girls, and is common after skeletal maturity.
-
25% of patients with spondylolisthesis, is more common in boys, and is common after skeletal maturity.
-
25% of patients with spondylolisthesis, is more common in girls, and is rare after skeletal maturity. 171.03
Answer: 1.
The incidence of spondylolysis, with or without spondylolisthesis, is 4.4% at the age of six compared with about 6% in adulthood. The male:female ratio of affected subjects is 2:1 at the age of 6 and in adulthood. Spondylolisthesis, does not exist at birth. Slipping, when it occurs, usually is demonstrable at about the same time that the pars interarticularis defect is first detected roentgenographically. Slipping may increase up to the age of 16, but does so rarely.
Progression of the slip occurs in 4-5% of cases, it is rare during adolescence
back to this question next question
and is more common in girls and in pts with dysplastic spondylolisthesis.
Fredrickson BE, Baker D, McHolick WP, Yuan HA, Lubicky JP: The natural history of spondylolysis and spondylolisthesis. J Bone Joint Surg Am 1984; 66:699-707.
Sponseller PD (ed): Orthopaedic Knowledge Update: Pediatrics 2. Rosemont, IL, American Academy of Orthopaedic Surgeons, 2002, pp 143-152.
-
Which of the following structures is the primary stabilizer of the ankle under physiologic loading conditions?
-
Deltoid ligament complex
-
Posterior talofibular ligament
-
Anterior talofibular ligament
-
Anterior distal tibiofibular ligament
-
Calcaneofibular ligament
Back answer
-
Which of the following structures is the primary stabilizer of the ankle under physiologic loading conditions? A. Deltoid ligament complex
-
Posterior talofibular ligament
-
Anterior talofibular ligament
-
Anterior distal tibiofibular ligament
-
Calcaneofibular ligament Answer: A Deltoid ligament complex
The deltoid ligament is the primary medial stabilizer of the ankle where it resists an eversion load under physiologic conditions. All other ligaments are lateral which are under compression loads, thus ligaments are secondary stabilizers.
Reference: Rockwood & Green / Fractures in adults / 5th Edition / pp2006-7
back to this question next question
-
-
-
When stabilizing an intertrochanteric femoral fracture, what is the most frequent complication associated with the device shown in Figure 60?
A Superficial Infection
B Deep Infection
C. Hardware Failure
D. Lag Screw Cut-out
E. Femoral Shaft Fracture
Back answer
-
When stabilizing an intertrochanteric femoral fracture, what is the most frequent complication associated with the device shown in Figure 60?
A Superficial Infection
B Deep Infection
C. Hardware Failure
D. Lag Screw Cut-out
E. Femoral Shaft Fracture Answer: D. Lag Screw Cut-out
A Superficial Infection 2.9%
B Deep Infection 1.4%
C. Hardware Failure None Reported
D. Lag Screw Cut-out 3.9%
E. Femoral Shaft Fracture 2.0%
Reference: Prospective randomized controlled trial of an intramedullary nail versus dynamic screw and plate for intertrochanteric fractures of the femur / JOT / 2001 / pp394-400
-
Figures 61a and 61b show the radiographs of a 44-year-old woman who has had great toe pain for the past 5 years. What is the most likely diagnosis?
-
Lupus Arthritis
-
Rheumatoid Arthritis
-
Gouty Arthritis
back to this question next question
-
Psoriatic Arthritis
-
Hallux Rigidus
-
-
-
Figures 61a and 61b show the radiographs of a 44-year-old woman who has had great toe pain for the past 5 years. What is the most likely diagnosis?
-
Lupus Arthritis
Back answer
-
Rheumatoid Arthritis
-
Gouty Arthritis
-
Psoriatic Arthritis
-
Hallux Rigidus Answer: E. Hallux Rigidus
-
Lupus Arthritis – Usually erosive
-
Rheumatoid Arthritis – Usually erosive
-
Gouty Arthritis – May be depositional or erosive but typically show crystals
-
Psoriatic Arthritis – Usually erosive particularly distally
-
Hallux Rigidus – Usually depositional/hypertrophic
Reference: Campbell’s Operative Orthopaedics / 10th Edition / pp3689-93 & 39974000
-
Figures 62a and 62b show the radiographs of a 3-year-old boy who has a brittle bone disorder. His parents have been told that anemia, dental abnormalities, and deafness may be associated. The pathophysiology of this condition is failure of A. Vitamin D absorption
-
osteoclastic and chondroclastic resorption
-
collagen cross linking
back to this question next question
-
copper excretion
-
cysteine production
-
-
-
Figures 62a and 62b show the radiographs of a 3-year-old boy who has a brittle bone disorder. His parents have been told that anemia, dental abnormalities, and deafness may be associated. The pathophysiology of this condition is failure of
-
Vitamin D absorption
-
osteoclastic and chondroclastic resorption
Back answer
-
collagen cross linking
-
copper excretion
-
cysteine production
Answer: B. osteoclastic and chondroclastic resorption
Typical of Osteopetrosis (aka Albers-Schonberg disease): increased density of entire skeleton, “Bone within Bone”, widened metaphyses, not shown but typical is hair on end appearance of thichened skull. Caused by failure of osteoclastic resorption
Reference: Orthopeadic management of osteopetrosis: results of a survey and review of the literature / J of Pediatric Orthopaedics / 1999 / pp122-32
-
The lengthening of a muscle during active resistance against an opposing force is classified as what type of contraction?
-
Isotonic
-
Isokinetic
-
Isometric
-
Concentric
-
Eccentric
back to this question next question
-
-
The lengthening of a muscle during active resistance against an opposing force is classified as what type of contraction?
-
Isotonic
-
Isokinetic
-
Isometric
-
Concentric
-
Eccentric Answer: E. Eccentric
-
Isotonic – with unchanging force
-
Isokinetic – with constant speed of motion
-
Isometric – with constant position = not moving
-
Concentric – moving against the force vector
-
Eccentric – moving with the force vector
Reference: OKU7 / pp3-18
-
-
What type of bone healing occurs at the contact area when a transverse radial shaft fracture is directly reduced and stabilized in compression with a plate?
-
Haversian Remodeling
-
Intramembranous ossification
-
endochondral ossification
Back answer
-
periosteal bone formation
-
secondary callus formation
back to this question next question
-
What type of bone healing occurs at the contact area when a transverse radial shaft fracture is directly reduced and stabilized in compression with a plate?
-
Haversian Remodeling
-
Intramembranous ossification
-
endochondral ossification
-
periosteal bone formation
-
secondary callus formation Answer: A. Haversian Remodeling
-
Haversian Remodeling – occurs with tight contact as with compression plate fixation
-
Intramembranous ossification – occurs when gaps are present – may bridge ½ bone diameter
-
endochondral ossification – replacement of cartilage model
-
periosteal bone formation – reactionary bone
-
secondary callus formation – not sure what this is Reference: Orthopeadic Basic Science / 2nd edition / pp377-384
Back answer
-
-
Based on the sagittal and axial MRI scans shown in Figures 63a and 63b, the patient most likely has which of the following clinical findings?
-
Hip Flexor weakness
-
quadriceps and tibialis anterior weakness
-
extensor hallucis longus and hip abductor weakness
-
plantar flexion and eversion weakness
-
cauda equina syndrome
back to this question next question
-
-
-
-
Based on the sagittal and axial MRI scans shown in Figures 63a and 63b, the patient most likely has which of the following clinical findings?
-
Hip Flexor weakness
-
quadriceps and tibialis anterior weakness
-
extensor hallucis longus and hip abductor weakness
-
plantar flexion and eversion weakness
-
cauda equina syndrome
Answer: B. quadriceps and tibialis anterior weakness
MRI shows a foraminal LDH @L4-5. This would affect L4 nerve root leading to TA and Quad weakness. EHL is L5. Eversion is S1. Cauda equine is more central.
Reference: OKU Spine 2 / pp323-331
Physical Examination of the Spine and Extremities / Hoppenfeld / pp250-254
-
-
When converting a hip arthrodesis to total hip arthroplasty, which of the following factors is most predictive of improved walking ability?
-
Length of time from arthrodesis to total hip arthroplasty
-
Limb-length discrepency
-
Position of the arthrodesis
-
Abductor function
-
History of Infection
Back answer
-
When converting a hip arthrodesis to total hip arthroplasty, which of the following factors is most predictive of improved walking ability?
-
Length of time from arthrodesis to total hip arthroplasty
-
Limb-length discrepency
-
Position of the arthrodesis
-
Abductor function
-
History of Infection Answer: D. Abductor function
“The only factor that was predictive of the final functional result with regard to walking ability was the intraopertive status of the gluteal muscles.”
Reference: Total hip arthroplasty for the treatment of ankylosed hips / JBJS / July 2001 / pp 992-8
-
-
A 45-year-old man with a history of smoking two packs of cigarettes a day for 20 years undergoes an arthroscopic Bankhart repair under an interscalene block and general endotracheal anesthesia. There are no intraoperative complications. Four hours after surgery, he remains in the recovery room where he continues to require supplemental oxygenation to maintain an oxygen saturation level above 90%. The patient has bilateral breath sounds and a normal electrocardiogram. What is the next most appropriate step in management?
-
Observation
-
Discharge
-
Ventilation-perfusion scan
-
Central venous line insertion
-
Chest radiograph
back to this question next question
-
-
-
A 45-year-old man with a history of smoking two packs of cigarettes a day for 20 years undergoes an arthroscopic Bankhart repair under an interscalene block and general endotracheal anesthesia. There are no intraoperative complications. Four hours after surgery, he remains in the recovery room where he continues to require supplemental oxygenation to maintain an oxygen saturation level above 90%. The patient has bilateral breath sounds and a normal electrocardiogram. What is the next most appropriate step in management?
-
Observation
-
Discharge
-
Ventilation-perfusion scan
-
Central venous line insertion
-
Chest radiograph Answer: E. Chest radiograph
With brachial plexus block there is a small chance of pneumothorax. Interscalene blocks may also result in partial or total spinal anesthesia which can reduce respiratory motor function.
Reference: Campbell’s Operative Orthopaedics / 10th Edition / pp3378-9 Rockwood & Green / Fractures in adults / 5th Edition / pp97-100
-
-
What normal antagonist muscle to the tibialis posterior everts the hindfoot and unlocks the transverse tarsal joint?
-
Anterior tibial
-
Flexor hallicus longus
-
Peroneus longus
-
Peroneus brevis
-
Gastrocnemius-soleus complex
Back answer
-
What normal antagonist muscle to the tibialis posterior everts the hindfoot and unlocks the transverse tarsal joint?
-
Anterior tibial
-
Flexor hallicus longus
-
Peroneus longus
-
Peroneus brevis
-
Gastrocnemius-soleus complex Answer: D. Peroneus brevis
-
Anterior tibial – Does not evert hindfoot
-
Flexor hallicus longus – Not an antagonist to TP
-
Peroneus longus – Does not Unlock Transverse Tarsal Joint
-
Peroneus brevis – Is Antagonist to TP, Everts Hindfoot, & Unlocks Transverse Tarsal Joint
-
Gastrocnemius-soleus complex – Not an Antagonist to TP
Reference: Atlas of Human Anatomy / Netter / pp495-7
-
-
A 26-year-old man sustains a pelvic injury in a motor vehicle accident. AP and Judet radiographs are shown in Figures
64a through 64c. Definitive management should consist of
-
Skeletal traction followed by progressive mobilization
-
closed reduction and percutaneous screw ficxation
-
open reduction and internal fixation via the Kocher-Langenbeck approach
back to this question next question
-
open reduction and internal fixation via the ilioinguinal approach
-
open reduction and internal fixation via the extended iliofemoral approach
-
-
-
-
Answer: C. open reduction and internal fixation via the Kocher-Langenbeck approach
Back answer
Posterior wall / posterior wall fracture should have ORIF via Kocher Langenbeck approach
Reference: Fractures of the Acetabulum: classification and surgical approaches for open reduction / JBJS / December1964 / pp1615-46&75
back to this question next question
![]() |
![]() |
![]() |
![]() |
![]() |
![]() |
![]() |
183.03
back answer
Question 183.03
Answer = 5 Reference(s)
Lovell and Winter’s Pediatric Orthopaedics 3rd ed.
Femoral anteversion decreases from about 40 deg at birth to 15 deg at maturity, the tibia increases lateral rotation from 5 deg at birth to 15 deg at maturity. Increased hip internal rotation is indicative of increased femoral anteversion, this will improve up to about age 8yrs,
(treatment now observation) Thigh foot angle is normal at 10 deg (range –5 to 30 deg). Limb length discrepancy of 0.5 cm is not currently significant if asymptomatic. Typically give a shoe lift around 1 cm, consider surgery around 2.5 cm(after prolonged observation, i.e. scanograms). Once again observe.
back to this question next question
184.03
![]() |
![]() |
![]() |
![]() |
back answer
Question 184.03
Answer = 3 Reference(s)
Stuart, The use of allografts in sports med., Instr Course Lect 2002:51:521-524
Preferred method of procurement is a sterile harvest with antibiotic soaks, with or with out low-dose radiation.(< 2 mrad). Preserved by deep freezing, (easy storage, lower cost, shelf life 3-5 yrs, NO donor cells survive) versus cryopreservation ( shelf life
![]() |
![]() |
10 yrs, SOME donor cells, increased cost). Indications: multiple lig recon., revisions, inadequate host tissue(old age), cosmetic. In animal research only host DNA observed at 4 wk, they concluded no donor DNA after 4 wk.
back to this question next question
185.03
back answer
Question 185.03
Answer = 4
Reference(s)
Hardinge, The direct lateral approach to the hip, JBJS Br 1982;64:17-19
In this approach a direct lateral incision is made the hip is exposed by incising the vastus lateralis and gluteus medius tendon in line with the femur distally then curving anterior-superior. The anterior capsule is now visible and can now by incised. The position of instability is therefore extension, external rotation (which stresses your suture repair), this is just the opposite of the posterior approach (i.e. approach of Barrack) which is unstable in flexion adduction internal rotation.
back to this question next question
186.03
![]() |
![]() |
![]() |
![]() |
![]() |
![]() |
![]() |
back picture
![]() |
![]() |
![]() |
![]() |
![]() |
186.03
back answer
Question 186.03
Answer = 5
Reference(s)
Weinstein. The Pediatric Spine, 2nd ed. 2001 p323.
The several key points of this question to understand are that infantile scoliosis often regresses, non-progressive curves and those < 40 degrees are often treated with observation and most importantly a ribvertebra angle difference RVAD of <20 deg is unlikely to progress. The RVAD is calculated by measuring the angle made between the rib and inferior end plate of the apex vertebra on both sides and subtracting the two.
back to this question next question
187.03
![]() |
![]() |
![]() |
![]() |
![]() |
back answer
Question 187.03
Answer = 1 Reference(s)
Dreinhofer. Isolated traumatic dislocation of the hip. JBJS Br 1994;76:6-12.
Anterior dislocation is associated with femoral head impaction answer 1. Posterior dislocation are associated with higher rates of answer 2-5. Overall per this study the poor results are seen in 42% of isolated hip dislocations this is contrary to popular belief that they all do well. Finally ant. dislocations do better than post.
back to this question next question
![]() |
![]() |
![]() |
![]() |
![]() |
![]() |
![]() |
![]() |
![]() |
![]() |
![]() |
188.03
back answer
Question 188.03
Answer = 3 References
Campbell's 10th ed
This patient has a flexible deformity with good muscle strength(strong enough for tendon transfer. Answer 1 may temporally correct the equinous but not the varus. Answer 2 may in fact fix the problem, but is typically reserved for fixed deformity or DJD. Answer 4 would worsen the problem by further flexing the 1st ray. Answer 5 is partially correct, it would help correct the equinous, but not the varus as the post tib tendon is still pulling the foot into varus. This leaves answer three which addresses but issues.
back to this question next question
189.03
back
![]() |
![]() |
![]() |
![]() |
![]() |
picture
![]() |
![]() |
![]() |
![]() |
![]() |
![]() |
![]() |
![]() |
![]() |
![]() |
![]() |
![]() |
![]() |
![]() |
![]() |
![]() |
189.03
back answer
Question 189.03
Answer = 4 Reference(s)
On this MRI one sees decreased signal intensity at L4-5 L5-S1, and a herniated disk on the sagital cut. Epidural abscess can be ruled out, one would see increases signal intensity, as can facet cyst for the same reason. With an annular fissure there would also be increased signal around the posterior aspect of the disk also not seen. Finally the herniation is posterior lateral not foraminal, this is seen on the axial cut.
back to this question next question
190.03
![]() |
![]() |
![]() |
![]() |
![]() |
![]() |
![]() |
back answer
Question 190.03
Answer = 2
Reference(s) Rockwood 5th ed
The interosseous ligament’s fibers are oriented to allow for transfer of force from the radius to the ulna. This helps prevent shorting of the radius with axial load, the radial head also helps with transfer of this force. The ligament is disrupted in the Essex-Lopresti lesion, in this case it is crucial to restore bony/ mechanical(radial head replacement) restraint to prevent proximal migration of the radius.
![]() |
![]() |
![]() |
![]() |
![]() |
back to this question next question
191.03
back answer
Question 191.03
Answer = 2 Reference(s)
Herring. Tachdjian’s Pediatric Orthopaedics. 3rd ed.
Marfan’s syndrome is characterized by tall stature, long thin limbs, dislocation of the ocular lenses, cardiac anomalies(aortic root dilatation), ligamentous laxity, pectus excavatum, spinal disorders (scoliosis in 30-100%), acetabular protrusio. It is transmitted by autosomal dominance, and is due to a defective FBN1 gene.
back to this question next question
192.03
![]() |
![]() |
![]() |
![]() |
![]() |
back picture
192.03
![]() |
![]() |
![]() |
![]() |
![]() |
![]() |
![]() |
back Next picture
![]() |
![]() |
![]() |
![]() |
![]() |
![]() |
![]() |
192.03
back answer
Question 192.03
Answer = 2
Reference(s) Rockwood 5th ed
Good Luck!
![]() |
![]() |
![]() |
![]() |
![]() |
![]() |
![]() |
![]() |
back to this question next question
193.03
![]() |
![]() |
![]() |
![]() |
![]() |
![]() |
back picture
193.03
![]() |
![]() |
![]() |
![]() |
![]() |
![]() |
![]() |
![]() |
![]() |
![]() |
![]() |
![]() |
![]() |
back Next picture
![]() |
![]() |
![]() |
![]() |
![]() |
![]() |
![]() |
![]() |
![]() |
![]() |
![]() |
![]() |
![]() |
![]() |
![]() |
193.03
back answer
Question 193.03
Answer = 4
Wheeless’s Text (on line)
Reference(s)
Low on T1 high on T2 think tumor. Path clearly not a cyst rules out #1, no fat cells rules out #5, not in the hand and not in a tendon sheath i.e. not #2.
Synovial sarcoma Histology: - two forms: biphasic (epitheloid) and monophasic (spindle
cell type); - biphasic form is composed of both epithelial-cell and spindle-cell components, where as the monophasic form can be either the epithelial-cell or the spindle-cell type; - spindle-cell form: - predominate spindle cell component (monophasic synovioma) contains cords of spindle cells which may resemble fibrosarcoma; - diff dx: malignant hemangiopericytoma - fibrosarcoma - spindle-cell squamous-cell carcinoma; epithelioid (w/ glandular component): - might be confused with adenocarcinoma; reveals a biphasic pattern: intermixed areas of "glandular" synovial like cells & spindle shaped fibrous cells; - this glandular area will stain PAS positive; - synovial cells: - have an acinar, ductal, or longitudinally arranged tall columnar cells around acellular "slits" containing mucin; - fibrous component: - arranged in the herringbone pattern of fibrosarcoma;
back to this question next question
Continued next slide
Question 193.03
Answer = 4 Reference(s)
Wheeless’s Text (on line)
Neurilemmoma- nerve sheath tumor which arises from schwann cells; - mass grows slowly and usually does not cause symptoms unless it grows within a confined space such as the tarsal tunnel; - histology shows interlacing fascicles (Antoni A areas) and pallisading of nucleii; - asked patient about any family history of type-I (peripheral) neurofibromatosis; - Treatment: - in most cases, the neurilemoma can be shelled out from the nerve w/ blunt dissection;
back to this question next question
194.03 Osteonecrosis occurs in association with which of the following diseases?
-
– Hemachromatosis
-
– Human immunodeficiency Virus
-
– Osteogenesis imperfecta
-
– Marfan Syndrome
-
- Osteoporosis
back answer
Question 194.03
Answer = 2
Reference(s): J. Arthroplasty 2002; 17:135-139
Bone disorders have emerged as a worrisome complication in persons with HIV infection. Over the past few years there have been increasing reports of osteoporosis (wasting of bone tissue due to bone mineral depletion) and osteopenia (moderate bone mineral depletion) in HIV-positive
adults and children. To date, the causes of bone mineral deficiencies in those with HIV have not been well characterized, and data related to bone health in this population are now increasingly being collected and analyzed. Though they are currently uncommon in HIV-positive individuals, skeletal fractures associated with osteoporosis can be painful, debilitating, and potentially life-threatening, offsetting any quality-of-life advantages gained by current use of antiretroviral medications where available.
back to this question next question
195.03 The application of 149° (65°) heat to collagen tissue immediately results in :
-
– increased tissue length
-
– increased collagen synthesis
-
– decreased tissue stiffness
-
– cellular proliferation
-
– no cellular changes
back answer
Question 195. 03
Answer = 3
Reference(s): Instr course Lect 2001; 50:3-11
It has been established that when collagenous tissues are exposed to high temp’s (65-85°C for most tissue types) they undergo rapid denaturation. At a constant temp, shrinkage increases w/increased heat exposure until it reaches a plateau value beyond which no additional shrinkage is observed. Research performed on cadaver glenohumeral capsular tissues suggests that the stiffness of heated collagenous tissue decreases while viscoelastic properties remain unaltered. Stiffness values down to 90% of normal have been measured. Similar to the shrinkage response, the decrease in stiffness is also dose-related. The higher the heating, the greater the loss of stiffness.
back to this question next question
196.03 What is the most common complication reported in the Surgical Treatment of olecranon fractures in adults?
-
– Infection
-
– Ulnar nerve neuritis
-
– Hardware failure
-
– Heterotopic bone
-
– Painful hardware
back answer
Question 196.03
Answer = 5
Reference(s):Instr Course Lect 1995; 44: 175-185
Excellent or good results were noted in 85-88% of pts w/olecranon fxs. Outcome was influence by patient age, malreduction, comminution, and timing of surgery. Complications assoc w/tx include painful hardware in 22-80% of pts, but hw is removed in only 33-66%. HW failure & infection are uncommon (0-6%). Ulnar nerve neuritis and HO are more common (2-13%).
back to this question next question
197.03 A 20 y/o man who sustained a twisting injury to his knee 3 wks ago now reports a persistent limp, is unable to fully extend his knee, and has difficulty doing deep knee bends. Radiographs are normal. What component of the physical examination is most likely to indicate the presence of a meniscus tear?
-
– Lachman’s test
-
– McMurray’s test
-
– Pivot shift test
-
– Joint line tenderness
-
– Medial or lateral joint line opening
back answer
Question 197.03
Answer = 4
Reference(s): JAMA 2001;286:1610-1620.
Nine studies in JAMA 2001 investigated the diagnostic accuracy of the examination for meniscal injuries; all used arthroscopy for the reference standard. Joint line tenderness with a mean sensitivity of 79% and a specificity of 15% was found to be the best predictor of meniscal injury over McMurray (sensitivity of 53%), Apley (sensitivity of 16%), and the medial-lateral grind test (sensitivity of 69%).
back to this question next question
198.03 A 13 y/o girl sustains the injury shown in Figures 69a and 69b in a fall. Examination reveals this to be an isolated injury, and the patient’s neurologic and vascular examinations are normal. Based on these findings, management should consist of
-
– a long arm cast
-
– closed reduction in the emergency dept and a long arm cast
-
– closed reduction under general anesthesia and a long arm cast
-
– closed reduction and percutaneous pin fixation
-
– open reduction and internal fixation
back answer Question 198.03
Answer = 5
Reference(s):Browner et al, Skeletal Trauma, ed 2 pp 1146-1154.
Type B & C lateral condyle fxs in children are potentially unstable. Mintzer and associates recommended pinning of these fractures in the articular surface is not disrupted. To determine the status of the articular surface, they recommended arthrography. If the articular surface is intact, perc
pinning may be performed. If the articular surface shows that the fx line has extended into the joint and the articular surface is separated, open reduction and pinning are necessary. Once the fx is stabilized, the elbow is immobilized for 4 wks. The pins are then removed & motion is begun.
back to this question next question
199.03 The sagittal oblique MRI scan shown in Figure 70 reveals a lesion in the shoulder that typically affects what neurologic structure?
-
– Axillary nerve
-
– Musculocutaneous nerve
-
– Long thoracic nerve
-
– Suprascapular nerve to the infraspinatus muscle
-
– Suprascapular nerve to the supraspinatus muscle
![]() |
![]() |
![]() |
![]() |
![]() |
back answer Question 199.03
Answer = 4
Reference(s): Am J Sports Med 1996; 18:225-228.
Ganglion cysts in the shoulder has been reported in the literature and when they occur in the shoulder typically compress the suprascapular nerve at the spinoglenoid notch primarily affecting the infraspinatus muscle, but depending on their size may also affect the supraspinatus motor brances.
The cysts form either because of a lesion of the capsulolabral complex at the superior/posterosuperior glenoid in the shoulder or because of myxoid degeneration of the capsule.
back to this question next question
200. Figures 71a and 71b show the radiographs of a 38 y/o woman who sustained an isolated right hip injury in a motor vehicle accident. Treatment should consist of
-
– balanced skeletal traction
-
– open reduction and internal fixation with a 135° sliding hip screw
-
– antegrade femoral nail using a piriformis entry point
-
– open reduction and internal fixation with a 95° fixed angle lateral implant
![]() |
5 – a calcar replacement bipolar hemiarthroplasty
![]() |
back
Question 200.03
Answer = 4
answer
Reference(s): JBJS Am 2001; 83:643-650.
Reverse obliquity fractures of the proximal part of the femur is a distinct fracture pattern that is mechanically different from most intertrochanteric fractures. The tx of these fxs remains controversial and their prevalence is unknown. In the series of 55 fxs in the JBJS study, a failure rate of 32%
was identified for all forms of fixation. Use of the 95° fixed angle device resulted in fewer failures (2 of 15) than did the sliding hip screw (9 of 16) and was touted as the implant of choice for this type of fracture pattern. The authors recommended primary prosthetic replacement only in the elderly and osteoporotic patient groups.
back to this question next question
201.03 Figure 72 shows the radiograph of a 4 y/o girl who has a painless right Trendelenburg limp. Management should consist of
-
– closed reduction
-
– open reduction with femoral shortening and pelvic osteotomy
-
– traction followed by closed reduction
-
– medial soft tissue release, traction, and closed reduction
-
– pelvic osteotomy without reduction of the joint
![]() |
![]() |
![]() |
![]() |
back answer Question 201.03
Answer = 2
Reference(s): OKU 7 2002, 387-394
Many high dislocations in children with DDH may remain mobile and pain free for decades despite an inefficient gait. Unilateral dislocations usually create difficulties with limb-length discrepancies and spinal malalignment
(unlike their bilateral counterparts which generally do not need to be reduced). Open reduction is used to obtain absolute concentric reduction. In the dislocated hip reduced at age 15 mths or older, there is usually enough associated bony deformity, either femoral, acetabular, or both to require stabilizing osteotomy to maintain the concentric reduction. Femoral shortening is often necessary to relax soft tissues before a perfect reduction is possible in children > 2 y/o.
back to this question next question
202.03 When using the ilioinguinal approach to the acetabulum, the middle (second) window uses the interval between which of the following structures?
-
– Iliac wing and iliopsoas muscle
-
– Iliopsoas muscle and femoral nerve
-
– Femoral nerve and external iliac artery
-
– External iliac artery and external iliac vein 5 – External iliac vein and pectineus muscle
back answer
Question 202.03
Answer = 3
Reference(s): Letournel E, The Treatment of Acetabular Fractures through the ilioinguinal approach. Clin Orthop 1993;292:62-76.
As described by Letournel: “Lateral retraction of the iliopsoas and femoral nerve, combined with medial retraction of the external iliac vessels, opens the middle, and probably most important, window of the ilioinguinal incision. This window provides access to a large portion of the pelvic brim, from the sacroiliac joint, distally, to the origin of the superior pubic ramus.”
back to this question next question
203.03 A patient is diagnosed with a disk herniation at the L4-L5 level. At the time of lumbar diskectomy, the most effective method of ensuring surgery at the proper level is with
-
– palpation of the spinous process
-
– palpation of the iliac crests
-
– an intraoperative lateral radiograph with a marker in place
-
– an intraoperative AP radiograph with a marker in place 5 – use of a computer-guided navigation system
back answer
Question 203.03
Answer = 3
Reference(s): AAOS Advisory Statement – Wrong-Site Surgery. Rosemont-IL, AAOS 1998.
As described by many spine texts, the 3 techniques employed to place the incision directly over the affected disc are:
-
Notation of the level of the iliac crest on the plain lumbar films
-
Palpation of the last spinous process (usually S1)
-
Preoperative lateral radiograph with skin marker and/or spinal needle (most reliable method).
back to this question next question
204.03 A 19 y/o track athlete who has aching and burning pain in her right leg reports that her symptoms usually begin about 10 minutes into the run and slowly resolve 30-40 minutes after the run. She has decreased her running distance in half, with no change in her symptoms. She notes tingling occasionally over the dorsal aspect of the right foot while running. Examination reveals mild diffuse tenderness over the anterolateral leg but a bone scan of the legs are normal. Stretching and strengthening exercises have failed to provide relief. What is the next step in evaluation?
-
– Three-phase bone scan
-
– Electronmyography and nerve conduction velocity studies
-
– Measurement of compartment pressures before and after exercise
-
– Arteriography 5 – MRI with contrast
back answer
Question 204.03
Answer = 3
Reference(s): Clin Orthop 1975; 113:68-80.
Intensive use of the muscles in the anterior or lateral compartments of the leg may dramatically increase the pressure within these spaces. In the chronic syndrome, recurrent exertional pain disappears after rest, and there is usually no tissue destruction. The pain is described as “burning”, “piercing”, “contracting”, or “cramp-like”. It usually disappears after a few minutes of rest but occasionally persists for hrs or even days. The exertion required to produce the symptoms is often considerable (long-distance runners) and peroneal paresthesias are not uncommon.
Measurement of compartments before and after exercise establishes the diagnosis.
back to this question next question
A 30 y.o. man reports vague right upper extremity discomfort following a recreational sports injury. Fig. 73a-d show a radiograph, a T1-weighted coronal MRI scan, and low- and high-power views of the biopsy specimen. What genetic defect
205.03
is associated with this disease? 11- Type 1 collagen mutation 22- FGF receptor mutation 33- Gs-alpha protein
back
answer
![]() |
![]() |
![]() |
![]() |
![]() |
![]() |
![]() |
![]() |
![]() |
44- Ring chromosome 12
55- Translocation (12;16)
![]() |
![]() |
![]() |
![]() |
![]() |
![]() |
Question 205.03
Answer = 3 Reference(s)
The disease is fibrous dysplasia of bone. Frassica grouped this into lesions with a characteristic appearance. It is a focal or generalized inability of bone forming tissue to produce mature lamellar bone. There is a characteristic whorled pattern in which immature woven bone may be present, having a ground glass appearance. This is what is shown in the figures. As described in the Journal of Bone and Mineral Research, (Vol. 15, No.1, 2000) it was known that an increased level of cAMP in the bone of McCune Albright syndrome patients was associated with a missence mutation in the Gs-alpha protein. The authors were then able to show that allfibrous dysplasia of bone is associated with the same mutation.
(12:16) translocation is associated with myxoid liposarcoma.(OKU7p.159)
FGF receptor point mutation is associated with achondroplasia and other severe forms of dwarfism.(Li C, et.al, Human Molecular Genetics 1999; 8: 35-44)
Type 1 collagen point mutation causes osteogenesis imperfecta.(Miller)
Ring chromosome 12 is associated in a syndrome in which abnormalities include short stature, microcephaly, mental deficiency, characteristic facies, highly arched palate, clinodactyly, and other defects, and is also associated with leiomyomas.
back to this question next question
206.03
CCompared to reamed nailing of tibial shaft fractures, an unreamed technique is associated with 11- more rapid restoration of endosteal blood flow.
22- more rapid fracture healing. 33- decreased cortical perfusion.
44- decreased new bone formation.
55- increased periosteal perfusion.
back answer
Question 206
Answer = 1 Reference(s)
In a study by Schemitsch, et. al., performed on a sheep model of tibia fractures, reamed vs unreamed nailing of spiral tibia fractures were compared. It was found that after 2 and12 weeks post-nailing, there was decreased new endosteal cortical bone formation in the reamed tibias as compared with the unreamed. This implies that reaming caused an insult to the endosteal blood flow. There was no difference in the groups in overallnew bone formation or speed of fracture healing. In a prior study by the same group, again using a sheep model, it was shown that blood supply returned to pre-fracture levels at 6 wks vs. 12 weeks for the reamed group.
It was implied that nailingmay decrease periosteal perfusion indirectly by causing more trauma to the fracture site, but reaming did not seem to make this better or worse. The above authors sited additional studies that found reaming to result in more rapid union, however, this may be due to the fact that reaming allows for a larger nail to be inserted, which in turn increases fracture stability by other means. They concluded that reaming should be avoided in fractures already compromised by severe soft tissue injury, ie., high grade open fractures.
back to this question next question
A 15 y.o. boy who has been undergoing tibial lengthening for the past 2 mo. Now reports pain for the past 5 days while
207.03 distracting the frame. Exam reveals that the pin sites are clean, and the pins are well tensioned. Radiographs show well-formed regenerate bone; however, the pins appear bent.
What is the reason for his pain? 11- Pin tract infection.
22- Pin placement near a cutaneous nerve. 33- Fracture through the regenerate bone
44- Premature consolidation of the regenerate bone 55- Poor tensile strength of the regenerate bone
back answer
Question 207.03
Answer = 4 Reference(s)
Pin tract infection is unlikely, due to the clinical description of clean sites and good tensioning. Radiographically you might also expect to see halos, representing pin loosening. This is a common complication and can be painful, but it is not the best answer for this scenario.
Placement near a cutaneous nerve would have shown up earlier. Fracture through the regenerate likely would have shown up on x-rays. Tensile strength of bone is completely unrelated to pain.
Premature consolidation is a common complication in limb lengthening. Typically, a rate of 1mm per day is initiated, in increments of 0.25 mm q 6hr, and adjustments are made based on the progress of the individual’s healing. Too rapid a rate can result in non-union, and too slow results in premature consolidation. X-ray would show well-formed, consolidated bone, and bending of the pins and wires away from the center since the distraction forces are still being applied. And pain results as well. A new osteotomy is required, a delay of 7-10 days is typically used, and then distraction begins again.
Other complications include soft tissue contractures, intolerable pain, axial misalignments, joint subluxations, fractures during and after frame placement, and peroneal nerve traction palsies.
back to this question next question
208.03
WWhat percent of patients who receive a steroid injection for carpal tunnel syndrome will experience at least transient relief of their symptoms?
11- |
0 % |
22- |
20 % |
33- |
50 % |
44- |
80 % |
55- 100 %
back answer
Question 208.03
Answer = 4 Reference(s)
Per OKU 7 Ch 34, Wrist and Hand Reconstruction, transient relief is achieved in 80% after steroid injection. However, only 22% will be symptom free at 1 yr. This low percentage increases for the subgroup with the following: Sx’s for <1yr., diffuse and intermittent numbness, normal 2-pt discrimination, and absence of weakness, thenar atrophy or denervation potentials on EMG. The high rate of even transient success of this procedure makes it a good diagnostic procedure as well.
209.03 of
back to this question next question
A 44 y.o. man has had aching right shoulder pain for the past several months. Fig. 74a-c show the plain radiograph, sagittal T2-weighted MRI scan, and biopsy specimen. Management should consist
11- extended intralesional curettage. 22- observation.
![]() |
33- wide intra-articular excision. 44- wide extra-articular excision. 55- forequarter amputation.
![]() |
![]() |
![]() |
![]() |
![]() |
![]() |
![]() |
![]() |
Question 209.03
Answer = 1
back
answer
Reference(s)
The lesion is a giant cell tumor. It has the characteristic metaphyseal, eccentric, purely lytic, permeative pattern on plain xray. It does not cross the joint. Pain is typically the initial presentation, with pathological fracture frequent due to the lytic nature of the lesion. The pathology shows the characteristic uniform collection of multinucleated giant cells in a background of mononuclear tumor cells with round or oval nuclei.
The success in treatment of this benign lesion is measured in the recurrence rate. The standard of care is intralesional curettage with high speed burring and some adjuvant therapy, commonly phenol, electrocautery or PMMA (heat of polymerization increases effective tumor kill zone). The lesion is then bone grafted with PMMA and internal fixation applied as necessary. All attempts are made to preserve the joint surface, and certainly to not affect the adjacent joint surface. The typical local recurrence rate with this type of treatment is about 13%, per Ghert, et al, in CORR No.
400, July, 2002. Simple curretage and bone grafting allow recurrence in 30-40% per Frassica. For a benign lesion, forequarter amputation is obviously not indicated. Observation is a setup for progression and for pathologic fracture, and is unacceptable. The results of intralesional curretage with adjuvant equal those of en bloc excision with much less morbidity and no need for complex reconstruction. ( S.Gitelis, et.al. Surgery for Bone and Soft-Tissue Tumors, Lippincott-Raven
1989 , pp 201-202)
back to this question next question
TThe incidence of postoperative nerve palsy is highest 210.03 after total hip arthroplasty for which of the following conditions?
-
Previous hip arthrodesis
-
Previous acetabular fracture
-
Hip dysplasia in women
-
Protusio in men
-
Two-stage exchange for infection.
back answer Question 210.03
Answer = 3 Reference(s)
In a study of 508 consecutive THR’s in 370 pts with DDH, Eggli, et al, found a nerve palsy rate of 1.6%. Most of these pts were women. The overall prevalence of nerve palsy in THR is commonly thought to be about 1%. The predisposing factors listed most commonly are female sex, Hx of DDH, and revision surgery. For the DDH group, the palsies have been associated more with the complexity of the procedure, and not specifically the amount to leg length discrepancy to be corrected.
back to this question next question
211.03
A 15 y.o. cross-country runner has had dull, achy right groin pain for the past 3 weeks. He began his training 16 wks prior to the development of his symptoms. He reports that he has not changed his running habits, shoes, or mileage during this time. He denies sharp pain, numbness, or tingling. Exam reveals an antalgic gait. Initial hip and pelvis radiographs are normal. What is the next most appropriate step in evaluation?
-
CBC and ESR
-
Repeat radiographs
-
CT
-
MRI
-
Femoral arteriography
-
back answer
Question 211.03
Answer = 4 Reference(s)
MRI is becoming the standard of care for suspected occult femoral neck stress fractures. In a study of 19 military patients engaged in endurance training, presenting with hip pain, negative x-rays and positive bone scans, MRI was 100% accurate in diagnosing the occult femoral neck fracture. (Shin, et al, AJSM Vol 24, No 2) The bone scan was only 68% accurate, with 32 % false positives associated with synovial pits, iliopsoas tears, iliopsoas tendonitis, obturator internus tendonitis, AVN of the humeral head, and unicameral bone cyst.
In this scenario, there is no reason to expect that this patient has an infection. At 15 y.o., and with the high level of activity he is able to sustain, a septic joint is unlikely the cause.
Immediate repeat radiographs would be a good idea if the originals were of poor quality, but we have to assume that good quality films were done and showed nothing. Repeat plain films in 6 wks, however, will show evidence of healing if stress fracture had occurred.
CT can actually miss occult femoral neck fractures because the cuts can be parallel to the fracture site and completely miss it. Femoral arteriography is an invasive procedure with way too much potential morbidity to justify its use before MRI.
Besides, if the displacement is so small that you cannot see it on plain films, it may not have had any effect of perfusion. MRI is also the next test of choice for occult femoral neck fracture in the elderly. They can have delayed inflammatory response causing a false-negative bone scan, which can be catastrophic to a geriatric pt if allowed to ambulate, thus displacing an occult fracture.
back to this question next question
A 42 y.o. man has severe LBP, urinary retention, and saddle anesthesia. His medical history is unremarkable.
212.03
What is the most likely diagnosis?
-
Spondylolisthesis
-
Cauda equina syndrome
-
Peripheral neuropathy
-
Herpes zoster infection 5- Cervical myelopathy
back answer
Question 212.03
Answer = 2
Reference(s)
This description is basically a definition of cauda equina syndrome, an orthopedic surgical emergency. In addition one could add bilateral sciatica, bowel incontinence and lower extremity motor weakness to complete the description. The point of this question is basically pattern recognition to emphasize that this constellation of sx’s should not be ignored, because spontaneous recovery does not occur, and bladder sx’s can be permanent. (The Spine, Rothman-Simeone, p.
626)
Spondylolisthesis is common, typically chronic and often asymptomatic. It typically would cause spinal stenosis symptoms, not cauda equina syndrome. A large central disk herniation is usually the culprit in cauda equina syndrome. 1.2-2% of pts with disk herniations with have a cauda equina syndrome. (The Spine, Rothman-Simeone, p.
626)
Peripheral neuropathy typically starts distally, eg, stocking glove distribution and is unlikely to cause severe LBP. Herpes zoster infection presents with severe pain and a vesicular eruptive rash in a unilateral dermatomal distribution, typically above the waist. There are case reports of presentation with LBP and radicular symptoms when lumbar sensory ganglia are involved, (eg, Shapiro, Orthopedics, Vol 19, No 11) but saddle anesthesia and urinary retention are not present.
back to this question next question
FFig. 75 shows the T1-weighted MRI scan of a patient with osteonecrosis. The area depicted by the arrow is supplied by which of the following arteries?
-
Subscapular
213.03 2- Suprascapular
-
Thoracoacromial
-
Posterior circumflex
-
Anterior circumflex
back answer
Question 213.03
Answer = 5
Reference(s)
The arrow on the figure points to the apex of the medial humeral head. It is a well marginated focus of abnormal subchondral bone when compared with the surrounding bone.
The principal blood supply to the humeral head has been shown to arise from the anterior humeral circumflex artery via the anterolateral ascending branch, with lesser contribution by the posterior circumflex. Post traumatic AVN has a reported incidence of 3-14% for 3-part fractures, and 13-34% in 4-part, per Gerber et.al, (J. Shoulder and Elbow Surgery, Nov/Dec, 1998). This reference also argued that even in the face of post-traumatic AVN, outcome varied considerably, and was positively affected by anatomic reduction of the fracture fragments.
back to this question next question
TThe iliac oblique radiograph shown in Fig. 76 shows which of the following acetabular fracture patterns?
214.03 1- Anterior column
-
-
Anterior wall
-
Posterior column
-
Posterior wall
-
Associated both column
back answer
Question 214.03
Answer = 2
Reference(s)
The given view is a poor iliac oblique of the affected side. Typically this best accentuates fractures of the anterior wall and posterior column. The answer given in the handout appears to be wrong. There is no fracture of the anterior column visible in the given figure, i.e., the superior and inferior rami are intact on the affected side. Though somewhat difficult to assess, the posterior column also seems to be intact. Nor is there an obvious posterior wall fragment. Therefore, the correct answer is Anterior WALL.
back to this question next question
FFig. 77 shows a T1-weighted MRI scan. The structure at the arrow is supplied by which of the following nerves?
215.03 1- Axillary
![]() |
![]() |
![]() |
![]() |
![]() |
![]() |
![]() |
![]() |
-
Subscapular
-
Suprascapular
-
Long thoracic
-
Spinal accessory
back answer
Question 215.03
Answer = 2
Reference(s)
The muscle is the subscapularis. It is innervated by the upper and lower subscapular nerve. (The lower subscapular also innervates teres major.) The axillary nerve innervates the deltoid and teres minor. The suprascapular nerve innervates the supraspinatus and the infraspinatus. The long thoracic innervates the serratus anterior (winging when absent), and the spinal accessory innervates the trapezius.
back to this question next question
03.216
A 9-year-old boy who has a slipped capital femoral epiphysis is in the 40th percentile for weight. Which of the following studies should be obtained?
-
MRI of the pelvis
-
CT of the pelvis
-
Bone scan
-
Serum BUN and creatinine levels 5- CBC and erythrocyte sedimentation rate
back answer Question 03.216 Answer = 4
Explanation: A quick SCFE review courtesy of Campbell’s - SCFE occurs more frequently in obese children and is almost twice as common in males as in females. It occurs twice as often in black children as in white children. The left hip is affected twice as often as the right, and bilateral involvement is reported to occur in from 25% to 40% of children. When bilateral slips occur, the second slip usually occurs within 12-18 months of the initial slip.
Several etiological factors have been suggested for SCFE, including local trauma, mechanical factors (especially obesity), inflammatory conditions, endocrine disorders (such as hypothyroidism, hypopituitarism, and chronic renal disease), and genetic factors. Acute slips are those with a sudden onset of usually severe symptoms that have been present for less than two weeks. Chronic slips are characterized by gradual onset and symptoms of more than two weeks’ duration. Acute-on-chronic slips are those with symptoms lasting longer than one month and a recent sudden exacerbation of pain after a relatively trivial injury (1481-1482).
According to Miller, stable slips are those in which weight bearing with or without crutches is possible. Unstable slips are those in which weight bearing is not possible because of severe pain. The recommended treatment is pinning in situ.
SCFE usually occurs between the ages of 10-16 years old, during the adolescent growth spurt. Patients presenting at less than 10 years of age should have an endocrine work-up. Prophylactic pinning of the opposite hip is recommended in patients diagnosed with an endocrinopathy (180).
The above question presents a non-obese patient younger than ten years of age. Therefore an endocrine workup is indicated. Since chronic renal disease is an associated endocrine disorders, a serum BUN and creatinine level are warranted.
back to this question next question
Reference(s)
Loder RT, Greenfield ML: Clinical characteristics of children with atypical and idiopathic slipped capital femoral epiphysis: Description of the age-weight test and implications for further diagnostic investigation. J Pediatr Orthop 2001;21:481-487.
03.217
A 12-year-old boy has a flexible pes planovalgus deformity of the right foot. Orthotic management should consist of 1- a semirigid orthotic with medial longitudinal arch support and medial heel posting
-
a semirigid orthotic with medial longitudinal arch support and lateral heel posting
-
an extra-depth shoe with SACH heel
-
an articulated ankle-foot orthosis
-
a plastizote insert
back answer Question 03.217 Answer = 1
According to the cited Mann reference, asymptomatic flexible flatfoot is a normal variant of the longitudinal arch. In the vast majority of cases it is nothing more than an anatomic observation and does not constitute a pathologic problem. The foot is functional and flexible and does not appear to be any more prone to injury or disability than the normal foot. In evaluating patients, observation of the foot with the patient standing reveals little or no observable longitudinal arch. When the patient stands on tiptoes, normal inversion of the calcaneus occurs, with normal function of the rear foot articulations. Special attention should be given to assessing the degree of tightness of the Achilles tendon, because excessive tightness can limit dorsiflexion of the ankle joint. This is often a source of underlying pathology in symptomatic flatfoot. Individuals with asymptomatic
flexible flatfoot should be reassured that no treatment is indicated and should be further counseled that treatment has not been shown to in any way prevent the development of future problems. Patients with symptomatic flexible flatfoot deformities will complain that their feet tire easily and become painful with prolonged standing. Treatment of symptomatic flexible flatfoot should usually be conservative, including the use of shoe modifications and orthotics that will support the arch and prevent hyperpronation.
Of the above listed answer options, a semi rigid orthotic with medial longitudinal arch support and medial heel posting supports this recommendation by restoring medial arch and preventing hyperpronation of the calcaneus. Of the other options, lateral heel posting will contribute to hyperpronation. SACH is the acronym for solid ankle cushion heel and is more often used in description of prosthetic choices in amputees, allowing for heel compression while ambulating. The articulated AFO does not address the underlying pathology, and a plastizote insert is an accommodative, heat moldable lining usually prescribed for people with insensate feet or arthritis to unload high-pressure areas.back to this question next question
Reference(s)
Mann RA: Flatfoot in adults, in Coughlin MJ, Mann RA (eds): Surgery of the Foot and Ankle, ed 7. St Louis, MO, Mosby, 1999 , pp 733-767. Bordelon RL: Surgical and Conservative Foot Care. Thorofare, NJ, Slack Inc, 1988.
03.218
A 40-year-old man who underwent an anterior cruciate ligament reconstruction 4 months ago now reports anterior knee pain and limited knee motion. Examination reveals well-healed incisions, no effusion, moderate anterior knee tenderness, decreased patellar mobility, active range of motion from 10 to 110*, and a good anterior end point. Radiographs show tibial and femoral tunnels to be in good position. In what anatomic location did this pathologic process most likely begin?
-
Suprapatellar pouch
-
Lateral gutter
-
Posteromedial recess
-
Intercondylar notch
-
Patellar tendon
-
back answer
Question 03.218 Answer = Item Deleted Reference(s)
back to this question next question
03.219
Following a subtrochanteric femoral fracture, the proximal fragment assumes a position of
-
extension, abduction, and internal rotation
-
extension, adduction, and external rotation
-
flexion, adduction, and internal rotation
-
flexion, abduction, and external rotation 5- flexion, abduction, and internal rotation
back answer Question 03.219 Answer = 4
Explanation: The subtrochanteric region starts at the lesser trochanter and extends 5cm distally. This proximal portion of the femur is composed primarily of cortical bone and is slower to heal than the metaphyseal bone of the intertrochanteric region. In subtrochanteric fractures, the proximal fragment is subjected to strong muscular pull from the iliopsoas (flexion), short external rotators (external rotation), and the hip abductors (abduction). Based on this anatomical knowledge, the answer choice is flexion, abduction, and external rotation
Reference(s)
Sims SH: Subtrochanteric femur fractures. Orthop Clin North Am 2002;33:113-126 , viii.
Herscovici D Jr, Pistel WL, Sanders RW: Evaluation and treatment of high subtrochanteric femur fractures. Am J Orthop 2000;29:27-33.
back to this question next question
03.220
Figures 78a and 78b show the MRI scans of a 59-year-old woman who has plantar foot pain and a mass. A biopsy specimen is shown in Figure 78 c. Management should consist of
![]() |
![]() |
![]() |
-
custom foot inserts
-
local steroid injection
-
marginal excision
-
wide excision
-
Syme’s amputation
back answer
Question 03.220 Answer = 4
The images reveal a plantar foot mass with low T1 signal intensity and high T2 signal intensity located on the medial half of the middle of the plantar fascia. The histologic specimen reveals proliferative fibroblastic cells with limited mitoses. The diagnosis is plantar fibromatosis, also known as Ledderhoses disease. The etiology of Ledderhoses disease is unknown. Many factors have been postulated including endocrinopathy, trauma, neuropathy, biochemical and metabolic imbalance, faulty development, infection and patient's occupation. Most of the lesions are asymptomatic and discovered by palpation. Sex ratio is 1:1 with a predilection for young adults. It has been occasionally reported in children. Clinically Ledderhoses disease presents as a mostly unilateral, locally invasive neoplasm, characterized by focal proliferation of fibroblasts in the subcutaneous tissue of the foot sole, usually involving the medial half of the middle of the plantar fascia. This lesion usually is tender but rarely causes a contracture. On MRI imaging, a round or oval, inhomogeneous mass at the foot sole with well-defined margins and signal intensity between skeletal muscle and subcutaneous fat are characteristics that make plantar fibromatosis unique. Surgery is indicated when the lesion is large and painful enough to be disabling. This benign lesion may recur with more aggressive features if incompletely resected – hence the need for wide excision. However, these lesions do not metastasize or become locally destructive.
Reference(s)
Cany L, Stoeckle E, Coindre JM, Kantor G, Bonichon F, Bui BN: Prognostic factors in superficial adult soft
tissue sarcomas: Analysis of a series of 105 patients. J Surg Oncol 1999;71:4-9. back to this question next question
Pisters PW, Leung DH, Woodruff J, Shi W, Brennan MF: Analysis of prognostic factors in 1,041 patients with localized soft tissue sarcomas of the extremities. J Clin Oncol 1996;14:1679-1689.
03.221
When a short cephalomedullary nail is compared to a sliding hip screw and side plate for stabilization of intertrochanteric femoral fractures, the intramedullary device has been associated with a significantly higher
-
amount of blood loss
-
surgical time
-
rate of lag screw cutout
-
rate of poor functional outcomes 5- rate of femoral shaft fracture
back answer Question 03.221 Answer = 5
According to Rockwood and Green, cephalomedullary nails (intramedullary sliding hip screw devices) have several potential advantages. First, due to location, intramedullary fixation theoretically can provide more efficient load transfer in addition to a shorter lever arm resulting in less tensile strain on the implant, thereby decreasing the risk of implant failure. Second, controlled fracture impaction is maintained by incorporation of the sliding hip screw. Third, the operative time is theoretically shorter with less soft tissue dissection and blood loss, potentially decreasing overall morbidity. Most studies comparing intramedullary hip screws with sliding hip screws have revealed no significant differences in surgical time, duration of hospital stay, infection rate, wound complications, implant failure, screw cut-out, or screw sliding. There is also no difference in mobility at six and twelve month follow-up. However, it has been found that patients treated with intramedullary hip screws are at increased risk for femoral shaft fracture at the nail tip and the insertion sites of the distal locking screws. Based on these findings, the authors conclude that routine use of intramedually hip screws cannot be recommended for intertrochanteric hip fracture stabilization.
Reference(s)
Adams CI, Robinson CM, Court-Brown CM, McQueen MM: Prospective randomized controlled trial of an intramedullary nail versus dynamic screw and plate for intertrochanteric fractures of the femur. J Orthop trauma 2001;15:394-400.
Hardy DC, Descamps PY, Krallis P, et al: Use of an intramedullary hip-screw compared with a compression
hip-screw with a plate for intertrochanteric femoral fractures: A prospective, randomized study of one back to this question next question hundred patients. J Bone Joint Surg Am 1998;80:618-630.
03.222
At age 5 years, the spinal canal has grown to approximately what proportion of the expected cross-sectional area? 1-25 %
2-33 %
3-50 %
4-75 %
5-95 %
back answer Question 03.222 Answer = 5
In the cited Lovell and Wintertext, the discussion is centered around the treatment of paralytic scoliosis. The crankshaft phenomenon occurs when there is a solid posterior arthrodesis with sufficient anterior growth remaining to produce a rotation of the spine and trunk with progression of the scoliotic curve. Therefore, considering the state of skeletal maturity and the amount of growth remaining is important prior to fusion. In the treatment of paralytic scoliosis, in which the curve is severe or rapidly progressive, circumferential fusion with segmental instrumentation is the best technique for avoiding the crankshaft phenomenon. As stated in the text, the spinal canal has grown to 95% of its definitive size by age 5 years. Therefore, circumferential arthrodesis is favored because it will have no influence on the size of the spinal canal.
Reference(s)
Dimeglio A: Growth in pediatric orthopaedics, in Morrissy RT, Weinstein SL (eds): Lovell and Winter’s Pediatric Orthopaedics, ed 5. Philadelphia, PA, Lippincott Williams & Wilkins, 2000, pp 33-62. Kabins MB: Congenital and developmental spinal stenosis, in Weinstein SL (ed): The
back to this question next question
Pediatric Spine: Principles and Practice, ed 2. Philadelphia, PA, Lippincott Williams & Wilkins, 2001, pp 203-218.
Dimeglio A, Bonnel F: Le rachis en croissance. Paris, France, Springer-Verlag, 1990. 03.223
What factor is necessary in the development of osteomalacia?
-
Insufficient dietary vitamin D
-
Insufficient dietary calcium
-
Failure to maintain serum calcium and phosphate levels
-
Enteric absorption defects
-
Renal insufficiency
back answer Question 03.223 Answer = 3
According to Buckwalter et al, osteomalacia refers to an increased, normal, or (most commonly) decreased mass of insufficiently mineralized bone matrix. Insufficient mineralization includes unmineralized osteoid and delayed mineralization of osteoid. The most common causes are chronic renal failure, vitamin D deficiency (found in approximately 30% of institutionalized elderly persons), abnormalities of the vitamin D pathway, and hypophosphatemic syndromes. Rarer causes are renal tubular acidosis, aluminum intoxication, hypophosphatasia, absorption defects, and mesenchymal tumors that lead to hypophosphatemic conditions. The single most important causative factor in the development of osteomalacia is the failure to maintain a serum calcium-phosphorus level sufficient to promote mineralization of newly-formed osteoid. Management is based upon the pathophysiology. The unifying theme in the treatment of all osteomalacic conditions is the restoration of the serum calcium-phosphorous level to normal so that normal mineralization of cartilage and bone matrix can occur.
Reference(s)
Buckwalter JA, Eback to this questioninhorn TA, Simon SR (eds): Orthopaedic Basic Scienext question nce: Biology and Biomechanics of the Musculoskeletal System, ed 2. Rosemont, IL, American Academy of Orthopaedic Surgeons, 2000, pp 319-370.
Mankin HJ: Metabolic Bone Disease. Instr Course Lect 1995;44:3-29.
Figures 79a through 79c show the radiograph, CT scan, and biopsy specimen of a 42-year-old woman who has a painful shoulder mass. What is the most likely diagnosis?
03.224
-
Extraskeletal osteosarcoma
-
Myositis ossificans
-
Synovial sarcoma
-
Tumoral calcinosis
-
Calcific myonecrosis
-
back
![]() |
![]() |
![]() |
answer
![]() |
![]() |
![]() |
![]() |
![]() |
![]() |
![]() |
![]() |
Question 03.224 Answer = 2
Myositis ossificans is a reparative process in which bone forms in skeletal muscle most commonly after an episode of trauma. Non-traumatic myositis ossificans occurs in the proximal portions of the extremities (most commonly the medial thigh) and the pelvic and shoulder girdles. It enlarges rapidly and, because of its clinical resemblance to a neoplasm, has been termed "pseudomalignant osseous tumor of soft parts". Despite its ominous early-growth pattern, it matures (like its post-traumatic counterpart) and reaches a static state in weeks or perhaps months. In the above question, no history is given so the diagnosis must be made by figure analysis. Analysis of the radiograph and CT image reveal a well-defined, extraosseous lesion (about the trapezius muscle) with the characteristic maturation-zoning pattern seen in myositis ossificans. This consists of a pattern of a peripheral, thin shell of mature bone about a more radiolucent central area that may or may not contain a faint, amorphous pattern of early ossification. The histology slide reveals mature trabeculae with some fatty marrow in the interior. Treatment consists of observation and physical therapy. Excision is considered only when the lesion is mature. Of the other answer options, the differential diagnosis rests between other lesions that form bone in the soft tissues (extraskeletal osteosarcoma) and lesions in the soft tissue with a pattern of calcification rather than ossification (tumoral calcinosis and calcific myonecrosis in which intrinsic calcification is common). It is important to distinguish between the fine trabecular pattern and modest radiodensity of newly-forming ossification and the speckled "salt and pepper" pattern of more radiodense soft-tissue calcification. Synovial sarcoma more commonly occurs in the lower extremity, is intimately related to tendons, tendon sheaths, and bursal structures, and has chords of epithelial cells with well-oriented fibrous tissue on histology.
Reference(s) back to this question next question
Enzinger FM, Weiss SW: Osseous soft tissue tumors, in Ezinger FM, Weiss SW (eds):
Soft Tissue Tumors, ed 3. St louis, MO, Mosby, 1995, pp 1013-1037. 03.225
When comparing the results of tibial shaft fractures treated with intramedullary nails to those treated in casts, patients treated with intramedullary nails can be expected to have an increased incidence of
-
nonunion
-
malunion
-
delayed union
-
restricted knee motion
-
knee pain
back answer Question 03.225 Answer = 5
Treatment options are numerous when it comes to treating the full spectrum of tibial shaft fractures. Most authors recommend <5 degrees of angular alignment, <10 degrees of rotation malalignment, and <12mm of shortening as guidelines for acceptable alignment of a tibial shaft fracture after healing. As a general rule (according to OKU 7), low-energy and minimally displaced fractures can be treated with initial reduction (as needed) and cast application, with subsequent transition to prefabricated patellar tendon bearing brace and early weight bearing. Intramedullary nailing remains the treatment of choice for unstable and/or displaced tibial shaft fractures. A high incidence of knee pain (10% to 60%) has been associated with IM nailing. Nail removal as a result of knee pain ranges from 10% to 55%, depending on the duration of follow up. The incidence of pain is higher when the nail was inserted by a patellar tendon-splitting approach versus a paratendon approach.
Reference(s)
Hooper GJ, Keddell RG, Penny ID: Conservative management or closed nailing for tibial shaft fractures: A randomized prospective trial. J Bone Joint Surg Br 1991;73:83-85.
Karladani AH, Granhed H, Edshage B, Jerre R, Styf J: Displaced tibial shaft fractures: A prospective back to this question next question randomized study of closed intramedullary nailing versus cast treatment in 53 patients. Acta Orthop Scand 2000;71:160-167.
Koval KJ (ed): Orthopaedic Knowledge Update 7. Rosemont, IL, American Academy of Orthopaedic Surgeons, 2002, pp 479-488. 03.226
What method most effectively identifies musculoskeletal problems during the preparticipation physical examination? 1- Flexibility testing
-
Radiographic evaluation
-
Complete orthopaedic history and questionnaire
-
Observation of athlete during sport specific activities
-
Skinfold examination
-
back answer Question 03.226 Answer = 3
Reference(s)
According to OKU 7, the primary objective of the preseason physical examination is to determine current medical status and identify any conditions that may predispose an athlete to injury or that may be potentially life threatening. In approximately 1% of athletes screened at all levels, something found on the examination disqualifies them from competition, most commonly from a musculoskeletal disorder. The second most common reason is a cardiac abnormality, with hypertrophic cardiomyopathy being the most common cause of sudden death in athletes age 12 to 32 years. The key components to the preparticipation examination are the history, vitals signs, general medical examination, and musculoskeletal examination. The routine use of radiographs and electrocardiograms are not recommended unless history or physical warrants further workup. Additional aspects such as fitness testing (including sport specific activities and flexibility) or body composition may be added but have not been found to be as effective as a basic history obtained by direct questioning.
Koval KJ (ed): Orthopaedic Knowledge Update 7. Rosemont, IL, American Academy of Orthopaedic Surgeons, 2002, pp 99-110. back to this question next question
Glover DW, Maron BJ, Matheson GO: The preparticipation physical examination: Steps toward consensus and uniformity. Physician and Sports Medicine 1999;27.
Gomez JE, Landry GL, Bernhardt DT: Critical evaluation of the two minute orthopedic screening evaluation. Am J Dis Child 1993;147:1109-1113. 227.03
![]() |
. Delayed onset or postexercise muscle soreness is best characterized by
-
muscle cytoskeletal hypertrophy
-
inflammation
-
reduced serum creatine kinase levels
-
increased serum calcium
-
local hematoma
227.03 Answer = 2
Rosemont, IL, American Academy of Orthopaedic Surgeons, 2000, pp 684-716.
Delayed onset muscle soreness, or what is commonly described as postexercise muscle soreness, is the sensation of muscular discomfort and pain during active contractions that occurs in a delayed fashion after strenuous exercise and usually begins at the muscle-tendon junction. In addition to tenderness, there will be strength loss, a reduced range of motion, and elevated levels of serum creatine kinase (3 is wrong). At the ultrastructural level, there is loss of the regular orientation of Z bands and cytoskeletal hypertrophy does not occur until the recovery stage (1 is wrong). Hematoma will not form unless muscle is torn (5 is wrong), but inflammation does occur in response to the release of bradykinin, 5hydroxytryptamine, and prostaglandins. These factors induce the inflammatory response that induces the infiltration of circulating monocytes that become macrophages after entering the affected tissue and cause proteolytic degradation of muscle tissue. This inflammatory response can be reduced by the use of NSAIDs
228.03
-
Increased angiotensin levels
-
Increased gluconeogenesis
-
Increased myocontractility
-
Neuroendocrine release of epinephrine
-
Interstitial fluid shift
-
228.03 Answer= 2
Browner BD, Jupiter JB, Levine AM, Trafton PG: Skeletal Trauma, ed 2. Philadelphia, PA, WB Saunders, 1998, pp 2327-2404.
![]() |
Beaty JH(ed): Orthopaedic Knowledge Update 6. Rosemont, IL, American Academy of Orthopaedic Surgeons, 1999, pp123-137.
The systemic metabolic response to trauma is divided into the ebb phase and flow phase. The ebb phase occurs first and is dominated by cardiovascular instability, alterations in circulating blood volume, impairment in oxygen transport, and heightened autonomic activity. This
![]() |
phase includes hypovolemic or traumatic shock manifested by adrenergic nerve stimulation (release of epinephrine – 4 is wrong), increased levels of angiotensin (1 is wrong). There is also a fluid shift from the interstitial space into the intravascular space (5 is wrong).
After effective resuscitation and restoration of oxygen transport, a secondary group of responses, the flow phase (which is what the question is about) occurs. In this phase, the body is responding to the intense increase in metabolic demand that occurs with the repair processes.
Glucose is the main fuel for wound healing. Gluconeogenesis occurs, using amino acids for its substrate (2 is right). This amino acid substrate is obtained from the skeletal muscle breakdown (3 is wrong).
If interested, page 140 of Skeletal Trauma, 3rd edition is a good source.
back to this question next question
![]() |
![]() |
![]() |
229.03
A 43-year-old man reports a snapping sensation in the left shoulder with overhead activities.
![]() |
![]() |
Figures 80a through 80e show the radiographs, MRI scan, bone scan, and biopsy specimen. Which is the most likely diagnosis?
-
Osteochondroma
-
Chondrosarcoma
-
Osteoblastoma
-
Osteogenic sarcoma
-
Myositis ossificans
![]() |
![]() |
![]() |
![]() |
229.03 Answer= 4
![]() |
Schajowicz F, McGuire MH, Snatini Araujo E, Muscolo DL, Gitelis S: Osteosarcoma arising on the surfaces of long bones. J Bone Joint Surg Am 1998;70:555-564.
![]() |
Temple HT, Scully SP, O’Keefe RJ, Katapurum S, Mankin HJ: Clinical outcome of 38 patients with juxtacortical osteosarcoma. Clin Orthop 2000;373:208-217.
The types of osteogenic sarcoma include high-grade intramedullary, parosteal (juxtacortical) osteosarcoma, periosteal osteosarcoma, and telangiectatic. This case represents a parosteal osteosarcoma, which is typically a low-grade malignant tumor but may harbor foci of higher-grade tumor. Treatment is complete excision, and adjuvant chemo is not necessary unless high-grade areas are found histologically. It is distinguished from osteochondroma in that it lacks corticomedullary trabecular continuity- a radiographic feature present in all osteochondromas (1 is wrong). The pattern of zonation seen radiographically and histologically is different in myositis ossificans where the periphery of the tumor appears more mature than the central portion. This pattern of maturation is reversed in osteosarcoma (5 is wrong).
The histology of parosteal osteosarcoma is characterized by a high degree of structure differentiation and consists of immature woven bone in different stages of maturation to lamellar bone. The lamellar trabeculae, which predominate in most areas, are separated by a fibrous stroma that is composed of spindle cells, which are minimally pleomorphic and have few mitoses. Osteoblastoma can be distinguished from osteosarcoma by osteoblastic rimming of newly formed bone and by the absence of permeation and entrapment of host bone (3 is wrong).
Chondrosarcoma can be intraosseous or juxtacortical and may occur as primary lesion or secondarily in preexisting benign tumors like osteochondroma or enchondroma (25% transformation in Ollier’s or Maffucci’s) and should be suspected with areas of deep scalloping of the endosteal surface of cortical bone
230.03
During surgical treatment of a bicondylar tibial plateau fracture, the surgeon elects to perform a posteromedial second approach to address condylar comminution. Surgical dissection should proceed between what two structures?
-
Gracilis and semitendinosus
-
Semitendinosus and semimembranosus
back to this question next question
-
Semimemranosus and medial gastrocnemius
-
Medial gastrocnemius and tibial nerve
-
Medial collateral ligament and gracilis
230.03 Answer= 3
Georgiadis GM: Combined anterior and posterior approaches for complex tibial plateau fractures. J Bone Joint Surg Br 1994;76:285-289.
![]() |
Burks RT, Schaffer JJ: A simplified approach to the tibial attachment of the posterior cruciate ligament. Clin Orthop 1990;254:216-219.
![]() |
In complex tibial plateau fractures with significant intra-articular comminution, these authors suggest that a single anterior incision does not always allow sufficient exposure for reduction. This is especially true of plateau fractures with a substantial posteromedial fragment (split) component. They report that this type of fracture is rare and often can be considered fracture dislocations. If the posteromedial fragment is not reduced, this can lead knee instability. The use of a second posterior incision allows access for reduction and the placement of a posterior buttress plate and avoids the need for large skin flaps in already traumatized soft tissue.
For this approach, a longitudinal posteromedial incision is made over the medial head of the gastrocnemius. Dissection is carried to the deep fascial layer keeping in mind the medial sural cutaneous nerve. The medial border of the medial gastrocnemius is identified, and the interval is developed between it and the semimembranosus tendon (answer 3). The medial head of the gastroc is retracted laterally to protect the neurovascular structures. This approach is also described for internal fixation of an avulsed tibial attachment of the PCL.
231.03
![]() |
Which of the following events occurs during healing of meniscal tears in the peripheral vascular zone?
-
A plasmin clot forms after injury
-
Inflammatory cells infiltrate the injured area
-
Differentiated meniscal cells cross into the injured area
-
Newly synthesized acellular collagen matrix bonds the wound edges together
-
Healing occurs without a fibrovascular scar
231.03 Answer= 2
![]() |
Buckwalter JA, Einhorn TA, Simon SR: Orthopaedic Basic Science: Biology and Biomechanics of the Musculoskeletal System, ed 2.
Rosemont, IL, American Academy of Orthopaedic Surgeons, 2000, pp 532-545.
![]() |
Rodkey WG: Basic biology of the meniscus and response to injury. Instr Course Lect 2000;49:189-193.
The work of Arnoczky and Warren have shown that about 10-30% of the periphery of the medial meniscus receives a vascular supply from the perimeniscal capillary plexus. The lateral meniscus is about 10-25% vascularized. This peripheral vascular zone is known as the red zone. The ability of the meniscus to heal is dependent on blood supply. The presence of blood supply allows classic wound healing to occur, which is initiated by the inflammatory phase as inflammatory cells infiltrate into the injured area (answer 2). Healing occurs with a cellular fibrovascular scar (5 is wrong). Fibrochondrocytes slowly cause the fibrovascular scar to mature into “normal” fibrocartilage (3 is wrong). Investigators have tried to promote healing in the avascular region with the use of a fibrin clot, but it the intraarticular environment, a naturally occuring fibrin clot from surgical bleeding might be rendered ineffective by synovial fluid dissolution (1 is wrong).
232.03
A 35-year-old carpenter who sustains an injury to his elbow in a fall undergoes emergency surgery. At his 6-week follow-up examination, he reports elbow pain and loss of motion. A radiograph is shown in Figure 81. Management should now consist of
-
intensive physical therapy
-
closed reduction and casting for 4 weeks
-
radial head replacement
-
open reduction and hinged external fixation
back to this question next question
-
total elbow arthroplasty
232.03 Answer= 4
![]() |
McKee MD, Bowden SH, King GJ, et al: Management of recurrent complex instability of the elbow with a hinged external fixator. J Bone Joint Surg Br 1998;80:1031-1036. Nielsen D, Nowinski RJ, Banberger HB: Indications, alternatives, and complications of external fixation about the elbow. Hand Clin 2002;18:87-97.
Traumatic dislocation of the elbow, complicated by associated fractures and/or extensive softtissue injuries, is a challenging problem. Given
![]() |
the propensity of the elbow to develop contracture after injury, one of the objectives of treatment should be to provide sufficient stability to
allow early postoperative mobilization (2 is wrong). Untreated, chronic instability can lead to loss of function and arthritis (1 is wrong). The use of hinged external rotation can be used when concentric stability of the elbow has not been restored by conventional treatment (as in this patient without a radial head and with subluxation of the ulnohumeral joint). Elbow stability depends on both bony congruity and the soft-tissue structures, especially the medial and lateral collateral ligaments, and restoration after injury depends on the repair or reconstruction of these damaged tissues(3 is wrong). The authors found that by treating complex elbow instability with external fixation placed at the time of surgery after first dealing with associated fractures and soft tissue injuries led to good results considering the difficulty in treating this condition. However, they did not suggest this treatment with complex fractures of the distal humerus
233.03
![]() |
What is the most common complication reported in the surgical treatment of intraarticular distal humeral fractures in adults?
-
Infection
-
Ulnar nerve injury
-
Malunion
-
Distal humeral nonunion
-
Brachial artery injury
233.03 Answer= 2
Kuntz DG Jr, Baratz ME: Fractures of the elbow. Orthop Clin North Am 1999;30:37-61.
![]() |
Koval KJ (ed): Orthopaedic Knowledge Update 7. Rosemont, IL, American Academy of Orthopaedic Surgeons, 2002, pp 307-316.
Complications reported in conjunction with treatment of distal humeral fractures include failure of fixation, malunion, nonunion, olecranon osteotomy nonunion, infection, prominent hardware and ulnar nerve neuritis. Malunion and nonunion are uncommon but are associated
![]() |
with high-energy injuries and inadequate fixation (3 and 4 are wrong). Infection ranges from 0-6% and is higher in open injuries (1 is wrong). The incidence of ulnar nerve injury has been reported as 15% and can occur at surgery or caused by implant or post-operative scarring. Transposition of the nerve should be considered if harware is placed distally on the medial column.
234.03
back to this question next question
![]() |
A 12-year-old boy sustains a 100% displaced closed SalterHarris type II fracture through the distal femoral physis while playing football. Neurovascular examination of the extremity is intact. Management should consist of
-
closed reduction and a knee immobilizer
-
closed reduction under anesthesia and percutaneous skeletal fixation
-
open reduction and transphyseal screw insertion
-
skeletal traction for 2 weeks followed by a spica cast
-
a retrograde rigid intramedullary nail
234.03 Answer= 2
Thomson JD, Stricker SJ, Williams MM: Fractures of the distal femoral epiphyseal plate. J Pediatr Orthop 1995;15:474-478.
![]() |
Lombardo SJ, Harvey JP Jr: Fractures of the distal femoral epiphyses: Factors influencing prognosis. A review of thirty-four cases. J Bone Joint Surg Am 1997;59:742-751.
![]() |
The Salter-Harris type II pattern is the most common type of separation at the distal femur and usually occurs in adolescents. Displacement is usually toward the side of the metaphyseal fragment. The incidence of premature growth arrest is significant even with adequate reduction. A nondisplaced separation can be immobilized in a well-molded long leg cast (1 is wrong). General anesthesia often is indicated for closed reduction to decrease muscle spasm and diminish the risk of further injury to the physis. The larger the metaphyseal fragment and the greater the displacement, the less stable the closed reduction is. If the metaphyseal fragment is large enough, treaded pins or screws can be directed transversely across the metaphysis after reduction (3 is wrong). In absence of a large metaphyseal fragment, smooth pins are directed through each side of the condyle to cross proximal to the fracture site. The authors reviewed the incidence of complications with distal femoral epiphyseal injuries which included limb length discrepancy, varus or valgus deformity, limitation of knee motion, ligament laxity, and quad atrophy. They found that prognoses made on the Salter harris classification alone were not reliable, but the development of deformity is related to the degree of initial fracture displacement and the exactness of reduction. There was no mention of retrograde IM nail or skeletal traction as treatment options
235.03
![]() |
To avoid “wrong site” surgery in a patient who is scheduled to undergo knee surgery, the American Academy of Orthopaedic Surgeons advocates which of the following actions?
-
Have the surgeon check on the radiographs
-
Have the surgeon put an “X” on the opposite leg
-
Have the surgeon initial the proper leg 4. Have the circulating nurse confirm the site
5. Keep a sock on the opposite leg.
235.03 Answer=3
![]() |
American Academy of Orthopaedic Surgeons Advisory Statement- WrongSite Surgery. Rosemont, IL, American Academy of Orthopaedic Surgeons, 1998.
![]() |
American Academy of Orthopaedic Surgeons Council on Education: Report of the Task Force on Wrong-Site Surgery. Rosemont, IL, American Academy of Orthopaedic Surgeons, 1998.
The AAOS advisory statement on wrong-site surgery lists effective methods of eliminating wrong site surgery. These include having the surgeon, in consultation with the patient when possible, place his or her initials on the operative site using a permanent marking pen and then operating through or adjacent to the initials. (3 is right). There is no mention of placing an X on the opposite leg (2 is wrong). The surgical team (including the surgeon, anesthesiologist, circulating nurse and scrub nurse) should perform a “timeout” which includes a double-check of the patient’s medical records and the X-rays (1 and 4 are wrong as isolated methods). Unless you’re at the VA, socks are taken off the patient before reaching the OR.
236.03
back to this question next question
![]() |
A 5-year-old boy who sustained a traumatic transtibial amputation 3 years ago now reports pain and tenderness over the lateral aspect of the residual limb. Radiographs reveal terminal overgrowth of the fibula, and prosthetic modifications have failed to provide relief. Treatment should now consist of
-
proximal fibular epiphysiodesis
-
shortening of the residual fibula
-
shortening of the residual fibula and tibia
-
resection of the entire fibula
-
conversion to a knee disarticulation.
236.03 Answer= 2
![]() |
Bowker JH, Michael JW (eds): Atlas of Limb Prosthetics : Surgical, Prosthetic, and Rehabilitation Principles, ed 2. St Louis, MO, Mosby Yearbook, 1992, pp 677-678.
The traumatic transosseous child amputee may experience rapid growth in length of the residual limb to the point where it grows through the skin. This is appositional bone growth and is not related to physeal growth, even though bony overgrowth usually ceases when physeal growth ceases. It is often seen in the humerus, fibula, tibia and femur in that order of relative frequency. This may occur several times during childhood and is easily treated by resection of sufficient bone to allow coverage with an adequate soft-tissue envelope(2 is right).
Proximal epiphysiodesis is
contraindicated since this has no influence on distal appositional bony overgrowth and will lead to unnecessary shortening (1 is wrong). The other answers have treatments that are too aggressive.
237.03
![]() |
Which of the following factors is associated with an increased risk for hip fracture?
-
Age less than 50 years
-
Smoking
-
Caucasian male
-
Rural dwelling
-
Obesity
237.03 Answer= 2
![]() |
Chevalley T, Herman FR, Delmi M, et al: Evaluation of the age-adjusted incidence of hip fractures between urban and rural areas: The difference is not related to the prevalence of institutions for the elderly. Osteopros Int 2002;13:113-118.
![]() |
Koval KJ (ed): ): Orthopaedic Knowledge Update 7. Rosemont, IL, American Academy of Orthopaedic Surgeons, 2002, pp 407-416.
Hip fractures in patients younger than 50 are rare and usually caused by highenergy trauma (1 is wrong). Women are two to three times more likely to be affected than men, and the incidence in white women is 2-3 times higher than reported for black and Hispanic women (3 is wrong). Risk factors include urban dwelling (4 is wrong), smoking (2 is right), physical inactivity, previous hip fracture, significant weight loss in midlife (5 is wrong), senile dementia, postmenarche, institutionalization, osteoporosis, and inadequate dietary calcium intake.
238. The threshold of slowly adapting skin receptors is best associated with:
back to this question next question
-
Static two-point discrimination
-
Moving two-point discrimination
-
Semmes-Weinstein monofilaments
-
Vibrometry
-
electromyelography
-
3 – Semmes-Weinstein monofilaments
Slowly adapting skin receptors (e.g., Merkel discs and Ruffini cylinders) are those that fire throughout a stimulus as opposed to fast adapting ones that fire only upon initiation or termination of a stimulus. Use of Semmes-Weinstein monofilaments was determined in the quoted articles to be the best way to ascertain the threshold of the slowly adapting fibers, superior to either moving or static two-point discrimination.
Vibration sensation is accomplished by the fast adapting receptors. Electromyelography is used to test muscle response to nerve stimulation.
-
Which of the following designs of cementless femoral components has been associated with an unacceptable incidence of distal osteolysis following total hip arthroplasty?
-
Circumferential proximal only coating
-
Proximal noncircumferential porous coating
-
Uncoated stems
-
Fluted stems
-
Fully coated stems
239. 2- proximal noncircumferential porous coating
Polyethylene debris from the bearing surface causes osteolysis by activating macrophages. The debris can be carried by joint fluid to sites remote from their origin. The bone-implant interface is protected by physical barriers to the flow of synovial fluid; therefore, implants with some shielding or impediment of flow to the load bearing surfaces have less osteolysis. Patchy or noncircumferential coated stems have the highest incidence of osteolysis because they do not have a circumferential barrier to the transport of polyethylene debris
(along smooth, noncoated stem) to the bone-implant interface. 240. What is the most common location for skeletal metastasis from
adenocarcinoma?
-
Femur
-
Humerus
-
Hand
-
Spine
-
Pelvis
back to this question next question
-
-
4- Spine
Spine, pelvis and ribs (in that order) are most likely sites of metastasis for adenocarcinoma. There are several reasons for this including: the axial skeleton is seeded more than the appendicular skeleton, partly due to the persistence of red bone marrow in the former; Batson's vertebral venous plexus allows cells to enter the vertebral circulation where sluggish blood flow is more conducive to tumor survival.
-
A 32-year-old man who fell from a second story balcony (in New
Orleans?) is hemodynamically stable and neurologically intact, but he reports
foot pain. Pre- and postreduction radiographs are shown in fig 82A-D. Management should consist of
-
Immediate application of
a short leg cast
-
Splinting, followed by short leg cast
-
Percutaneous pinning
-
Percutaneous screw fixation
-
Open reduction internal fixation
back to this question next question
-
5 – open reduction internal fixation
The patient has a Lisfranc injury with obvious disruption of the normal anatomical alignment of the lateral boarders of the first metatarsal base with the medial cuneiform and the medial boarders of the second metatarsal base and the middle cuneiform in the AP view. Postreduction views show an unstable injury pattern with greater than 2mm shift in alignment. Surgical stabilization with open reduction, internal fixation in indicated to evaluate the whole Lisfranc joint complex. Nonoperative and percutaneous techniques are not therefore recommended.
-
A 21-year-old woman lacerated the peroneal nerve at the level of the fibular head in a skiing accident 24 mos ago. Because she does not want to continue to use an
ankle-foot orthosis, treatment should now consist of
-
Transfer of the Achilles tendon to the peroneus longus
-
Transfer of the FHL to the medial cuneiform
-
Interosseous transfer of the posterior tibial tendon to the lateral foot
-
Entensor hallucis tendon transfer
-
Splint anterior tibial tendon transfer
-
-
-
-
3- interosseous transfer of the posterior tibial tendon to the lateral foot
The injury is of the common peroneal nerve and thus the patient will complain of equinovarus deformity on the affected side. Treatment of chronic common peroneal nerve palsy is a transfer of the tibialis posterior tendon to the lateral aspect of the foot. This can restore dorsiflexion and eversion of the foot. The other treatment options in this question either do not achieve these movements, are less mechanically advantageous or destabilize ankle balance.
-
A 20-year-old man who is an avid surfer has had left medial knee pain and numbness over the anteromedial leg for the past 3 wks. Exam reveals no effusion, no joint line tenderness, active ROM from o -135 deg. And marked tenderness about the medial
calf
proximally. McMurray’s test worsens the pain. What anatomic structure is ost likely responsible for these symptoms?
-
Medial meniscus
-
Medial collateral ligament
-
Saphenous nerve
-
Peroneal nerve
-
Popliteal artery
-
3- saphenous nerve
back to this question next question
Saphenous neuritis can be caused from repeated trauma or entrapment in the adductor canal. You don’t really have to know the mechanism of chronic injury to answer this question; it is essentially an anatomy question. The question is pointing to a neuritis or neuropathy as there are no joint symptoms per-se so medial meniscus and popliteal artery is out. Medial collateral ligament is not ANTEROmedial. The peroneal
n. is lateral so that leaves the saphenous nerve, which ennervates the skin overlying the medial knee and calf proximally.
-
A 42-year-old man has severe shoulder pain and is unable to raise his arm after playing softball. He reports no
history of shoulder problems. Examination reveals dense weakness of abduction, enternal rotation and elbow extension. Which of the following studies would best aid in diagnosis?
-
MRI with gadolinium
-
Bone scan
-
Double-contrast arthrography
-
Electromyography
-
Cervical myelography
-
-
-
4- electromyelography
The case describes an acute injury resulting in dense weakness involving C5, C6-7, C7-T1. As this is most likely a nerve injury, i.e., brachial plexopathy or acute brachial neuritis, electromyelography is indicated to confirm the diagnosis.
-
Which of the following changes takes place in skeletal muscle as the result of immobilization or disuse?
-
Hypertrophy
-
Increased weight
-
Increased fatigability
-
Increased ability to use fats in aerobic metabolic pathways
-
Decreased strength per cross-sectional area
-
3- increased fatigability
Disuse atrophy results in decreased muscle mass and decreased muscle cross-sectional diameter. This results in increased fatigability. Ability to use fats in aerobic metabolic pathways is increased or the same and strength per crosssectional area is the same or decreased (due to decreasing recruitment of muscle fibers).
-
Control of refractory pain in diffuse blastic metastatic disease can be accomplished with systemic IV administration of
back to this question next question
-
Iodine
-
Strontium
-
Tamoxifen
-
Gold
-
Selenium
-
-
2- Strontium
Strontium-89 emits low-energy beta particles, a type of radiation that penetrates only about 3mm into tissue. As a result, its bone marrow toxicity is low. Clinical trials have shown that strontium-89 is well tolerated and highly effective for the palliation of painful blastic bone metastases. Iodine can be used for treatment of Graves disease and some thyroid tumors; tamoxifen is an estrogen-antagonist; gold can be used as an anti-inflammatory; Selenium is an antioxidant.
-
Osteomyelitis is a unique disorder in the neonate because of which of the following findings?
-
-
Epiphyseal extension from the metaphysis, multiple site involvement and the presence of Staph aureus
-
Epiphyseal extension from the metaphysis, multiple site involvement and the presence of group B strep
-
Epiphyseal extension from the metaphysis, single site involvement and the presence of group B strep
-
Metaphyseal and single site involvement and the presence of Staph aureus
-
Metaphyseal and single site involvement and the presence of group B strep
247. 2- Epiphyseal extension from the metaphysis, multiple site involvement and the presence of group B strep
Epiphyseal spread- in neonates the periosteum is loosely applied to the cortical surface, which can allow bacteria and purulence to track upwards between these surfaces and extend over the length of the bone or spread from the metaphysis to epiphysis through ample blood supply feeding the active growth plate. Hematogenous spread allows for seeding of multiple sites. Group B strep is the most common causative bacteria in community-acquired disease whereas Staph is indeed most common in nosocomial infections.
back to this question next question
248 – Item Deleted
back to this question next question
249.03
![]() |
Stabilization of femoral shaft fractures in the first 24 hours after injury has been shown to decrease complications in which of the following patient groups?
-
Patients with diabetes mellitus
-
Patients with an Injury Severity Score of greater than 18
-
Patients with a head injury and elevated intracranial pressure
-
Pediatric patients
-
All patients
249.03 The correct answer is 2.
250.03
-
Prevent digital thrombosis
-
Debride avascular tissue 3. Increase arterial inflow
-
-
Treat venous congestion
-
Diminish ischemic pain
250.03 The correct answer is 4
![]() |
There is a reference article for this from 1989 but you don’t need to look it up. Leeches do contain an anticoagulant called hirudin but this does not prevent digital thrombosis, it just thins the blood in the local area so it flows better for sucking. Leeches do not debride so 2 is wrong.
Avascular tissue can be monitored or debrided after replant, but if the tissue appears avascular, you should make sure your arterial anastomosis is working. Leeches have no effect on arterial inflow, problems with arterial inflow have to be treated by a repeat trip to the OR. Leeches are used to decrease venous congestion. They remove blood from the digit that is not being removed by incompetent veins until collateral venous drainage develops. This is the correct answer. Leeches have no effect on ischemic pain. As noted earlier, if the problem is ischemia(lack of arterial inflow), get out of bed and go back to the OR.
251.03
![]() |
The process of thrombosis (as indicated by elevation of markers of thrombotic generation and fibrin formation) is initiated at what point during total hip arthroplasty?
back to this question next question
-
1- Hip dislocation
-
2- Hip reduction
-
3- Acetabular reaming
-
4- Femoral head removal
-
5- Femoral canal preparation
251.03 The correct answer is 5.
The reference on this is an article in CORR in 1995 entitled Thrombogenesis during total hip arthroplasty. This won the Charnley award, so it’s pretty good.
This is the basis of the heparin protocol which Barrack uses. In his protocol, you give extra heparin before canal preparation. In this study they measured levels of thrombogenic factors during certain aspects of THA. Prothrombin F1.2, thrombinantithrombin complexes, fibrinopeptide A and D-dimer did not increase during neck osteotomy or acetabular insertion, but did increase during femoral component insertion. These thrombogenic factors were higher with cemented than noncemented components. This question, however, is a little tricky. The key is the
markers of thrombosis, because you see significantly decreased oxygen tension with hip location(in cemented components, but not press-fit) also, but you don’t see the increased factors. This study proved that the administration of 1000 units of unfractionated heparin before insertion of the femoral component blunts the rise of fibrinopeptide A and decreases thrombogenesis. The conclusions of this study were: 1) greatest risk of activation of clotting cascade is with femoral component insertion. 2) risk is higher with cemented components. 3) measures to prevent dvt should begin intra-op instead of post-op
252.03
-
Sacroiliitis
-
Aortitis
-
Uveitis
-
Peripheral joint synovitis
-
Decreased chest expansion(less than 2 cm)
-
252.03 The answer is 1
![]() |
Ankylosing Spondylitis is an asymmetric large joint arthritis that typically starts in adolescent boys. Symptoms include arthritis, heel pain, eye symptoms, hip and back pain. Back pain is the cardinal symptom but may develop late. HLA B27 is + in 90-95% of AS patients but is also present in Reiters so this is not a good screen. The progression of symptoms by Xray is bilateral sacroiliitis, followed by joint space narrowin, later frank ankylosis, and late vertebral scalloping. Sacroiliitis is almost always the initial manifestation. Uveitis is fairly rare, and aortitis can occur but usually later.
253.03
back to this question next question
![]() |
Which of the following is the best predictor of an increased risk for foot complications in a patient with diabetes mellitus?
-
Diabetes type (type I versus type II)
-
Elevated level of HBA1C
-
Use of insulin
-
Associated renal dysfunction (dialysis or transplantation)
-
History of a foot ulcer
253.03 The answer is 5
. Reference is OKU 2 and an article in Foot Ankle Int in 1999. Foot infection is the most common reason for hospitalization of diabetic patients in the U.S. Ulcers of the foot lead to infection, sepsis, and amputation. Prophylactic foot care decreases morbidity, utilazion of resources , amputation and premature death. None of the other options have as strong a correlation with future complications as history of ulcer. Once diabetic patients have developed ulcers they are at risk for all of the above complications. Type of diabetes nor control (as indicated by A1C) do not necessarily cause ulcers. (although uncontrolled diabetes can lead to earlier onset of peripheral neuropathy and predispose to ulceration)
254.03
-
Oblique
-
Radial
-
Crossing
-
Circumferential
-
Vertical
-
254.03 The answer is 4
![]() |
Menisci are arranged in several layers. The Superficial layer consists of mesh-like fibers oriented primarily radially. The Surface layer has irregularly aligned collagen bundles and the deep layer has parallel circumferential fibers. The circumferential fibers help dissipate hoop stresses. There is no such thing as crossing, vertical or oblique fibers. The radial fibers assist in allowing the meniscus to expand under compressive forces.
255.03
![]() |
A 42 year old woman with a 20 year history of type I diabetes mellitus has had pain and stiffness in her dominant shoulder for the past 3 months. Initial management should consist of
back to this question next question
-
A subacromial corticosteroid injection
-
Manipulation under anesthesia
-
Physical therapy
-
Arthroscopic capsular release
-
Open capsular release
255.03 The answer is 3
256.03
-
Experimental
-
Observational
-
Interventional
-
Descriptive
-
Case-controlled
-
256.03 The answer is 2
![]() |
You should immediately eliminate experimental, interventional and descriptive as these are not actual types of studies. A case controlled study requires at least 2 groups of patients and is a prospective study, this was a retrospective study. Observational studies are retrospective studies which assess risk factors and rates of disease in a select group of patients.
257.03
![]() |
The parents of a 7-year-old girl report the development of multiple nontender subcutaneous nodules over both distal tibiae for the past few months. Incisional biopsy results reveal granuloma annulare. What is the best course of action?
-
Reassure the family that the nodules are selflimiting
-
Surgically remove all nodules
back to this question next question
-
Administer steroids
-
Administer low-dose irradiation
-
Administer low-dose methotrexate
257.03 The answer is 1
![]() |
Subcutaneous granuloma annulare is a disease of childhood in which deep dermal or subcutaneous nodules lie on or near the periosteum and are distributed on the feet, lower legs, fingers, hands, forearm, scalp, and forehead. GA is a self-limited cosmetic disease without any systemic medical complications. On rare occasions it may involve fascia and tendons and may cause sclerosis, lymphedema, and deformities such as joint ankylosis. GA has been linked to DM, necrobiosis lipoidica diabeticorum and rheumatoid nodules. Females predominate 2:1, mean age at presentation in kids is 4.
258.03
![]() |
A 14-year-old boy who plays football reports that he is unable to complete the required laps during the second week of practice because of right knee pain. Examination reveals a positive patellar grind test and tenderness and swelling over the tibial tuberosity. What is the best course of action?
-
Discontinuation of all sports for the season
-
Crutches with no weight bearing on the right side
-
Knee immobilizer
-
Nonsteroidal anti-inflammatory drugs and quadriceps stretches
-
Arthroscopy
258.03 The answer is 4
![]() |
This patient has an apophyseal injury of the proximal tibia. The reference is out of (YIKES!) American Family Physician in 1995. Apophyseal injuries are unique to the adolescent athlete. They cause inflammation at the site of major tendinous insertion onto a growing bony prominence. Presentation is typically periarticular pain associated with repetitive microtrauma.(running in our patient) They have different names based on where they occur. Sever’s disease is posterior calcaneus, Sindig-Larsen-Johansson is inferior patella, medial epicondylitis is humeral medial condyle, apophysitis of the hip is iliac crest or ischial tuberosity, and last but not least is Osgood-Schlatter as this patient has. Treatment is conservative with rest, ice, compression, elevation, NSAID’s, modification of activity, and exercises for increased flexibility and strengthening.
259.03
![]() |
A new scoring system to predict the need for amputation following high-energy lower extremity trauma is being developed. When this system is applied, a score above a threshold level will result in the decision to perform an amputation. To avoid unnecessary amputations, it is most important this scoring system have
-
high sensitivity
-
low sensitivity
-
high specificity
-
low specificity
-
low negative predictive value
259.03 The answer is 3
back to this question next question
![]() |
This is a statistics question. Sensitivity is the ability of a test to detect that which is truly positive. Specificity is the ability of a test to detect that which is truly negative. Positive predictive value is the chance that a condition truly exists if the test is positive. Negative predictive value is the chance that the condition truly does not exist when the test is negative. This study should have high specificity. It should be able to detect which patients truly don’t need an amputation.
03.260
An otherwise healthy 76-year old woman who sustained a two-part intertrochanteric hip fracture in a fall was treated with reduction and placement of a 135-degree sliding hip screw with a twohole side plate. Examination following the procedure reveals an excellent reduction, and the tip of the screw is 1 cm from the apex of the femoral head on both the AP and lateral views. These findings will most likely result in
-
varus deformity with screw cutout
-
failure at the two-hole side plate
-
non-union
-
aseptic necrosis 5 uneventful healing
back answer
Question 03.260
Answer = 5
Femoral Neck: Bimodal age distribution. Garden I&II (impacted/non-displaced) and Garden III&IV (displaced). Lag screws are the preferred treatment in the young and a prosthesis in the elderly. In the nondisplaced fx, non-union and AVN occur <10%. In the displaced fx, those complications climb to 33%. In the displaced fracture, close reduction should be attempted before open reduction. If opening is required, an anterolateral incision is preferred. Acceptable reduction may have up to 15º varus and 10º AP angulation. Reduction is the key to success. The preferred prosthesis is a cemented endo. Unipolar are advocated over bi-polars secondary to lower price and similar clinical outcome. THA following fx is advocated only if there is pre-existing acetabular arthritis
Intertrochanteric fracture/basicervical fracture: Same demographics as femoral neck. Stability is key to classification. An intact posteromedial cortical buttress indicates stability. Loss of this, subtroch extension, and reverse obliquity are unstable patterns. Stable fractures are treated with a sliding hip screw construct. 135-150º plates are available. Placement of the screw to within 1cm of subchondral bone is paramount. Intra medullar devices can be just as affective but have a higher incidence of thigh pain. This is the device of choice for a subtroch, reverse obliquity, and unstable patterns of injury.
Reference(s)
Koval KJ (ed) OKU 7. AAOS. 2002. pp407-416
Baumgaetner MR et al.: Awareness of the tip-apex distance reduces failure of fixation of trochanteric fractures of the hip. JBJS Br 1997;79:969-971
Bolhofner Br et al. Results of intertrochanteric femur fractures treated with a 135-degree sliding screw with a two-hole side plate. J Orthop Trauma 1999;13:5-8
03.261
Which of the following factors is LEAST important in determining a patients’
perception of the results after lower extremity limb amputation?
-
Residual limb comfort
-
Residual limb length
-
Contralateral limb condition
-
Participation in recreational activities
back to this question next question
-
Social factors
-
back answer
Question 03.261
Answer = 2
In a study of 148 lower extremity amputees, the residual limb length (transtibial, transfemoral, Symes, etc.) did not have ANY correlation with a successful amputational outcome. The remaining 4 choices proved to have some impact on ultimate outcome. In unsuccessful amputees, look for mental disorders such as anxiety, PTSD, sexual dysfunction. On a side note, remember the key numbers for successful amputation: Albumin > 3.5 g/dL, total lymphocyte >1500 /mm3 , ABI > .5, toe pressures > 30mm Hg
Reference(s)
Koval KJ (ed) OKU 7. AAOS. 2002. p127-137
Matsen Sl. Et al: Correlation with patients’ perspectives on the results of lower-extremity amputation. JBJS Am 2000:82:1089-1095
back to this question next question
03.262
An 18-year-old female musician cuts the tip of her non-dominat finger, transversely removing about 25% of the distal nail bed and the distal tuft of the distal phalanx. The patient brings along the tip. Management should consist of
-
defatting the tip and suturing it in place as a composite graft
-
a cross finger flap to the tip
-
a volar V-Y advancement flap
-
microvascular replantation 5. bone shortening and primary closure
back answer
Question 03.262
Answer = 3
The primary goal of treatment of an injury to the fingertip is a painless fingertip with durable and sensate skin. For injuries with soft-tissue loss and no exposed bone, healing by secondary intention or skin grafting is the method of choice. When bone is exposed and sufficient nail matrix remains to provide a stable and adherent nail plate, coverage with a local advancement flap should be considered. The V-Y advancement is useful when the direction of amputation is transverse or dorsal oblique. See figures below
![]() |
![]() |
![]() |
If the angle of amputation does not permit local flap coverage, a regional flap (cross-finger or thenar) may be indicated. If the amputation is more proximal or if the patient is not a candidate for a regional flap because of advanced age, osteoarthritis, or other systemic condition, shortening with primary closure is preferred. Composite reattachment of the amputated tip may be successful in young. The outcome of nail-bed injuries is most dependent on the severity of injury to the germinal matrix.
![]() |
![]() |
Reference(s)
J Am Acad Orthop Surg 1996;4:84-92
Atasoy E,et al. Reconstruction of the amputated finger tip with a triangular volar flap, A new surgical prcdure. JBJS Am 1970, 52;921-926
Kutler W; A method of repair of finger amputations. Ohio State Medical Journal 1944; 40:126-145
back to this question next question
03.263
Patients with metal-on-metal total hip replacements, when compared to patients with metal-on polyethylene total hip replacements, have been shown to have
-
Increased serum and urine metal concentrations
-
Increased amount of osteolysis
-
Greater range of motion
-
A lower rate of loosening 5 An increased incidence of cancer
back answer
Question 03.263
Answer = 1
Lets talk about plastic first. Regular UHMWPE has a survival rate of 10-20 years (80-90%). But when gamma irradiated without air during sterilization, it has the potential to highly cross-link which will improve its wear properties. Most cross-linking is done at high temperatures. If oxygen is present during the irradiation process, instead of cross-linking, the plastic will form a carbonyl group (oxidation ….BAD). Even when done correctly, there is a downside to cross-linking: it decreases the rest of the plastics’ mechanical properties (lower yield strengths, ultimate tensile strength, fatigue strength). In the hip, there are low contact stresses so surface wear predominates (perfect for highly cross-linked). On the otherhand, the knee has high contact areas so highly cross-linked plastic usually doesn’t have the fatigue properties needed. Typically the counter surface is a cobalt chrome or ceramic coated (oxinium ) metal
.
Modern metal on metal typically has performed very similarly to plastic, but different aspects of its design are crucial to its success. The key words are clearance, roughness, and carbon content. Clearance is the space between the ball and shell. Too small and it locks up…too large and the contact stresses are way too high. Roughness will also accelerate wear. And as the question infers, the metal concentrations found in the body are higher.
Ceramic have excellent wear characteristics but can have catastrophic fractures or wear particles Reference(s)
Koval KJ (ed) OKU 7. AAOS. 2002. p39-45
back to this question next question
03.264
A 27-year old man injures his neck in a motor vehicle accident. Neurologic examination reveals intact light touch and pinprick sensation. His deltoids and biceps are 5/5, and he has 3/5 strength in his left hand and 2/5 strength in the right hand. He has 4/5 strength throughout his left lower extremity and 4+/5 on the right side. His neurologic status could most accurately be classified as
-
complete cord injury
-
posterior cord syndrome
-
anterior cord syndrome
-
central cord syndrome
-
Brown-Sequard syndrome
-
back answer
Question 03.264
Answer = 4
-
Complete cord injury implies unequivocal absence of motor or sensory function distal to injuryin absence of spinal shock; w/ complete injuries, an improvement of one nerve root level can be expected in 80% of patients, and approximately 20% will recover 2 additional function levels; there is also evidence that operative decompression can also allow recovery of an additional nerve root level;An incomplete lesion is present when there is any distal sparing of motor or sensory function along with sparing of perirectal sensation
-
Posterior cord syndrome is very rare. Its loss of dorsal column function (deep pressure and proprioception). Prognosis is good and motor function is preserved. Walking is extremely difficult or impossible because of persistent impairment of proprioception
-
Anterior cord syndrome- second most common spinal cord injury. Its defined as loss of neurologic function in anterior 2/3 of the spinal cord (related to vascular insufficiency). The affected regions are spinothalamic (pain and temperature) and corticospinal (motorfunction). There is sparing of the posterior columns (position sense, proprioception, vibration, deep pressure). There is greater motor loss in the legs than the arms. The mechanism of injury is flexion-compression. This injury has the worst prognosis
-
Central cord syndrome- most common incomplete spinal cord injury. Most often found in the elderly with pre-existing cervical spondylosis who sustain a hyperextension injury. Greater loss of function to the upper extremity than the lower extremity with variable sensory sparing. This injury has a fair prognosis
-
Brown-Sequard syndrome damages half of the cord. It causes ipsilateral motor loss and position/proprioception loss and contralateral pain and temperature loss (usually two levels below the insult). This injury has the best prognosis
Reference(s)
McGuire RA: Physical examination in spinal trauma. Spine Trauma. Philadelphia, PA WB Saunders. 1998 p16-27
Sibtag VKH. Et al. Controversies in spinal cord syndromes in Garfin SR, Northup BE (eds) Surgery for Spinal Cord Injuries. New York, NY
Raven Press pp 15-32
back to this question next question
03.265
What is the most common complication associated with open surgical repair of an Achilles tendon rupture?
-
Infection
-
Neurologic Injury
-
Loss of motion
-
Rerupture
-
Skin Healing Problems
-
back answer
Question 03.265
Answer = 5
Operative treatment is the more popular method of treating an acute Achilles rupture in the younger active patient. In reality, with new functional bracing techniques, the rates of re-rupture are similar (10%) if a conservative non-opeartive route is chosen. Using non-operative techniques, if a diastastis remains with the foot in 20 degress of equinus, operative repair is indicated. The main complication of an open repair
(including percutaneous), are skin complications followed by skin necrosis/infection and nerve injury (sural ).
Reference(s)
Wong J, Barrass V, Maffulli N: Quantitative review of operative and non-operative management of Achilles tendon ruptures. Am J Sports Med 2002;30:565-575
Lo IK, Kirkley A, Nonweiler B: Operative versus nonoperative treatment of acute Achilles tendon ruptures. A quantitative review. Clin J Sports Med 1997;7:207-211
back to this question next question
03.266
An 18-year-old basketball player sustains a moderate inversion ankle sprain.
Early functional recovery is best achieved with
-
a prolonged period of no weight bearing
-
immediate muscle stimulation
-
immobilization followed by strengthening
-
early use of ultrasound and electrical stimulation 5- strength and proprioception training
back answer
Question 03.266
Answer = 5
Inadequate rehabilitation most often occurs after a grade 3 ligamentous injury. Motion may be painful, and stiffness is common, although the radiographs are normal. Many athletes return to sports before they are fully rehabilitated and often incur a reinjury or an additional injury.
Examination demonstrates loss of range of motion, such as limited dorsiflexion or a plantar flexion contracture. Atrophy of the lower-leg muscles is common. Initial treatment should include a short period of ankle protection by brace, bandage, or tape and early mobilization and weight-bearing.
Rehabilitation exercises are the most important step in the treatment process, with the goal of reestablishing ankle range of motion, muscle strength, and neuromuscular control. Emphasis should be placed on strength training of the peroneal muscles, the anterior and posterior muscles, and the intrinsic muscles of the foot. Proprioceptive training on an ankle tilt board should be combined with increasing agility and sports skills training. If functional treatment of an acute injury fails, surgery may be necessary
Reference(s)
Koval OKU 7 2002
Taker SB, et al. The prevention of ankle sprains in sports: A systematic review of the literature. Am J Sports med 1999:27:753-760
back to this question next question
03.267
A 14-year-old volleyball player reports that her shoulder dislocated and spontaneously reduced while she was serving a volleyball. In the following 6 months, she had three similar episodes. Which of the following conditions is most likely present?
-
Capsular laxity
-
Hill-Sachs lesion
-
Anterior inferior labral tear
-
Superior labral anterior posterior tear
-
Rotator cuff tear
back answer
Question 03.267
Answer = 1
Multidirectional instability of the shoulder is a complex entity . Affected patients have global (anterior, inferior, and posterior) excessive laxity of the glenohumeral joint capsule and a rotator interval capsule defect. The onset of symptoms is frequently related to atraumatic events. The chief complaint is more often relateed to pain than to instability per se. Symptoms are mostly experienced within the midrange of glenohumeral motion.
These factors may include subtle losses of strength and/or neuromotor coordination of the rotator cuff and scapular stabilizing muscles, defective proprioceptive responses, and the absence of a limited joint volume. Most patients can be successfully treated nonoperatively with a specific exercise program. If a 6-month trial of nonoperative management fails, the patient is a candidate for surgical reconstruction. The most time-honored procedure is an open inferior capsular shift, which corrects excessive global laxity of the capsule and the rotator interval defect.
Most patients with recurrent instability can be classified into traumatic and traumatic groups. The characteristics of each group can be remembered with use of the mnemonic devices which have been derived as follows: TUBS and AMBRII. Instability related to Traumatic event presents as a Unidirectional instability problem, usually involves a Bankart lesion, and frequently requires Surgery to achieve stability. Instability that arises Atraumatically occurs in patients prone to Multidirectional instability who have Bilateral excessive laxity; this instability usually responds to a Rehabilitation program that emphasizes strengthening of the rotator cuff, but when operative intervention is undertaken, it must tighten the Inferior capsule and the rotator Interval capsule. Most patients in whom MDI is diagnosed are young adults in their third decade (range, teenage to middle age). The occurrence of bilateral instability is not infrequent. Symptoms associated with MDI are pain, varying degrees of instability, and transient neurologic symptoms in the affected extremity.
Reference(s)
Burkhead WZ, Rockwood CA. Treatment of instability of the shoulder with an exercise program. JBJS Am 1992; 74; 890-896 Matsen FA III, Thomas SC etal. Anterior glenohumeral instability, in the shoulder. Philadelphia, PA, WB Saunders. 1990 pp 526-622. Schenk TJ, Brems JJ. Multidirectional instability of the shoulder: Pathophysiology, diagnosis and management. JAAOS 1998;6:65-72
back to this question next question
03.268
The apical ectodermal ridge is an embryonic structure necessary for growth and development of the
-
anterior neuropore
-
neural tube
-
limbs
-
rostral sclerotomes
-
ventral gut
back answer
Question 03.268
Answer = 3
Apical ectodermal ridge is an ectodermal thickening that runs along the distal margin of the limb bud (craniocaudally). It is essential for limb bud development. Removal of AER leads to failure of limb growth & development (phocomelia). The AER determines time- dependent differentiation of the
proximal- distal limb bud mesoderm. Early mesoderm forms proximal limb segment while late mesoderm forms distal limb segment
Reference(s)
Diestz Fr et al. Embryology and development of the musculoskeletal sytstem, in Morrissy RT: Lovell and Winter’s Pediatric Prthopaedics ed
5 Philadelphia PA
Lippincot and Williams 200 p 1-31
Sifakis S et al Distal limb malformations: Underlying mechanisms and clinical associations. Clin Genet 2001 ; 60:165-172
back to this question next question
03.269
A patient with ankylosing spondylitis is scheduled to undergo total hip arthroplasty. Which of th following is an important preoperative consideration?
-
Flexion-extension cervical spine radiographs
-
Radiation therapy consultation
-
MRI of the spine
-
Urology consultation
-
Discontinuation of methotrexate
back answer
Question 03.269
Answer = 2
Anylosing Spondylitis, formerly known as Becheterew’s disease, is a chronic systemic disease that primarily affects the spine and may lead to stiffness of the back. The joints and ligaments that normally permit the back to move become inflamed. The joints and bones may grow (fuse) together. A gene called HLA-B27 occurs in 90 percent of those with ankylosing spondylitis. Most common symptoms are chronic pain and stiffness in the lower back, which usually starts where the lower spine is joined to the pelvis, or hip.
The risk of severe HO after THA is higher in patients who have developed HO after previous surgery, in men with hypertrophic osteoarthrosis and in patients with ankylosing spondylitis or diffuse idiopathic skeletal hyperostosis. Other possible predisposing factors include extensive intra operative bleeding with hematoma, post-operative infection, operative approach with trochanteri costeotomy, or dislocation of the prosthesis during the first post-operative week. While THA failure is approximately 1% in the general population, it has been reported as high as 10% in the AS population.
HO has been reported as high as 77% following THA in the AS population. NSAIDS and post-operative radiation (500cGy within 96 hours) should be considered in this population undergoing THA.
Reference(s)
Joshi AB et al: An analyis of 181 hips. J Arthoplasty 2002; 2002; 17:427-433
Pellici PM, et al: OKU Hip and Knee Reconstruction 2. Rosemont IL, AAOS 200 3-6
back to this question next question
03.270
Orthotripsy (shock wave therapy) has been approved for the treatment of
-
plantar fibramatosis
-
sinus tarsi syndrome
-
rheumatoid enthesopathy of the Achilles tendon
-
cacific tendonitis of the heel
-
chronic proximal plantar fasciitis
-
back answer
Question 03.270
Answer = 5
In 2000, the FDA approved the use of shock wave therapy for plantar fasciitis. Plantar fasciitis is the most common cause of chronic infereomedial heel pain. The plantar fascia has three bands (lateral, medial, central) with the central one most likely being diseased with a diagnosis of fasciitis. Patients require anesthetic (ankle block) to receive this loud, painful treatment.
Reference(s)
Ogden JA et al, Shock Wave therapy (Orthotripsy) in musculoskeletal disorders. Clin Orthop 2001; 387;17:427-433
Alvarez R: Preliminary results on the safety and efficacy of the OssaTron for treatment of plantar fasciitis. Foot Ankle Int 2002;23 197-203 back to this question next question
03.271
Which of the following factors has been shown to contribute to poor results after anatomic reduction of posterior wall acetabulum fractures?
-
Marginal articular impaction
-
Involvement of the weight-bearing dome
-
Delay in reduction of hip dislocation of greater than 12 hours
-
Size of wall fragment
-
Patient age of less than 40 years
back answer Question 03.271
Answer = 3
Only 30% of posterior-wall acetabular fractures involve a single large fragment. The majority are multifragmentary or have areas of impaction. Unsatisfactory clinical results occur in more than 80% of patients treated nonsurgically. Operative management usually offers the best chance of preserving long-term joint function, but only if an anatomically reconstructed acetabulum can be achieved without complication. The keys to surgical success include maintaining the viability of the fracture fragments and the femoral head itself, using bone grafts and buttress plating to support elevated and comminuted fragments, and protecting the neurovascular structures at risk.
Complications can include sciatic nerve injury (incidence, 3% to 18%), heterotopic ossification (7% to 20%), and osteonecrosis of the femoral head (5% to 8%). Despite the relative simplicity of this acetabular fracture, unsatisfactory outcomes after surgical repair of the posterior wall occur in at least 18% to 32% of cases, results that are worse than for most of the other more complex acetabular fracture patterns.
Reference(s)
Moed BR, et al. Results of operative treatment of fractures of the posterior wall of the acetabulum. JBJS AM 2002:84:752-758
Matta JM: Fractures of the acetabulum: Accuracy of reduction and clinical results in patients managed operatively within three weeks after the injury. JBJS Am 1996;78:1632-1645
back to this question next question
03.372
During total knee arthoplasty with the trial components in position, the knee can be fully extended but is tight in flexion. Which of the following actions would improve the flexion-extension mismatch?
-
Resect more distal femur
-
Resect more proximal tibia
-
Downsize the femoral component
-
Downsize the tibial component
-
Increase flexion of the femoral component
-
back answer
Question 03.272
Answer = 3
The article used to explain this answer actually compares two different referencing systems for making the AP cuts on the femur (a posterior and anterior referencing system). This becomes important when the size of the femur does not match the size of implants and a decision between larger or smaller components is needed. I’ll explain each referencing system
Anterior referencing system. This system uses the anterior cortex as its reference to make the distal cuts. The size of the component then determines your posterior condylar cuts. If your femur is in between sizes, usually the smaller component is chosen. Because the smaller size is chosen, you’ll cut some more condylar bone to make it fit. This will make your knee a little laxer in flexion. Usually not a problem if using a cruciate retaining knee
ADVANTAGES: more normally reproduces extensor mechanism, avoids notching femur DISADVANTAGES: Can create laxity in flexion if component small
Posterior referencing system. This system uses the posterior condyles as its reference and the distal and posterior cuts are made equally. The size of the component determines the anterior cut. If in between sizes, a larger one is usually chosen. This will make the knee a little tight in flexion but will avoid NOTCHing the femur
ADVANTAGES: More balanced flexion/extension spaces
DISADVANTAGES: Can create overstuffing the extensor mechanism hindering flexion if larger size chosen, NOTCHING is smaller size chosen
Reference(s)
Corey “Ya know what I mean” Burak
Ries MD. AP Positioning of the Femoral Component in TKA. Seminars in Arthroplasty. Vol 10, No 4, pg 219-224. 1999
back to this question next question
03.273
In an athlete who has full, painless range of motion and a normal neurological examination, which of the following is considered an absolute contraindication to participation in a collision sport such as football?
-
A history of c4-C5 anterior discectomy and fusion
-
A history of spinal cord neurapraxia with transient quadriparesis that has now resolved
-
Os odontoideum
-
Congenital C5-C6 fusion
-
Congenital Stenosis
back answer Question 03.273
Answer = 3
The combination of congenital stenosis with instability, disk disease (bulge or herniation), degenerative change (osteophytes), MR imaging evidence of cord abnormality, neurologic findings lasting longer than 36 hours, or more than one recurrence is considered an absolute contraindication to sports participation. Congenital stenosis (Pavlov ratio less than 0.8) without instability is not considered a contraindication to play. Congenital anomalies of the upper cervical spine are an absolute contraindication to participation in all contact sports. This includes os odontoideum, odontoid hypoplasia or aplasia, and atlantooccipital fusion, even if asymptomatic. During play, if neurological symptoms resolve quickly and the neurologic examination is normal with full motor strength, the patient may return to the game. Persistence of symptoms or lack of a pain-free range of motion requires further evaluation, including cervical spine radiographs. Players should be restricted from further play until they have recovered full muscle strength. Cervical disk herniations can have serious permanent neurologic complications. A disk bulge without herniation as demonstrated by MR imaging, can be treated conservatively with activity modification. Return to play may occur when pain-free full range of motion is demonstrated and radicular symptoms are completely resolved. Symptomatic disk herniation with cord or root impingement may require anterior diskectomy with interbody fusion. A limited fusion (one or two levels) of the subaxial cervical spine is not considered a contraindication to future play if the segments above and below the fusion are normal. A return to play cannot be recommended until there is radiographic evidence that the graft is well incorporated, the symptoms are completely resolved, and the player demonstrates a painless range of motion and full motor strength. With the exception of spear tackler Õs spine, there is no evidence that transient neurapraxia of the cord predisposes an individual to subsequent permanent quadriplegia or quadriparesis.
Reference(s)
Thomas BE, et al. Cervical spine injuries in football players. J AM Acad Orthop Surg 1999;7:338-347
Torg JS et al: Neurapraxia of the cervical spinal with transientquadriplegia. JBJS Am 1986:68:1354-
1370
back to this question next question
03.274
A 22-year old professional skier is unconscious after hitting a retaining fence during a race.
The patient’s helmet and face shield are intact. Appropriate initial management should consist of
-
Removal of the helmet to access the airway
-
Removal of the helmet with immediate transportation
-
Removal of the helmet followed by cervical spine immobilization
-
Removal of the shield to access the airway without immobilization
-
Immobilization of the cervical spine, leaving the helmet in place and removing the face shield.
-
back answer
Question 03.274
Answer = 5
Initial involvement of the orthopaedic surgeon in the care of an athlete with a cervical spine injury frequently begins on the field. Essential sideline equipment should include a spine board, a stretcher, and tools necessary to remove face masks from helmets and to perform cardiopulmonary resuscitation. Preparedness is paramount to timely, successful management. It is necessary to remove the face mask for airway control of the unconscious athlete while simultaneously protecting the cervical spine.
The chin strap and helmet are best left in place. The jaw thrust and chin lift are the safest ways of opening the airway in a patient with a suspected cervical injury. The head-tilt method is not considered safe. Transportation to a medical facility is necessary for the player with altered mental status, neck pain or tenderness, limited cervical motion, and symptoms referable to a cord injury. The patient should be fully immobilized on a spine board with helmet and shoulder pads remaining in place. Marked alterations in the position of the cervical vertebrae can occur with either helmet or
shoulder pad removal. If desired, cervical radiographs can be obtained with all of the protective gear still in place. The helmet should be removed only when permanent immobilization in a con-trolled setting can be instituted. At that time, the chin strap should be detached, and the ear flaps of the helmet spread. The helmet is gently pulled in line with the cervical spine while the head is supported under the occiput.
Reference(s)
Warren WL et al. On the field evaluation of athletic neck injuries. Clin Sports Med 1998:17:99-110
Torg JS: Athletic Injuries to the Head Neck and Face. Ed 2. St Louis, MO Mosby 1991 pp426-437
back to this question next question
03.275
What is the most common site for metastases from the synovial sarcoma?
-
Liver
-
Lung
-
Lymph Nodes
-
Bone
-
Synovium
back answer
Question 03.275
Answer = 2
Synovial sarcoma comprises 8-10% of all sarcomas and most commonly affects adults in the third-to-fifth decades of life. The malignancy most commonly involves the extremities, especially the lower extremities around the knees. Specific cytogenetic abnormalities have been identified. More than 90% of patients have a t(X;18) translocation mutation, which is not associated with other sarcomas. The translocation involves the SYTgene on chromosome 18 (at 18q11) and the SSX1or SSX2gene on the X chromosome (at Xp11). Synovial sarcoma is the fourth most commonly occurring sarcoma, accounting for 8-10% of all sarcomas. Approximately 800 new cases of synovial sarcoma are diagnosed in per year in the United States.
Overall, survival rates are 23.5-64% at 5 years and 11.2-34% at 10 years. For synovial sarcoma of the head and neck region, the prognosis is better than that of sarcoma involving the extremities, with 5-year survival rates of 47-82%. A slight male predilection exists in synovial sarcoma. The male-to-female ratio is 3:2. Synovial sarcoma can occur in patients with a wide age range, but it is most common in patients in the third-to-fifth decades of life. Most synovial sarcomas are found within 5 cm of a joint. Despite the misnomer, only 10% of cases are intra-articular. Synovial sarcomas are slow growing, but they can be locally aggressive.
Approximately one fourth of synovial sarcomas are associated with pulmonary metastases at the time of initial presentation. Although local control of sarcomas has improved with curative resection and adjuvant irradiation, metastases eventually develop in most patients. The sites most commonly involved are the lung and skeleton. Regional lymph nodes are involved in 10-23% of patients.
Reference(s)
Enzinger FM, Weiss SW (eds): Soft Tissue Tumors, ed 4 St Louis MO, Mosby, 2001 pp 21-44
back to this question next question